maths book part 2 obtains rs. 1100 after lending out rs.x at 5% per annum for 2 years and obtains...

221
1 Maths Book Part 2 By Abhishek Jain

Upload: doanduong

Post on 27-May-2018

216 views

Category:

Documents


0 download

TRANSCRIPT

Page 1: Maths book part 2 obtains Rs. 1100 after lending out Rs.x at 5% per annum for 2 years and obtains Rs. 1800 after lending out Rs.y at 10% per annum for 2 years. Find the value of x

1

Maths Book

Part 2

By Abhishek Jain

Page 2: Maths book part 2 obtains Rs. 1100 after lending out Rs.x at 5% per annum for 2 years and obtains Rs. 1800 after lending out Rs.y at 10% per annum for 2 years. Find the value of x

2

Topics 1. Simple interest and compound

interest 2. Probability

3. Data Interpretation 4. Mixtures & Alligation

5. Algebra 6. Geometry & mensuration

7. Triginometry & Height and distance

Page 3: Maths book part 2 obtains Rs. 1100 after lending out Rs.x at 5% per annum for 2 years and obtains Rs. 1800 after lending out Rs.y at 10% per annum for 2 years. Find the value of x

3

Simple&Compoundinterest

Simpleinterest:-Interest:TheamountofmoneythatyoupaytoborrowmoneyortheamountofmoneythatyouearnonadepositAnnualInterestRate:Thepercentofinterestthatyoupayformoneyborrowed,orearnformoneydepositedGeneralInformationS.I=!"#

$%%

1.Principle:Themoneyborrowedorlentoutforacertainperiodiscalledtheprincipalorthesum(P).2.Interest:Theborrowerpaysacertainamountfortheuseofthismoneyiscalledinterest(S.I).3.Time:TheborrowingisforaspecifiedperiodcalledTime(T).4.Rateofinterest:Thespecifiedtermisexpressedaspercentoftheprincipaliscalledrateofinterest(R%).5.Amount:ThesumoftheprincipalandtheinterestiscalledtheamountorFuturevalue.𝐴𝑚𝑜𝑢𝑛𝑡 = 𝑝𝑟𝑖𝑛𝑐𝑖𝑝𝑎𝑙 + 𝑖𝑛𝑡𝑒𝑟𝑒𝑠𝑡 = 𝑃 + !"#

$%%= 𝑃 1 + "#

$%%

Compoundinterest:-Wheninterestchargedoveraperiodoftimeisaddedupintheprincipal,theinterestsochargedonthisnewprincipaliscalledcompoundinterest.IfP=sumorPrincipaln=timeinyearsR=ratepercentperannum

Then,amount=P 1 + "$%%

9

(i)Wheninterestiscompoundedhalf-yearly,

Amount=P 1 +:;$%%

<9

(ii)Wheninterestiscompoundedquarterly,

Amount=P 1 +:=$%%

>9

Page 4: Maths book part 2 obtains Rs. 1100 after lending out Rs.x at 5% per annum for 2 years and obtains Rs. 1800 after lending out Rs.y at 10% per annum for 2 years. Find the value of x

4

(iii)Wheninterestiscompoundedannuallybuttimeinfraction,say2$?years.

Amount=P 1 + "$%%

<× 1 +

"AB$%%

$

(iv)Whenratesaredifferentfordifferentyears,sayR1%,R2%,andR3%for1st,2ndand3rdyearrespectivelythen,Amount=𝑃 1 + "$

$%%1 + "<

$%%1 + "C

$%%

(v)PresentworthofRs.Xduenyearshenceisgivenby:Presentworth=

D

$E :AFF

G

VeryImportantFormulae’sThedifferencebetweenthesimpleinterestandcompoundinterestfor2year(orterms)isgivenbytheformula

Difference=𝑃 "$%%

<

Thedifferencebetweenthesimpleinterestandcompoundinterestfor3year(orterms)isgivenbytheformula

Difference=𝑃 "; "EC%%$%%H

ConceptofEqualInstallmentsinCompoundinterest

P=𝑿

𝟏E 𝒓𝟏𝟎𝟎

𝒏 +𝑿

𝟏E 𝒓𝟏𝟎𝟎

(𝒏O𝟏) + ⋯……… . . + 𝑿

𝟏E 𝒓𝟏𝟎𝟎

P=PrincipalX=installmentR=rateN=numberofyearsEx.1.RobinlendsRs.9toRahulontheconditionthattheloanisrepayablein10monthsin10equalinstallmentsofRe.1each.Findtherateofinterestperannum.Sol.LettherateofinterestpermonthberTotalamountrepaid=Rs.10Interest=Re.1T$%%

(9+8+7+6+5+4+3+2+1)=1

Sor=$%%>?

Hence,therateofinterestperannum=($%%>?

)12=26<C%

Page 5: Maths book part 2 obtains Rs. 1100 after lending out Rs.x at 5% per annum for 2 years and obtains Rs. 1800 after lending out Rs.y at 10% per annum for 2 years. Find the value of x

5

Ex.2.AmilkmanborrowedRs.2500fromtwomoneylenders.Foroneloan,hepaid5%p.a.andfortheotherhepaid7%p.a.ThetotalinterestpaidfortwoyearswasRs.265.Howmuchdidheborrowat5%andhowmuchat7%?Sol.LettheloanbeRs.xat5%p.aandRs(2500–x)at7%p.a.

Totalinterestfor2years=[V×?×<E(<?%%WV)×X×<]

$%%

Givensimpleinterestforthetotalamount=Rs.265[V×?×<E(<?%%WV)×X×<]

$%%=265

Solving,wegetx=2125Amountborrowedat5%=Rs.2125Amountborrowedat7%=Rs.375Ex.3.ThreepersonsAmar,AkbarandAnthonyinvesteddifferentamountsinafixeddepositschemeforoneyearattherateof12%perannumandearnedatotalinterestofRs.3240attheendoftheyear.IftheamountinvestedbyAkbarisRs.5000morethantheamountinvestedbyAmarandtheamountinvestedbyAnthonyisRs.2000morethantheamountinvestedbyAkbar,whatistheamountinvestedbyAkbar?Sol.P=Amar+Akbar+Anthony=x+x+5000+x+7000=3x+12000

C<>%×$%%<

=3x+12000⇒27000-12000=3x⇒15000=3x⇒x=5000Akbar=5000+5000=Rs.10000Ex.4.VankatlaltakesmoneyfromtheEmployeesCooperativeSocietyatlowerrateofinterestandinvestsinascheme,whichgiveshimacompoundinterestof20%,compoundedannually.Findtheleastnumberofcompleteyearsafterwhichhissumwillbemorethandouble.Sol.LetP=Rs.100After4years,theamountwillbe100x1.2x1.2x1.2x1.2=Rs.207.3,whichismorethanthedoubleofRs.100Ex.5.RaminvestedacertainamountofmoneyandearnedacompoundinterestofRs.420inthesecondyearandRs.462inthethirdyear.AtwhatrateofinterestdidRaminvest?Sol.C.I.inthirdyear–C.I.insecondyear=462–420=42Thus,Rs.42istheinterestonRs.420.i.e.10%of420Hence,Rate=10%

Page 6: Maths book part 2 obtains Rs. 1100 after lending out Rs.x at 5% per annum for 2 years and obtains Rs. 1800 after lending out Rs.y at 10% per annum for 2 years. Find the value of x

6

Ex.6.Asumisinvestedatcompoundinterestpayableannually.TheinterestintwosuccessiveyearsstartingfromthefirstyearisRs.500andRs.540respectively.Findthesum.

Sol.Rate%=>%×$%%?%%×$

=8%?%%×$%%[×$

=Rs.6250Ex.7.SeemaclosedheraccountinaninvestmentoptionschemebywithdrawingRs.10000.Oneyearago,shehadwithdrawnRs.6000,twoyearsago,shehadwithdrawnRs.5000andthreeyearsago,shehadnotwithdrawnanymoney.Howmuchmoneyhadshedeposited(approx)atthetimeofopeningtheaccountfouryearsago,giventhatannualsimpleinterestis10%?Sol.Letxbethemoneyshedepositedatthetimeofopeningtheaccount.So,after1year(i.e.3yearsago)itwouldamountto1.1x.Sincenomoneywaswithdrawnatthispoint,after2years(i.e.2yearsago)itwouldamountto1.2x.Atthispoint,seemawithdrawsRs.5,000.Hence,herprincipalforthenextyear=(1.2x–5000)Nextyear,sheearns10%interestonthis,whichwillamountto1.1(1.2x–5000)Atthispoint,shewithdrawsRs.6,000.Hence,herprincipalforthenextyearwouldbe(1.32x–11500).Sheearns10%interestonthis,whichamountsto1.1(1.32x–11500)=(1.452x–12650)Butthisisequalto10000.Hence,x=15600Ex.8.Thesimpleinterestonasumofmoneyfor2yearsisRs.100andthecompoundinterestonthesamesumatthesamerateforthesametimeisRs.104.Findtherateofinterest.Sol.Simpleinterestfor2years=Rs.100Compoundinterestfor2years=Rs.104Simpleinterestfor1styearissameascompoundinterestfor1styear.Simpleinterestforbothyearsissame.So,S.I.for1year=Rs.50Compoundinterestforfirstyear=Rs.50Compoundinterestfor2ndyear=Rs.54ThismeansRs.4extra.So,whatpercentis4of50.or >

?%×100 = 8%

Ex.9.RaghavobtainsRs.1100afterlendingoutRs.xat5%perannumfor2yearsandobtainsRs.1800afterlendingoutRs.yat10%perannumfor2years.Findthevalueofx+y.Sol.HereCaseI:x+0.1x=1100CaseII:y+0.2y=1800Solvingtheabovetwoequations,wegetx+y=2500.

Page 7: Maths book part 2 obtains Rs. 1100 after lending out Rs.x at 5% per annum for 2 years and obtains Rs. 1800 after lending out Rs.y at 10% per annum for 2 years. Find the value of x

7

Ex.10.Thecompoundinterestandsimpleinterestoncertainsumfor2yearsarerespectivelyRs.41andRs.40.Findthesumandratepercent.Sol.LetthesumbeRs.P,rateofinterestber%AnnualS.I.=Rs.2020×1×

T$%%

=1⇒r=5%

P×1×?$%%

=20⇒P=Rs.400Ex.11.Mr.ThomasinvestedanamountofRs.13,900dividedintwodifferentschemesAandBatthesimpleinterestrateof14%p.a.and11%p.a.respectively.Ifthetotalamountofsimpleinterestearnedin2yearsbeRs.3508,whatwastheamountinvestedinSchemeB?Sol.LetthesuminvestedinSchemeAbeRs.xandthatinSchemeBbeRs.(13900-x).ThenD×$>×<

$%%+ ($C_%%WD)×$$×<

$%%=3508

⇒28x-22x=350800-(13900x22)⇒6x=45000⇒x=7500.So,suminvestedinSchemeB=Rs.(13900-7500)=Rs.6400.Ex.12.ReenatookaloanofRs.1200withsimpleinterestforasmanyyearsastherateofinterest.IfshepaidRs.432asinterestattheendoftheloanperiod,whatwastherateofinterest?Sol.Letrate=R%andtime=Ryears.Then$<%%×"×"

$%%=432

⇒12R2=432⇒R2=36⇒R=6.Ex.13.Anautomobilefinancierclaimstobelendingmoneyatsimpleinterest,butheincludestheinteresteverysixmonthsforcalculatingtheprincipal.Ifheischarginganinterestof10%,theeffectiverateofinterestbecomes:Sol.LetthesumbeRs.100.Then,SIFor6months=Rs.$%%×$%×$

$%%×<=Rs.5

SIForlast6months=Rs.$%?×$%×$

$%%×<=Rs.5.25

So,amountattheendof1year=Rs.(100+5+5.25)=Rs.110.25Effectiverate=(110.25-100)=10.25%Orshortcut=5+5+?×?

$%%=10.25%

Page 8: Maths book part 2 obtains Rs. 1100 after lending out Rs.x at 5% per annum for 2 years and obtains Rs. 1800 after lending out Rs.y at 10% per annum for 2 years. Find the value of x

8

Ex.14.AsumofRs.725islentinthebeginningofayearatacertainrateofinterest.After8months,asumofRs.362.50moreislentbutattheratetwicetheformer.Attheendoftheyear,Rs.33.50isearnedasinterestfromboththeloans.Whatwastheoriginalrateofinterest?Sol.LettheoriginalratebeR%.Then,newrate=(2R)%.Note:Here,originalrateisfor1year(s);thenewrateisforonly4monthsi.e.$

Cyear(s).

SoX<?×"×$

$%%+C`<.?×<"×$

$%%×C=33.5

Or2175+725R=33.5×100×3(2175+725)R=33.50x100x3(2175+725)R=10050(2900)R=10050R=10050/2900=3.46Sooriginalrate=3.46%Ex.15.AcertainamountearnssimpleinterestofRs.1750after7years.Hadtheinterestbeen2%more,howmuchmoreinterestwouldithaveearned?Sol.WeneedtoknowtheS.I.,principalandtimetofindtherate.Sincetheprincipalisnotgiven,sodataisinadequate.Ex.16.Thereis60%increaseinanamountin6yearsatsimpleinterest.WhatwillbethecompoundinterestofRs.12,000after3yearsatthesamerate?Sol.LetP=Rs.100.Then,S.I.Rs.60andT=6years.SoR=

$%%×`%$%%×`

=10%p.a.Now,P=Rs.12000.T=3yearsandR=10%p.a.

SoCI=Rs.12000 1 + $%$%%

C− 1 =Rs.3972

Ex.17.AlbertinvestedanamountofRs.8000inafixeddepositschemefor2yearsatcompoundinterestrate5p.c.p.a.HowmuchamountwillAlbertgetonmaturityofthefixeddeposit?Sol.

Amount=Rs.8000 1 + ?$%%

<=8000× <$

<%× <$<%=Rs.8820

Ex.18.Theeffectiveannualrateofinterestcorrespondingtoanominalrateof6%perannumpayablehalf-yearlyis:Sol.AmountofRs.100for1year

Whencompoundedhalfyearly=100 1 + C$%%

<=106.09

Page 9: Maths book part 2 obtains Rs. 1100 after lending out Rs.x at 5% per annum for 2 years and obtains Rs. 1800 after lending out Rs.y at 10% per annum for 2 years. Find the value of x

9

AmountofRs.100for1yearwhencompoundedhalf-yearly

=Rs.

100x

1+3

2

=Rs.106.09100

orEffectiverate=(106.09-100)%=6.09%Ex.19.Simpleinterestonacertainsumofmoneyfor3yearsat8%perannumishalfthecompoundinterestonRs.4000for2yearsat10%perannum.Thesumplacedonsimpleinterestis:Sol.

CI=Rs.Rs.4000 1 + $%$%%

<− 1 =Rs.840orsum=Rs.><%×$%%

C×[=Rs.1750

Ex.20.RiteshdepositedRs.8,000inStateBankofIndia.Hehadtopay10%ofthefirstyear’sinterestashiscollegefees,afterwhichRs.900oftheinterestamountwasleft.Findtheratepercentatwhichinterestwaspaid.Sol.Lettherate%bex.

S.I.=[%%%×D×b

$%%=[%%%D

$%%(t=1year)

10%ofinterestpaid⇒90%ofinterestleft=Rs.900(given)Socompleteinterest(100%)=Rs.1000

1000=[%%%D$%%

⇒x=

$%%[=12.5%

Ex.21.Asumwasputatsimpleinterestatacertainratefor4years.Haditbeenputat5%higherrate,itwouldhavefetchedRs.200more.Findthesum.Sol.Rs.200moreinterestisonaccountof5%higherrate.

200=c×?×>$%%

Ex.22.AsumofmoneyamountstoRs.767in3yearsandRs.806in4yearsatsimpleinterest.Findthesum.Sol.Interestfor1year=806–767=39Interestfor3year=117ThereforePrincipal=767–117=Rs.650Ex.23.Amanborrowsmoneyatsimpleinterestatarateof5%perannumandlendsitoutat4%perannumcompoundedhalfyearly.Findhisnetgainorlossattheendof2yearsasapercentageofsumborrowedbyhim.Sol.LetPrincipalbeRs.100AtSI,amount=Rs.110AtCI,amount=100×(1.02)4=108.24Thereforeloss%=1.76

Page 10: Maths book part 2 obtains Rs. 1100 after lending out Rs.x at 5% per annum for 2 years and obtains Rs. 1800 after lending out Rs.y at 10% per annum for 2 years. Find the value of x

10

Exercise

1.AcertainsumofmoneyatsimpleinterestamounttoRs.1040in3yearsandtoRs.1360in7years.Thenthatsumisa.Rs.800b.Rs.850c.Rs.820d.Rs.780e.Noneofthese2.OutofasumofRs.625,apartwaslentat5%andtheotherat10%simpleinterest.Iftheinterestonthefirstpartaftertwoyearsisequaltotheinterestonthesecondpartafterfouryears,thenthesecondsumisequaltoa.Rs.120b.Rs.122c.Rs.125d.Rs.100e.Noneofthese3.x,yandzarethreesumsofmoneysuchthatyisthesimpleinterestonxandzisthesimpleinterestonyforthesametimeandsamerateofinterest.Then,wehavea.y2=zb.x2=yzc.z2=xyd.y2=xze.Noneofthese4.AsumofRs.2668amountstoRs.4669in5yrattherateofsimpleinterest.Findtheratepercenta.10%b.35%c.20%d.12%e.Noneofthese5.Whatwillbesimpleinterestfor1yrand4monthsonasumofRs.25800attherateof14%perannum?a.Rs.4816b.Rs.4810c.Rs.4916d.Rs.4618e.Noneofthese6.Atwhatrateofannualsimpleinterest,acertainsumwillamounttofourtimesin15yr?a.10%b.15%c.20%d.12%e.Noneofthese7.Asumbecomesitsdoublein10yr.Findtheannualrateofsimpleinteresta.10%b.15%c.20%d.12%e.Noneofthese8.Atsimpleinterest,asumbecomes3timesin20yr.Findthetimeinwhichthesumwillbedoubleatthesamerateofinterest?a.10yrb.15yrc.20yrd.12yre.Noneofthese9.Acertainsumbecomes3foldat4%annualrateofinterest.Atwhatrate,itwillbecome6fold?a.10%b.15%c.20%d.12%e.Noneofthese10.AtasimpleinterestasumamountstoRs.1012in2.5yrandbecomesRs.1057.54in4yr.Whatistherateofinterest?a.2%b.3%c.5%d.4%e.Noneofthese11.FindthedifferenceinamountandprincipalforRs.4000attherateof5%annualinterestin4yra.Rs.800b.Rs.850c.Rs.820d.Rs.780e.Noneofthese

Page 11: Maths book part 2 obtains Rs. 1100 after lending out Rs.x at 5% per annum for 2 years and obtains Rs. 1800 after lending out Rs.y at 10% per annum for 2 years. Find the value of x

11

12.RakeshlentoutRs.8750at7%annualinterest.Findthesimpleinterestin3yra.Rs.1850b.Rs.1836.5c.Rs.1837.5d.Rs.1838e.Noneofthese13.PriyadepositsanamountofRs.65800toobtainasimpleinterestattherateof14%perannumfor4yr.WhattotalamountwillPriyagetattheendof4yr?a.Rs.102486b.Rs.102648c.Rs.103648d.Rs.102864e.Noneofthese14.Asumwaslentoutforacertaintimeatsimpleinterest.ThesumamountstoRs.400at10%annualinterestrate.Whenthesumwaslentoutat4%annualinterestrate,itamountstoRs.200.Findthesuma.Rs.<%%

Cb.Rs.$%%

Cc.Rs.<%%

Xd.Rs.>%%

Ce.Noneofthese

15.Asumwasinvestedfor4yratacertainrateofsimpleinterest,ifithadbeeninvestedat2%moreannualrateofinterest,whenRs.56morewouldhavebeenobtained.Whatisthesum?a.Rs.800b.Rs.720c.Rs.750d.Rs.700e.Noneofthese16.<

Cpartofmysumislentoutat3%,$

`partislentourat6%andremainingpartislentoutat

12%.Allthethreepartsarelentoutatsimpleinterest.IftheannualincomeisRs.25,whatisthesum?a.Rs.600b.Rs.550c.Rs.650d.Rs.520e.Noneofthese17.AsumofRs.1521islentoutintwopartsinsuchawaythattheinterestononepartat10%for5yrisequaltothatofanotherpartat8%for10yr.Whatwillbethetwopartsofsum?a.Rs.800andRs.885b.Rs.Rs.936andRs.585c.Rs.Rs.926andRs.585d.Rs.Rs.936andRs.595e.Noneofthese18.HarshamakesafixeddepositofRs.20000inBankofIndiaforaperiodof3yr.Iftherateofinterestbe13%slperannumchargedhalf-yearly,whatamountwillhegetafter42months?a.Rs.29000b.Rs.29200c.Rs.29100d.Rs.28100e.Noneofthese19.ThedifferenceofsimpleinterestfromtwobanksforRs.1000in2yrisRs.20.Findthedifferenceinratesofinteresta.2%b.3%c.5%d.4%e.Noneofthese20.SureshborrowedRs.800at6%andNareshborrowedRs.600at10%.Afterhowmuchtime,willtheybothhaveequaldebts?a.16<

Cyrb.16$

Cyrc.15<

Cyrd.15$

Cyre.Noneofthese

21.RajulentRs.400toAjayfor2yrandRs.100toManojfor4yrandreceivedfrombothRs60ascollectiveinterest.Findtherateofinterest,simpleinterestbeingcalculateda.2%b.3%c.5%d.4%e.Noneofthese

Page 12: Maths book part 2 obtains Rs. 1100 after lending out Rs.x at 5% per annum for 2 years and obtains Rs. 1800 after lending out Rs.y at 10% per annum for 2 years. Find the value of x

12

22.Rameshinvestedanamountthatis10%ofRs.10000atsimpleinterest.After3yr,theamountbecomesRs.2500.Findoutthe4timesofactualinterestratea.200%b.300%c.500%d.400%e.Noneofthese23.OnwhatsumofmoneywilltheS.Ifor4yearsat8%p.a.ishalfoftheS.IonRs.400for2yearsat10%p.a.?a.Rs.100b.Rs.120c.Rs.125d.Rs.144e.Noneofthese24.Anilborrowedcertainmoneyattherateof6%perannumforthefirst2yr,attherateof9%perannumforthenext3yrandattherateof14%perannumfor4yr.IfhepaysatotalinterestofRs.22800attheendof9yr,howmuchmoneydidheborrow?a.Rs.22000b.Rs.24000c.Rs.48000d.Rs.30000e.Noneofthese25.AsumofRs.500amountstoRs.650in3yratsimpleinterest.Iftheinterestrateisincreasedby3%,itwouldamounttohowmuch?a.Rs.690b.Rs.680c.Rs.685d.Rs.695e.Noneofthese26.Neetaborrowedsomemoneyattherateof6%perannumforthefirst3yr,attherateof9%perannumforthenext5yearandattherateof13%perannumfortheperiodbeyond8yr.IfshepaysatotalinterestofRs.8160attheendof11yr,howmuchmoneydidsheborrow?a.Rs.8000b.Rs.6000c.Rs.4000d.Rs.10000e.Noneofthese27.ReenahadRs.10000withheroutofthismoneyshelentsomemoneytoAkshayfor2yrat15%simpleinterest.ShelentremainingmoneytoBrijeshforanequalnumberofyearsattherateof18%.After2yr,ReenafoundthatAkshayhadgivenherRs.360moreasinterestascomparedtoBrijesh.TheamountofmoneywhichReenahadlenttoBrijeshmustbea.Rs.8000b.Rs.6000c.Rs.4000d.Rs.10000e.Noneofthese28.AsumofRs.800amountstoRs.956in3yratsimpleinterest.Iftheinterestrateisincreasedby3%,itwouldamounttohowmuch?a.Rs.1020b.Rs.1025c.Rs.1030d.Rs.1015e.Noneofthese29.Whatwillbetheratioofsimpleinterestearnedbycertainamountatthesamerateofinterestfor12yrandfor18yr?a.3:2b.2:3c.4:5d.5:4e.Noneofthese30.AsumofRs.1550waslentpartlyat5%andpartlyat8%perannumsimpleinterest.Thetotalinterestreceivedafter4yrwasRs.400.Theratioofthemoneylentat5%tothatlentat8%?a.16:15b.15:16c.17:9d.9:17e.Noneofthese31.TheannualpaymentofRs.160in5yrat5%perannumsimpleinterestwilldischargeadebtofa.Rs.800b.Rs.600c.Rs.400d.Rs.880e.Noneofthese

Page 13: Maths book part 2 obtains Rs. 1100 after lending out Rs.x at 5% per annum for 2 years and obtains Rs. 1800 after lending out Rs.y at 10% per annum for 2 years. Find the value of x

13

32.Rameshlentout40%ofacertainsumattheannualrateof15%,helent50%oftheremainingattheannualrateof10%andtherestamountwaslentoutat18%,perannum.Findtheannualrateonwholesuma.14%b.13%c.14.4%d.datainadequatee.Noneofthese33.AprivatefinancecompanyAclaimstobelendingmoneyatsimpleinterest.Butthecompanyincludestheinterestevery6monthsforcalculatingprincipal.IfcompanyAischarginganinterestof10%,theeffectiverateofinterestafter1yrbecomesa.10.5%b.10.25%c.10.75%d.11%e.Noneofthese34.RajnishinvestedcertainsuminthreedifferentschemesP,QandRwiththeratesofinterest10%perannum,12%perannumand15%perannum,respectively.Ifthetotalinterestaccruedin1yrwasRs.3200andtheamountinvestedinschemeRwas150%oftheamountinvestedinschemePand240%oftheamountinvestedinschemeQ.WhatwastheamountinvestedinschemeQ?a.Rs.8000b.Rs.6000c.Rs.4000d.Rs.5000e.Noneofthese35.Thesimpleinterestonasumofmoneyat9%perannumfor5yrishalfthesum,isa.Rs.80b.Rs.60c.Rs.40d.datainadequatee.Noneofthese36.AanchalborrowedRs.500at3%perannumSlandRs.600at4.5%perannumSlontheagreementthatthewholesumwillbereturnedonlywhenthetotalinterestbecomesRs.252.Thenumberofyearsafterwhichtheborrowedsumistobereturned,isa.2yrb.3yrc.5yrd.6yre.Noneofthese37.Thesimpleinterestonasumofmoneyat8%perannumfor6yrishalfthesum.Whatisthesum?a.Rs.800b.Rs.600c.Rs.400d.datainadequatee.Noneofthese38.Theratesofsimpleinterestintwobanksxandyareintheratioof10:8.Ranjiwantstodeposithistotalsavingsintwobanksinsuchawaythatshereceivesequalhalf-yearlyinterestfromboth.Sheshoulddepositthesavingsinbanksxandyintheratioofa.3:2b.2:3c.4:5d.5:4e.Noneofthese39.Asumbecomes6foldat5%perannum.Atwhatrate,thesumbecomes12fold?a.10.5%b.10.25%c.10.75%d.11%e.Noneofthese40.Asumofmoneybecomes9timesin20yr.Findthe10timesofrateofinteresta.200%b.300%c.500%d.400%e.Noneofthese41.ThesimpleinterestonasumofmoneywillbeRs.200after5yr.Inthenext5yr,principleistripled.Whatwillbethetotalinterestattheendofthe10thyr?a.Rs.800b.Rs.600c.Rs.400d.900e.Noneofthese

Page 14: Maths book part 2 obtains Rs. 1100 after lending out Rs.x at 5% per annum for 2 years and obtains Rs. 1800 after lending out Rs.y at 10% per annum for 2 years. Find the value of x

14

42.WhatmustbetheprincipalthatamountstoRs.720in2years6monthsas5%perannumsimpleinterest?a.Rs.800b.Rs.640c.Rs.700d.840e.Noneofthese43.Acertainsumgivenonsimpleinterestbecamedoublein20yrs.Inhowmanyyearswillitbefourtimes?a.20yrb.30yrc.50yrd.60yre.Noneofthese44.FindoutthecapitalrequiredtoearnamonthlyinterestofRs.600permonthas6%simpleinteresta.Rs.1,80,000b.Rs.1,60,000c.Rs.1,40,000d.Rs.1,20,000e.Noneofthese45.AmanderiveshisincomefromaninvestmentofRs.2,000atacertainrateofinterestandRs.1,600at2%higher.Thewholeinterestin3yrsisRs.960.Findtherateofinteresta.8%b.6%c.5%d.4%e.Noneofthese46.AsumofRs.1,550waslentpartlyat5%andpartlyat8%simpleinterest.Thetotalinterestreceivedafter3yrswasRs.300.Theratioofmoneylentat5%to8%isa.13:12b.12:13c.16:15d.15:16e.Noneofthese47.Rs.793isdividedintothreepartsassuchthattheiramountafter2,3,and4yrsmaybeequaltherateofinterestbeing5%.Findratiobetweentheseparts

a.<$$%

: $$$?

: $$<%

b.$$$%

: $$$?

: $$<%

c.C$$%

: $$$?

: $$<%

d.C$$%

: <$$?

: $$<%

e.Noneofthese48.IftheC.Ionacertainsumfor3yearsat20%p.a.isRs.728,whatisthesuminvested?a.1500b.800c.1200d.1000e.Noneofthese49.AmanbuysahouseandpaysRs.8,000cashandRs.9,600at5yearscreditat4%perannumsimpleinterest.Findthecashpriceofthehousea.Rs.18000b.Rs.16000c.Rs.14000d.Rs.12000e.Noneofthese50.FindthesimpleinterestonRs.600from3rdMarchto15thMayofayearat6%p.a.a.Rs.7.2b.Rs.7c.Rs.6.5d.6e.Noneofthese51.AsumofRs.2,600islentintwopartssothattheinterestonthefirstpartforaperiodof3yearsat5%maybeequaltotheinterestonthesecondpartfor6yearsat4%.Thesecondpartisequaltoa.Rs.1250b.Rs.1200c.Rs.1000d.Rs.1100e.Noneofthese52.Asumdoublesin20yearsatsimpleinterest.Howmuchistherate?a.8%b.6%c.5%d.4%e.Noneofthese

Page 15: Maths book part 2 obtains Rs. 1100 after lending out Rs.x at 5% per annum for 2 years and obtains Rs. 1800 after lending out Rs.y at 10% per annum for 2 years. Find the value of x

15

53.AsumofRs.5984becomesRs.8976in6yearsatSI.Whatistherate?a.8$

C%p.a.b.8<

C%p.a.c.7$

C%p.a.d.8$

?%p.a.e.Noneofthese

54.Thesimpleinterestonacertainprincipal@4%p.afor5yearsisRs.800.Howmuchistheprincipalamount?a.Rs.8000b.Rs.6000c.Rs.4000d.Rs.2000e.Noneofthese55.AcertainsumofmoneybecomesRs.1250inaspanof5yearsatsimpleinterestandfurthertoRs.1700inthespanof8years.Atthesamerate,whatwoulditamounttoattheendof12years?a.Rs.2100b.Rs.2500c.Rs.2300d.Rs.2000e.Noneofthese56.Thesimpleinterestearnedonacertainsumofmoneyfor10yearsattherateof5%p.a.washalfthesum.Howmuchisthesum?a.Rs.3000b.Rs.5000c.Rs.4000d.datainadequatee.Noneofthese57.Rs.800becomesRs.956in3yearsatcertainsimplerateofinterest.Iftherateofinterestisincreasedby4%,whatamountwillRs.800becomein3years?a.Rs.1050b.Rs.1052c.Rs.1060d.Rs.1048e.Noneofthese58.Srinivasaninveststwoequalamountsintwobanksgiving10%and12%rateofinterestrespectively.AttheendofyeartheinterestearnedisRs.1650.Findthesuminvestedineacha.Rs.7000b.Rs.7200c.Rs.7500d.Rs.5000e.Noneofthese59.ThesimpleinterestonasumofmoneywillbeRs.600after10years.Iftheprincipalistrebledafter5years,whatwillbetotalinterestattheendofthetenthyear?a.Rs.1000b.Rs.1200c.Rs.1400d.Rs.800e.Noneofthese60.ApersoninvestsRs.5000at5%p.a.simpleinterestsforacertainperiodandearnsRs.750.IfheearnsRs.720onRs.6000inthesametimeperiodwhatistherateofinterest?a.8%b.6%c.5%d.4%e.Noneofthese61.TherateofinterestatwhichanamountofRs.1800oncompoundinterestbecomesRs.1984.50in2yearisa.8%b.6%c.5%d.4%e.Noneofthese62.WhichistheprincipalamountwhichearnsRs.132ascompoundinterestforthesecondyearat10%perannum?a.Rs.1000b.Rs.1200c.Rs.1400d.Rs.800e.Noneofthese63.TheamountofRs.7500atcompoundinterestat4%perannumfor2years,isa.Rs.8112b.Rs.8110c.Rs.8100d.Rs.8200e.Noneofthese

Page 16: Maths book part 2 obtains Rs. 1100 after lending out Rs.x at 5% per annum for 2 years and obtains Rs. 1800 after lending out Rs.y at 10% per annum for 2 years. Find the value of x

16

64.Thedifferenceincompoundinterestandsimpleinterestonacertainamountat10%perannumattheendofthethirdyearisRs.620.Whatistheprincipalamount?a.Rs.30000b.Rs.50000c.Rs.40000d.Rs.20000e.Noneofthese65.Tofindoutthetotalcompoundinterestaccruedonasumofmoneyafter5years,whichofthefollowinginformationgiveninthestatementsPandQwillbesufficient?P:ThesumwasRs.20,000Q:Thetotalamountofsimpleinterestonsamesumatsamerateofinterestandforthesametimeis2000a.onlyPneededb.onlyQneededc.BothPandQareneededd.answercannotbeansweredevenbyusingbothPandQe.Noneofthese66.Thedifferencebetweencompoundinterestandthesimpleinterestearnedonasumofmoneyattheendof4yearsisRs.256.40.Tofindoutthesum,whichofthefollowinginformationgiveninthestatementsPandQis/arenecessary?P:Amountofsimpleinterestaccruedafter4years500Q:Rateofinterestis10%perannuma.onlyPneededb.onlyQneededc.BothPandQareneededd.answercannotbeansweredevenbyusingbothPandQe.Noneofthese67.Tofindoutthetotalcompoundinterestaccruedonasumofmoneyafter5years,whichofthefollowinginformationgiveninthestatements.AandBis/aresufficient?P:Therateofinterestwas6%perannumQ:Thetotalsimpleinterestonthesameamountafter5yearsatthesameratewillbeRs.600a.onlyPneededb.onlyQneededc.BothPandQareneededd.answercannotbeansweredevenbyusingbothPandQe.Noneofthese68.AsumofmoneyinvestedatcompoundinterestamountstoRs.800in3yearsandRs.840in4years.Whatistherateofinterestforperannum?a.8%b.6%c.5%d.4%e.Noneofthese69.AmanborrowedRs.800at10%perannumsimpleinterestandimmediatelylentthewholesumat10%perannumcompoundinterest.Whatdoeshegainattheendof2years?a.Rs.8b.Rs.6c.Rs.5d.Rs.4e.Noneofthese

Page 17: Maths book part 2 obtains Rs. 1100 after lending out Rs.x at 5% per annum for 2 years and obtains Rs. 1800 after lending out Rs.y at 10% per annum for 2 years. Find the value of x

17

70.Thecompoundinterestonacertainsumat5%perannumfor2yearsisRs.328.Thesimpleinterestforthatsumatthesamerateandforthesameperiodwillbea.Rs.300b.Rs.320c.Rs.340d.Rs.360e.Noneofthese71.AsumofmoneyatcompoundinterestamountstoRs.578.40in2yearsandtoRs.614.55in3years.Therateofinterestperannumisa.6C

>%b.5$

>%c.6$

>%d.6?

>%e.Noneofthese

72.Acertainamountofmoneyisinvestedatthesimpleinterestof15%perannum.Ifithadbeeninvestedatcompoundinterest,anextrainterestofRs.450wouldhavebeenobtainedinthesecondyear.Whatmustbetheamountinvested?a.Rs.30000b.Rs.50000c.Rs.40000d.Rs.20000e.Noneofthese73.WhatisthedifferencebetweentheCIandSIonasumofRs.1600at5%p.a.forperiodof2years?a.Rs.8b.Rs.6c.Rs.5d.Rs.4e.Noneofthese74.WhatisthedifferencebetweenthecompoundinterestandthesimpleinterestonacapitalofRs.16,000attherateof15%perannumforaperiodof2years?a.Rs.480b.Rs.360c.Rs.350d.Rs.450e.Noneofthese75.AtwhatrateofinterestperannumwouldthedifferencebetweenthecompoundinterestandthesimpleinterestattheendoftwoyearsonthecapitalofRs.60000beRs.1944?a.18%b.16%c.15%d.14%e.Noneofthese76.Thedifferencebetweencompoundinterestandsimpleinterestonanamountforaperiodof1$

<yearsis62rupees.Rateofinterestis20%perannuminbothcasesandincaseof

compoundinteresttheinterestisbeingcompoundedhalfyearly.Whatistheamount?a.Rs.3000b.Rs.5000c.Rs.4000d.Rs.2000e.Noneofthese77.Theincomeofacompanyincreases20%perannum.IfitsincomeisRs.26,64,000intheyear1999,whatwasitsincomeintheyear1997?a.Rs.1850000b.Rs.1800000c.Rs.1852000d.Rs.1950000e.Noneofthese78.Thedifferencebetweenthesimpleinterestonacertainsumattherateof10%perannumfor2yearsancompoundinterestwhichiscompoundedevery6monthsisRs.124.05.Whatistheprincipalsum?a.Rs.7000b.Rs.8000c.Rs.9000d.Rs.10000e.Noneofthese79.Thedifferencebetweenthecompoundinterestandsimpleinterestonacertainsumat5%for2yearsisRs.1.50.Thesumisa.Rs.300b.Rs.500c.Rs.400d.Rs.600e.Noneofthese

Page 18: Maths book part 2 obtains Rs. 1100 after lending out Rs.x at 5% per annum for 2 years and obtains Rs. 1800 after lending out Rs.y at 10% per annum for 2 years. Find the value of x

18

80.Thedifferentbetweenthecompoundinterestandsimpleinterestonacertainsumofmoneyfor2yearsat10%perannumisRs.15.Findthesumofmoneya.Rs.1300b.Rs.1500c.Rs.1400d.Rs.1600e.Noneofthese81.Exchangerateofdollarvsrupeeincreasesattherateof10%permonth.IfthecurrentrateisRs.40perdollar,whatwillbetherateattheendofamonth?a.Rs.45b.Rs.44c.Rs.40d.Rs.41e.Noneofthese82.BalanborrowedRs.1,000at10percentperannumsimpleinterest.Heimmediatelylentthewholesumat10percentperannumcompoundinterest.Attheendof2years,hewouldgaina.Rs.30b.Rs.50c.Rs.10d.Rs.20e.Noneofthese83.AcertainamountearnssimpleinterestofRs.1750after7years.Hadtheinterestbeen2%more,howmuchmoreinterestwouldithaveearned?a.Rs.135b.Rs.125c.Rs.124d.cannotbedeterminede.Noneofthese84.HowmuchwillRs.25000amounttoin2yearsatcompoundinterest,iftheratesforthesuccessiveyearsbe4and5percentperyear?a.Rs.27500b.Rs.27300c.Rs.27400d.Rs.27600e.Noneofthese85.WhatisthedifferencebetweencompoundinterestandsimpleinterestforthesumofRs.2000overa2yearperiod,ifthecompoundinterestiscalculatedat20%andsimpleinterestiscalculatedat23%?a.Rs.45b.Rs.44c.Rs.40d.Rs.41e.Noneofthese86.FindthecompoundinterestofRs.1000attherateof20%perannumfor18monthswheninterestiscompoundedhalf-yearlya.Rs.331b.Rs.325c.Rs.330d.Rs.320e.Noneofthese87.Atwhatpercentageperannum,willRs.10,000amounttoRs.17280inthreeyears?(Compoundinterestbeingreckoned)a.20%b.30%c.50%d.40%e.Noneofthese88.VinaydepositedRs.8000inICICIBank,whichpayshim12%interestperannumcompoundedquarterly.Whatistheamountthathereceivesafter15months?a.Rs.9750b.Rs.9730c.Rs.9740d.Rs.9760e.Noneofthese89.RanjeetmakesadepositofRs.50,000inthePunjabNationalBankforaperiodof2$

<years.

Iftherateofinterestis12%perannumcompoundedhalf-yearly,findthematurityvalueofthemoneydepositedbyhim(approx.)a.Rs.66900b.Rs.66800c.Rs.66911d.Rs.66800e.Noneofthese90.VinodmakesadepositofRs.1,00,000intheSyndicateBankforaperiodof2years.Iftherateofinterestbe12%perannumcompoundedhalf-yearly,whatamounthewillgetafter2

Page 19: Maths book part 2 obtains Rs. 1100 after lending out Rs.x at 5% per annum for 2 years and obtains Rs. 1800 after lending out Rs.y at 10% per annum for 2 years. Find the value of x

19

years(approx.)a.Rs.126000b.Rs.125000c.Rs.126250d.Rs.126247e.Noneofthese91.AsumofmoneyisborrowedandpaidbackintwoequalannualinstalmentsofRs.882,allowing5%compoundinterest.Thesumborrowedwasa.Rs.1540b.Rs.1740c.Rs.1640d.Rs.1660e.Noneofthese92.Ifthedifferencebetweenthesimpleinterestandcompoundinterestonsomeprincipalamountat20%perannumfor3yearsisRs.48,thentheprincipalamountmustbea.Rs.275b.Rs.350c.Rs.375d.Rs.325e.Noneofthese93.Asumofmoneydoublesitselfin5years.Inhowmanyyearswillitbecomefourfold(ifinterestiscompounded)?a.8yrb.9yrc.6yrd.10yre.Noneofthese94.Ifthecompoundinterestonacertainsumat10%perannumfor2yearsisRs.21.Whatcouldbethesimpleinterest?a.Rs.20b.Rs.30c.Rs.40d.Rs.10e.Noneofthese95.Ifthecompoundinterestonacertainsumofmoneyfor2yearsat10%isRs.25200,findthesimpleinterestatthesamerateforthesametimea.Rs.25000b.Rs.24600c.Rs.24000d.Rs.23000e.Noneofthese96.ThedifferencebetweenCIandSIonasumofmoneyfor3yearsat5%perannumisRs.61.Findthesuma.Rs.7000b.Rs.8000c.Rs.9000d.Rs.10000e.Noneofthese97.Asumofmoneydoublesitselfatcompoundinterestin15years.Inhowmanyyearsitwillbecomeeighttimes?a.40b.35c.45d.50e.Noneofthese98.AsumofRs.400amountstoRs.441in2years.Whatwillitamounttoiftherateofinterestisincreasedby5%?a.Rs.480b.Rs.484c.Rs.450d.Rs.425e.Noneofthese99.Ifthedifferencebetweenthecompoundinterest,compoundedeverysixmonths,andthesimpleinterestonacertainsumofmoneyattherateof12%perannumforoneyearisRs.36,thesumisa.Rs.7000b.Rs.8000c.Rs.9000d.Rs.10000e.Noneofthese100.AbuilderborrowsRs.2550tobepaidbackwithcompoundinterestattherateof4%perannumbytheendof2yearsintwoequalyearlyinstalments.Howmuchwilleachinstalmentbe?a.Rs.1350b.Rs.1340c.Rs.1352d.Rs.1360e.Noneofthese

Page 20: Maths book part 2 obtains Rs. 1100 after lending out Rs.x at 5% per annum for 2 years and obtains Rs. 1800 after lending out Rs.y at 10% per annum for 2 years. Find the value of x

20

Answers1.A 2.C 3.D 4.B 5.A 6.C 7.A 8.A 9.A 10.B11.A 12.C 13.B 14.A 15.D 16.E 17.B 18.C 19.E 20.A21.C 22.A 23.C 24.B 25.D 26.A 27.C 28.E 29.B 30.A31.D 32.D 33.B 34.D 35.D 36.D 37.D 38.C 39.D 40.D41.A 42.B 43.D 44.D 45.A 46.C 47.B 48.D 49.B 50.A51.C 52.C 53.A 54.C 55.C 56.D 57.B 58.C 59.B 60.D61.C 62.B 63.A 64.D 65.C 66.B 67.D 68.C 69.A 70.B71.C 72.D 73.D 74.B 75.A 76.D 77.A 78.B 79.D 80.B81.B 82.C 83.D 84.B 85.C 86.A 87.A 88.E 89.C 90.D91.C 92.C 93.D 94.A 95.C 96.B 97.C 98.B 99.D 100.C

Page 21: Maths book part 2 obtains Rs. 1100 after lending out Rs.x at 5% per annum for 2 years and obtains Rs. 1800 after lending out Rs.y at 10% per annum for 2 years. Find the value of x

21

Permutation&combinationThefactorialfunction(symbol:!)meanstomultiplyaseriesofdescendingnaturalnumbers.Examples:

4!=4×3×2×1=247!=7×6×5×4×3×2×1=50401!=1

PERMUTATIONSApermutationis"are-arrangementofelementsofaset".So,whatdoesthismean?ItmeansapermutationisONLYinterestedinre-arrangingtheelementsofthesetAnyduplicationofthecollectedelementsindifferentordersisfine.Apermutationthereforetendstobealargenumber.Example:Takingthe4letters,ABCD,writedownallthepermutationsof3oftheseleters:ABCBACCABDABACBBCACBADBAABDBADCADDACADBBDACDADCAACDBCDCBDDBCADCBDCCDBDCBorthereare24permutations.Inotherwords,justtakingeachletterandcollectingthemintosetsof3fromthe4andwritingthemout,gives24variations.Done.Here,ifyoulike,theordermatters,sinceABCisdifferenttoACBanddifferenttoBCAanddifferenttoCABetc.Permutationsseetheseasalldifferentanswers.COMBINATIONSAcombinationis"oneormoreelementsselectedfromasetwithoutregardtotheorder"The"withoutregard"meansthatthecollectionmattersratherthanorderincombinations,sointheaboveexample,thefactweABC,ACB,BAC,BCA,CAB,CBA...forcombinations,theseareall1combinationoflettersA,BandC.So,questionsconcerningpickingateamof5peoplefromasquadof11...youwouldneedcombinations,sinceitishaving"Bert,Ernie,Fred,BillandBob"thatmatters,notthefactthatyouhavesomanydifferentpermutationsofthese5people.

Page 22: Maths book part 2 obtains Rs. 1100 after lending out Rs.x at 5% per annum for 2 years and obtains Rs. 1800 after lending out Rs.y at 10% per annum for 2 years. Find the value of x

22

Example:Takingthe4letters,ABCD,writedownallthecombinationsof3oftheseletters:ABC,ABD,ACD,BCDortherearejust4combinations.Youcannotpickanyother3lettersfromABCD,thatisnotpartoftheabove4combinations.Itisenlighteningtoseethelettermissingineach:inorderwehave"noD","noC","noB"andfinally"noA"...thissometimeshelpsyouto"see"allthepossibleanswers.Tocalculatecombinations,itisa2-stageprocess:1.Youcalculatealltheequivalentpermutationsfirst.2.Youthencorrectthislistbycuttingoutanyduplicates.Asyoucanseefromthis,combinationsareasubsetofPermutations.MathematicsofPermutationsTofindthenumberofpermutationsofrelementsfromasetofn,theformulais:

P(n,r)=9!9WT !

So,theaboveexamplewouldbeMathematicsofCombinationsTofindthenumberofcombinationsofkelementsfromasetofn,theformulais:

C(n,r)=9!

T!× 9WT !

Ex.1.Inhowmanywayscanthelettersoftheword“PREUNIVERSITY”bearranged?

Solution:Numberofways=𝟏𝟑!

𝟐!×𝟐!×𝟐!

Becausethereare13lettersinwhichthereare2R’s,2’Eand2I’s.Ex.2.Howmanyeightletterwordscanbeformedfromthelettersoftheword“Courtesy”,beginningwithCandendingwithY?Sol.ThefirstandthelastplacearefixedwithCandYrespectively.Theremaining6letterscanbearrangedin6!Ways.Therefore,thetotalnumberofways=720.Ex.3.Outof8gentlemenand5ladies,acommitteeof5istobeformed.Findthenumberofwaysinwhichthiscanbedonesoastoincludeatleast3ladies.Sol.Thepossiblecombinationsfulfillingthegivenconditionsare:3ladiesand2gentlemen,4ladiesand1gentlemen,andallthe5ladies.a)3ladiesand2gentlemencanbeselectedin:⁵C₃X⁸C₂=280waysb)4ladiesand1gentlemancanbeselectedin:⁵C₄X⁸C₁=40waysc)5ladiescanbeselectedin1waysThetotalnumberofwaysthecommitteecanbeformed=321ways

Page 23: Maths book part 2 obtains Rs. 1100 after lending out Rs.x at 5% per annum for 2 years and obtains Rs. 1800 after lending out Rs.y at 10% per annum for 2 years. Find the value of x

23

Ex.4.Howmanydifferent7digittelephonenumberscanbeformedfrom0,1,2,3,4,5,6,7,8,9?Sol.Since0cannotbethefirstdigitinthe7-digittelephonenumber,thereare9differentpossibilitiesforthefirstposition.Theremaining6digitscanbeselectedfromthe10digitsin10⁶ways.Thetotalnumberofways=10⁶x9(Rememberthatintelephonenumbers,digitscanberepeated.Hence,foreverysingledigit,exceptforthefirstposition,thereare10differentpossibilities)Ex.5.From3mangoes,4applesand2oranges,howmanyselectionsoffruitscanbemade,takingatleastoneofeachkind?Sol.Youmaybetemptedtoadopttheapproachwetookinthepreviousquestion.However,thereisnoconditiongivenregardingthemaximumnumberoffruitsinselection.Hence,wehavetotryadifferenttechnique:Numberofwaysinwhichmangoescanbeselected=2³.Butthisalsoincludesthecasewhereallthreemangoesarenotselected.Hence,numberofwaysinwhichatleastonemangoesisselected=(2³-1)=7Similarly,numberofapplescanbeselectedin(2⁴–1)ways=15numberoforangescanbeselectedin(2²–1)ways=3Thetotalnumberofwaysis15x3x7=315ways(wehavetomultiplyandnotsimplyadd)Ex.6.Inhowmanydifferentwayscanthelettersoftheword'LEADING'bearrangedinsuchawaythatthevowelsalwayscometogether?Sol.Theword'LEADING'has7differentletters.WhenthevowelsEAIarealwaystogether,theycanbesupposedtoformoneletter.Then,wehavetoarrangethelettersLNDG(EAI).Now,5(4+1=5)letterscanbearrangedin5!=120ways.Thevowels(EAI)canbearrangedamongthemselvesin3!=6ways.

Requirednumberofways=(120x6)=720.Ex.7.Inhowmanydifferentwayscanthelettersoftheword'CORPORATION'bearrangedsothatthevowelsalwayscometogether?Sol.Intheword'CORPORATION',wetreatthevowelsOOAIOasoneletter.Thus,wehaveCRPRTN(OOAIO).Thishas7(6+1)lettersofwhichRoccurs2timesandtherestaredifferent.Numberofwaysarrangingtheseletters=7!/2!=2520Now,5vowelsinwhichOoccurs3timesandtherestaredifferent,canbearrangedIn5!/3!=20ways.SoRequirednumberofways=(2520x20)=50400.

Page 24: Maths book part 2 obtains Rs. 1100 after lending out Rs.x at 5% per annum for 2 years and obtains Rs. 1800 after lending out Rs.y at 10% per annum for 2 years. Find the value of x

24

Ex.8.Inhowmanywayscanthelettersoftheword'LEADER'bearranged?Sol.Theword'LEADER'contains6letters,namely1L,2E,1A,1Dand1R.Requirednumber=`!

<!=720/2=360

Ex.9.Howmany3-digitnumberscanbeformedfromthedigits2,3,5,6,7and9,whicharedivisibleby5andnoneofthedigitsisrepeated?Sol.Sinceeachdesirednumberisdivisibleby5,sowemusthave5attheunitplace.So,thereis1wayofdoingit.Thetensplacecannowbefilledbyanyoftheremaining5digits(2,3,6,7,9).So,thereare5waysoffillingthetensplace.Thehundredsplacecannowbefilledbyanyoftheremaining4digits.So,thereare4waysoffillingit.Requirednumberofnumbers=(1x5x4)=20Ex.10.Howmanymultiplesof5aretherefrom10to95?Sol.Asyouknow,multiplesof5areintegershaving0or5inthedigittotheextremeright(i.e.theunit’splace).Thefirstdigitfromtherightcanbechosenin2ways.Theseconddigitcanbeanyoneof1,2,3,4,5,6,7,8,9i.e.Thereare9choicesfortheseconddigit.Thus,thereare2×9=18multiplesof5from10to95.Ex.11.Inacity,thebusroutenumbersconsistofanaturalnumberlessthan100,followedbyoneofthelettersA,B,C,D,EandF.Howmanydifferentbusroutesarepossible?Sol.Thenumbercanbeanyoneofthenaturalnumbersfrom1to99.Thereare99choicesforthenumber.Thelettercanbechosenin6ways.Numberofpossiblebusroutesare99×6=594Ex.12.Thereare3questionsinaquestionpaper.Ifthequestionshave4,3and2solutionsrespectively,findthetotalnumberofsolutions.Sol.Herequestion1has4solutions,question2has3solutionsandquestion3has2solutions.Bythemultiplication(counting)rule,Totalnumberofsolutions=4×3×2=24Ex.13.ConsiderthewordROTOR.Whicheverwayyoureadit,fromlefttorightorfromrighttoleft,yougetthesameword.Suchawordisknownaspalindrome.Findthemaximumpossiblenumberof5-letterpalindromes.Sol.Thefirstletterfromtherightcanbechosenin26waysbecausethereare26alphabets.Havingchosenthis,thesecondlettercanbechosenin26ways.Thefirsttwoletterscanbechosenin26×26=676waysHavingchosenthefirsttwoletters,thethirdlettercanbechosenin26ways.Allthethreeletterscanbechosenin676×26=17576ways.Itimpliesthatthemaximumpossiblenumberoffiveletterpalindromesis17576becausethefourthletteristhesameasthesecondletterandthefifthletteristhesameasthe

Page 25: Maths book part 2 obtains Rs. 1100 after lending out Rs.x at 5% per annum for 2 years and obtains Rs. 1800 after lending out Rs.y at 10% per annum for 2 years. Find the value of x

25

firstletter.Ex.14.Howmany3-digitnumberscanbeformedwiththedigits1,4,7,8and9ifthedigitsarenotrepeated?Sol.Threedigitnumberswillhaveunit’s,ten’sandhundred’splace.Outof5givendigitsanyonecantaketheunit’splace.Thiscanbedonein5ways-------(i)Afterfillingtheunit’splace,anyofthefourremainingdigitscantaketheten’splace.Thiscanbedonein4ways-------(ii)Afterfillinginten’splace,hundred’splacecanbefilledfromanyofthethreeremainingdigits.Thiscanbedonein3ways-------(iii)Bycountingprinciple,thenumberof3digitnumbers=5×4×3=60Ex.15.SupposeyoucantravelfromaplaceAtoaplaceBby3buses,fromplaceBtoplaceCby4buses,fromplaceCtoplaceDby2busesandfromplaceDtoplaceEby3buses.InhowmanywayscanyoutravelfromAtoE?Sol.ThebusfromAtoBcanbeselectedin3ways.ThebusfromBtoCcanbeselectedin4ways.ThebusfromCtoDcanbeselectedin2ways.ThebusfromDtoEcanbeselectedin3ways.So,bytheGeneralCountingPrinciple,onecantravelfromAtoEin3×4×2×3=72Ex.16.SupposeyouwanttoarrangeyourEnglish,Hindi,Mathematics,History,GeographyandSciencebooksonashelf.Inhowmanywayscanyoudoit?Sol.Wehavetoarrange6books.Thenumberofpermutationsofnobjectsisn!=n.(n–1).(n–2)...2.1Heren=6andtherefore,numberofpermutationsis6.5.4.3.2.1=720Ex.17.Suppose7studentsarestayinginahallinahostelandtheyareallotted7beds.Amongthem,ParvindoesnotwantabednexttoAnjubecauseAnjusnores.Then,inhowmanywayscanyouallotthebeds?Sol.Letthebedsbenumbered1to7.Case1:SupposeAnjuisallottedbednumber1.Then,Parvincannotbeallottedbednumber2.SoParvincanbeallottedabedin5ways.AfterallottingabedtoParvin,theremaining5studentscanbeallottedbedsin5!ways.So,inthiscasethebedscanbeallottedin5×5!=600ways.Case2:Anjuisallottedbednumber7.Then,Parvincannotbeallottedbednumber6AsinCase1,thebedscanbeallottedin600ways.Case3:Anjuisallottedoneofthebedsnumbered2,3,4,5or6ParvincannotbeallottedthebedsontherighthandsideandlefthandsideofAnju’sbed.Forexample,ifAnjuisallottedbednumber2,bedsnumbered1or3cannotbeallottedtoParvin.Therefore,Parvincanbeallottedabedin4waysinallthesecases.

Page 26: Maths book part 2 obtains Rs. 1100 after lending out Rs.x at 5% per annum for 2 years and obtains Rs. 1800 after lending out Rs.y at 10% per annum for 2 years. Find the value of x

26

AfterallottingabedtoParvin,theother5canbeallottedabedin5!ways.Therefore,ineachofthesecases,thebedscanbeallotted4×5!=480ways.soThebedscanbeallottedin:2×600+5×480=1200+2400=3600waysEx.18.Inhowmanywayscanananimaltrainerarrange5lionsand4tigersinarowsothatnotwolionsaretogether?Sol.Theyhavetobearrangedinthefollowingway:|L|T|L|T|L|T|L|T|L|The5lionsshouldbearrangedinthe5placesmarked‘L’.Thiscanbedonein5!ways.The4tigersshouldbeinthe4placesmarked‘T’.Thiscanbedonein4!ways.Therefore,thelionsandthetigerscanbearrangedin5!×4!=2880waysEx.19.Thereare4booksonfairytales,5novelsand3plays.Inhowmanywayscanyouarrangethesesothatbooksonfairytalesaretogether,novelsaretogetherandplaysaretogetherandintheorder,booksonfairytales,novelsandplays.Sol.Thereare4booksonfairytalesandtheyhavetobeputtogether.Theycanbearrangedin4!ways.Similarly,thereare5novels.Theycanbearrangedin5!ways.Andthereare3plays.Theycanbearrangedin3!ways.So,bythecountingprincipleallofthemtogethercanbearrangedin4!×5!×3!=17280waysEx.20.Inhowmanywayscan4girlsand5boysbearrangedinarowsothatallthefourgirlsaretogether?Sol.Let4girlsbeoneunitandnowthereare6unitsinall.Theycanbearrangedin6!ways.Ineachofthesearrangements4girlscanbearrangedin4!ways.orTotalnumberofarrangementsinwhichgirlsarealwaystogether=6!×4!=720×24=17280Ex.21.Howmanyarrangementsofthelettersoftheword‘BENGALI’canbemade(i)Ifthevowelsarenevertogether.(ii)Ifthevowelsaretooccupyonlyoddplaces.Sol.Thereare7lettersintheword‘Bengali;ofthese3arevowelsand4consonants.(i)Consideringvowelsa,e,iasoneletter,wecanarrange4+1lettersin5!waysineachofwhichvowelsaretogether.These3vowelscanbearrangedamongthemselvesin3!ways.orTotalnumberofwords=5!×3!or120×6=720Sotherearetotalof720waysinwhichvowelsareALWAYSTOGEGHER.Now,Sincetherearenorepeatedletters,thetotalnumberofwaysinwhichthelettersoftheword‘BENGALI’canbearranged:

Page 27: Maths book part 2 obtains Rs. 1100 after lending out Rs.x at 5% per annum for 2 years and obtains Rs. 1800 after lending out Rs.y at 10% per annum for 2 years. Find the value of x

27

or7!=5040So,Totalno.ofarrangementsinwhichvowelsarenevertogether:orALLthearrangementspossible–arrangementsinwhichvowelsareALWAYSTOGETHERor5040–720=4320(ii)Thereare4oddplacesand3evenplaces.3vowelscanoccupy4oddplacesinP(4,3)waysand4constantscanbearrangedinP(4,4)itmeansNumberofwords=P(4,3)×P(4,4)=4!×4!=576Ex.22.12pointslieonacircle.Howmanycyclicquadrilateralscanbedrawnbyusingthesepoints?Sol.Foranysetof4pointswegetacyclicquadrilateral.Numberofwaysofchoosing4pointsoutof12pointsisC(12,4)=495Therefore,wecandraw495quadrilaterals.

Page 28: Maths book part 2 obtains Rs. 1100 after lending out Rs.x at 5% per annum for 2 years and obtains Rs. 1800 after lending out Rs.y at 10% per annum for 2 years. Find the value of x

28

Exercise1.Whatisthevalueof18C16?a.140b.135c.145d.150e.Noneofthese2.HowmanysixletteredwordsstartingwiththeletterTcanbemadefromallthelettersofthewordTRAVEL?a.120b.135c.130d.150e.Noneofthese3.Thenumberof2digitevennumbersformedfromthedigits1,2,3,4,5and6ifrepetitionofdigitsisnotalloweda.12b.13c.14d.15e.Noneofthese4.Howmanydifferentwordscanbeformedbytakinganythreelettersformtheword‘CHEMISTRY’?a.512b.502c.504d.500e.Noneofthese5.6studentsappearinanexamination.Inhowmanywayscantheresultbeannounced?a.32b.64c.128d.36e.Noneofthese6.Inhowmanyways4boysand3girlscanbeseatedinarowsothatboysandgirlsarealternate?a.132b.164c.128d.144e.Noneofthese7.Whatwillbethesumofallthefourdigitnumbersthatcanbemadewiththedigits0,1,2and4?(Repetitionofdigitsinanumberisnotallowed)a.45100b.45102c.45108d.44100e.Noneofthese8.Inhowmanydifferentways3differentringscanbeworninfivefingersofahand?a.210b.140c.60d.280e.Noneofthese9.Inhowmanydifferentwayscanthelettersofthefollowingbearrangedsothatthevowelsmayoccupyonlytheoddposition“DETECT”a.45b.36c.120d.60e.Noneofthese10.Howmanyfourdigitnumberscanbemadewiththedigits0,1,2and7sothatatleastoneofthedigitsisrepeatedineverynumber?a.144b.164c.158d.174e.Noneofthese11.Atotalof66gameswereplayedinatournamentwhereeachplayerplayedoneagainsttherest.Thenumbersofplayersarea.10b.12c.11d.13e.Noneofthese

Page 29: Maths book part 2 obtains Rs. 1100 after lending out Rs.x at 5% per annum for 2 years and obtains Rs. 1800 after lending out Rs.y at 10% per annum for 2 years. Find the value of x

29

12.Inhowmanywayscanyouselect2oddnumbersand2evennumbersoutofthefirst128wholenumbers?a.64C2х63C2b.64C2х64C2c.63C2х63C2d.65C2х63C2e.Noneofthese13.Thereare13couples,5singlemalesand7singlefemalesinaparty.Everymaleshakehandwitheveryfemaleoncebutnooneshakeshandwithhiswife.Howmanyhandshakestookplaceintheparty?a.340b.345c.347d.346e.Noneofthese14.Howmanynumbersaretherebetween100and1000suchthateverydigitiseither4or5?a.10b.8c.9d.11e.Noneofthese15.FindthetotalnumberofwordsthatcanbemadebyusingallthelettersfromthewordMACHINEusingonlyoncea.5!b.6!c.7!/2!d.7!e.Noneofthese16.Howmanyfivedigitnumberscanbeformedusingthedigits0,1,2,3,4and5whicharedivisibleby5,withoutrepeatingthedigits?a.216b.120c.96d.288e.Noneofthese17.Howmanyoddintegersfrom0to8000(inclusive)havedistinctdigits?a.1700b.1740c.1736d.1750e.Noneofthese18.Thereare5boysand6girls.Acommitteeof4istobeselectedsothatitmustconsistatleastoneboyandatleastonegirl?a.310b.312c.300d.320e.Noneofthese19.Apalindromeisanumberthatreadsthesamelefttorightasitdoesfromrighttoleft,suchas131.Howmany6-digitspalindromesaretherewhichareeven?a.200b.400c.300d.500e.Noneofthese20.HowmanydifferentwordscanbemadefromthewordEDUCATIONsothatallthevowelsarealwaystogether?(Donotbotheraboutanymeaninglesswords)a.14400b.12100c.16900d.14200e.Noneofthese21.Fivedistinctpairsofshoesaredisplaced.Inhowmanydifferentwayscanthreeshoesbeselectedcontainingamatchedpair?a.20b.40c.30d.50e.Noneofthese22.Fromaclassof12students5aretobechosenforanexcursion.But3veryclosefriendsdecideamongthemselvesthateitherallthreeofthemwillgoornoneofthemwillgo.Inhowmanywayscantheexcursionpartybechosen?a.144b.165c.169d.162e.Noneofthese

Page 30: Maths book part 2 obtains Rs. 1100 after lending out Rs.x at 5% per annum for 2 years and obtains Rs. 1800 after lending out Rs.y at 10% per annum for 2 years. Find the value of x

30

23.Howmanyevennumbersoffourdigitscanbeformedwiththedigits1,2,3,4,5,6(repetitionsofdigitsareallowed)?a.640b.642c.648d.646e.Noneofthese24.ThelettersofthewordLABOURarepermutedinallpossiblewaysandthewordsthusformedarearrangedasinadictionary.WhatistherankofthewordsLABOUR?a.242b.240c.246d.244e.Noneofthese25.Apersonwantstoselect2toysforthischild.Onevarietyoftoyshas9modelsandanothervarietyhas6models.Inhowmanywayscanheselectthe2toysonefromeachofthevariety?a.44b.54c.52d.48e.Noneofthese26.Findthevalueof9C5a.120b.117c.130d.126e.Noneofthese27.If(28)C(2r):(24)C[2(r-4)]=225:11,findra.8b.5c.2d.9e.Noneofthese28.Howmanymotorvehicleregistrationnumbersof4digitscanbeformedwiththedigits0,1,2,3,4,5(nodigitbeingrepeated)?a.200b.400c.300d.500e.Noneofthese29.Inhowmanywayscanthelettersoftheword‘POSSESS’bearrangedsothatthefourS’sareinalternatepositionsonly?a.8b.7c.6d.9e.Noneofthese30.Inhowmanywavescanacommitteeof3menand2womenbeformedoutofatotalof4menand4women?a.24b.25c.26d.23e.Noneofthese31.7C2+5C1+8C3-7C5-5C4-8C5a.2b.0c.4d.5e.Noneofthese32.Thetotalnumberofpermutationoftheword‘KOLKATA’willbea.1242b.1240c.1260d.1244e.Noneofthese33.Whatisthenumberofdiagonalsofaregularpolygonwith10sides?a.45b.35c.36d.55e.Noneofthese34.Asixfaceddie,aneightfaceddieandatenfaceddiearethrowntogether.Whatistheprobablenumberofoutcomes?a.240b.460c.480d.500e.Noneofthese

Page 31: Maths book part 2 obtains Rs. 1100 after lending out Rs.x at 5% per annum for 2 years and obtains Rs. 1800 after lending out Rs.y at 10% per annum for 2 years. Find the value of x

31

35.Inaboxthereare5distinctwhiteand6distinctblackballs.Apersonhastopickuptwoballsfromtheboxsuchthatthereisoneeachofboththecolours.Inhowmanywayshecanpicktheballs?a.20b.40c.30d.50e.Noneofthese36.Howmanytrianglescanbeformedbyjoiningtheverticesofaheptagon?a.45b.35c.36d.55e.Noneofthese37.Inaculturalfestival,sixprogrammersaretobestaged,threeonadayfortwodays.Inhowmanywaystheprogrammerscouldbearranged?a.720b.640c.680d.780e.Noneofthese38.Alltheoddnumbersfrom1to9arewrittenineverypossibleorder.Howmanynumberscanbeformedifrepetitionisnorallowed?a.240b.120c.480d.360e.Noneofthese39.Howmanynumberslyingbetween3000and4000andmadewiththedigits3,4,5,6,7and8aredivisibleby5?Repetitionsarenotallowed?a.24b.12c.48d.36e.Noneofthese40.Inameetingbetweentwocountrieseachcountryhas12delegates.Allthedelegatesofonecountryshakehandswithalldelegatesoftheothercountry.Findthenumberofhandshakespossible?a.144b.165c.169d.162e.Noneofthese41.Inthepreviousproblemifallthedelegatesshakehandwitheachotherirrespectiveofthecountrytheybelongtothentotalnumberofhandshakesisa.270b.265c.288d.276e.Noneofthese42.FivepersonsA,B,C,D,EoccupyseatsinarowsuchthatAandBsitnexttoeachother.Inhowmanypossiblewayscanthesefivepeoplesit?a.24b.12c.48d.36e.Noneofthese43. Thereare5boysand6girls.Acommitteeof4istobeselectedsothatitmustconsistatleastoneboyandatleastonegirl?a.310b.120c.480d.360e.Noneofthese44.Findthenumberofdiagnolsinhexagon?a.8b.7c.6d.9e.Noneofthese45.Howmanynumbersdivisibleby2andlyingbetween50000to70000canbeformed,fromthedigits3,4,5,6,7,8,9nodigitbeingrepeatedinanynumber?a.240b.120c.480d.360e.Noneofthese

Page 32: Maths book part 2 obtains Rs. 1100 after lending out Rs.x at 5% per annum for 2 years and obtains Rs. 1800 after lending out Rs.y at 10% per annum for 2 years. Find the value of x

32

46.Howmanydiagonalsarethereinann-sidedpolygon(n›3)?a.n(n-3)/2b.n(n-2)/2c.n(n-3)/6d.(n-1)(n-3)/2e.Noneofthese47.Aclassinaschoolhas40students.Threestudentsofthisclassaretobeselectedasclassmonitor,gamesinchargeandlibrarian.Inhowmanywayscantheybeselected,ifastudentcanholdonlyonepositionatatime?a.59820b.59280c.59082d.59028e.Noneofthese48.Sherrybuys7novelsfromabookfair.Merrybuys8novelsfromthefair,noneofwhichiscommonwiththoseboughtbysherry.Theydecidetoexchangetheirbooksoneforone.Inhowmanywayscantheyexchangetheirbooksforthefirsttime?a.56b.54c.52d.58e.Noneofthese49.Howmany6-digitevennumberscanbeformedfromthedigits1,2,3,4,5,6and7sothatthedigitsshouldnotrepeatandthesecondlastdigitiseven?a.480b.360c.720d.960e.Noneofthese50.Outof7consonants4vowels,howmanywordsof3consonantsand2vowelscanbeformed?a.14400b.22500c.25600d.25200e.Noneofthese51.Howmany3-letterswordswithorwithoutmeaning,canbeformedoutofthelettersoftheword,'LOGARITHMS',ifrepetitionoflettersisnotallowed?a.720b.120c.1440d.240e.Noneofthese52.Apolygonhas54diagonals.Findthenumberofsidesa.14b.16c.15d.12e.Noneofthese53.Fourdicearerolledsimultaneously.Whatisthenumberofpossibleoutcomesinwhichatleastoneofthedieshows5?a.1296b.671c.625d.125e.Noneofthese54.ThefigurebelowshowsthenetworkconnectingcitiesA,B,C,D,EandF.Thenarrowsindicatepermissibledirectionoftravel.WhatisthenumberofdistinctpathsfromAtoF?

a.8b.9c.10d.12e.Noneofthese

Page 33: Maths book part 2 obtains Rs. 1100 after lending out Rs.x at 5% per annum for 2 years and obtains Rs. 1800 after lending out Rs.y at 10% per annum for 2 years. Find the value of x

33

55.Theproductofanyrconsecutivepositiveintegersisdivisiblebywhichofthefollowing?a.r!b.(r-1)!c.(r+1)!d.both(a)and(b)e.Noneofthese56.Inhowmanywayscan3childreninafamilyhavealldifferentbirthdays?a.366х365х364b.365х364х363c.366х365х363d.365х365х364e.Noneofthese57.Inhowmanywayscanaleapyearhave53Sundays?a.3b.4c.2d.1e.Noneofthese58.Inhowmanywayscan5lettersbepostedin3postboxes,ifanynumberofletterscanbepostedinallofthethreepostboxes?a.125b.243c.25d.81e.Noneofthese59.Ontheir10thweddinganniversaryaBengalicouplebought10differentsweetsandthendistributeditbetweentwooftheirfamilyfriendssuchthatbothofthemgotfivesweetseach.Findthenumberofdifferentwaysinwhichthisdistributioncanbedonea.252b.25c.100d.125e.Noneofthese60.InthecountryofUtopiathelanguagecontainsonly4alphabets.FindthemaximumnumberofwordsthatcanbethereintheUtopiandictionaryifnoalphabetcanberepeatedinaworda.68b.69c.64d.60e.Noneofthese61.Acompanycouldadvertiseaboutitsnewproductin4magazines,3newspapersand2televisionchannels.Butinalatermoveitdecidedtogiveadvertisementsinonly2ofthemagazines,1ofthenewspapersand1oftheTVchannels.Inhowmanywayscantheyadvertisetheirproduct?a.48b.36c.73d.96e.Noneofthese62.Thefirst5oddnaturalnumbersarewrittenineverypossibleorder.Howmanynumberscanbeformedifnorepetitionisallowed?a.4!b.5!c.6!d.7!e.Noneofthese63.Inhowmanydifferentwaysonecanwear3differentringsinfingersofonehand?a.210b.60c.120d.180e.Noneofthese64.Inastaircasethereare3steps.Apersoncanjumponestep,twostepsorthreesteps.Inhowmanywayscanhereachthetop?a.4b.5c.6d.7e.Noneofthese65.Howmanywayscan5prizesbegivenawayto4boys,ifeachboyiseligibleforalltheprizes?a.54b.53c.45d.44e.Noneofthese

Page 34: Maths book part 2 obtains Rs. 1100 after lending out Rs.x at 5% per annum for 2 years and obtains Rs. 1800 after lending out Rs.y at 10% per annum for 2 years. Find the value of x

34

66.Aftergroupdiscussionandinterview6candidateswereselectedforadmissioninacollege.Butunfortunatelythenumberofseatsleftis2.So,itwasleftwiththeprincipaltoselect2candidatesoutofthem.Inhowmanywayscanheselect2candidates?a.14b.15c.16d.17e.Noneofthese67.Inanexamination10questionsaretobeansweredchoosingatleast4fromeachofpartAandpartB.Ifthereare6questionsinpartAand7inpartB,inhowmanywayscan10questionsbeanswered?a.248b.236c.273d.266e.Noneofthese68.AfteraconditioningcampofIndianCricketteam,thefinalteamof11playersoutofatotalof15playersistobedecidedsuchthattwoplayersSouravandSachinisalwayschosen.Findthetotalnumberofwaysthefinalteamcanbeselecteda.714b.715c.716d.717e.Noneofthese

Answers1.E 2.A 3.D 4.C 5.B 6.D 7.C 8.A 9.B 10.D11.B 12.A 13.C 14.B 15.D 16.A 17.C 18.A 19.B 20.A21.B 22.D 23.C 24.A 25.B 26.D 27.E 28.C 29.C 30.A31.B 32.C 33.B 34.C 35.C 36.B 37.A 38.B 39.B 40.A41.D 42.C 43.A 44.D 45.D 46.A 47.B 48.A 49.C 50.D51.A 52.D 53.B 54.C 55.D 56.A 57.C 58.B 59.A 60.C61.B 62.B 63.A 64.A 65.C 66.B 67.D 68.B

Page 35: Maths book part 2 obtains Rs. 1100 after lending out Rs.x at 5% per annum for 2 years and obtains Rs. 1800 after lending out Rs.y at 10% per annum for 2 years. Find the value of x

35

ProbabilityConceptofProbability:Probabilitydealswiththeanalysisofrandomphenomena.Itisawayofassigningeveryeventavaluebetweenzeroandone,withtherequirementthattheeventmadeupofallpossibleresultsisassignedavalueofone.Experiment:Anoperationwhichcanproducesomewell-definedoutcomesiscalledanexperimentRandomExperiment:Anexperimentinwhichallpossibleoutcomesareknowandtheexactoutputcannotbepredictedinadvance,iscalledarandomexperiment.Examples:

i. Rollinganunbiaseddice.ii. Tossingafaircoin.iii. Drawingacardfromapackofwell-shuffledcards.iv. Pickingupaballofcertaincolourfromabagcontainingballsofdifferentcolours.

Details:

i. Whenwethrowacoin,theneitheraHead(H)oraTail(T)appears.ii. Adiceisasolidcube,having6faces,marked1,2,3,4,5,6respectively.Whenwethrow

adie,theoutcomeisthenumberthatappearsonitsupperface.iii. Apackofcardshas52cards.

Ithas13cardsofeachsuit,nameSpades,Clubs,HeartsandDiamonds.Cardsofspadesandclubsareblackcards.Cardsofheartsanddiamondsareredcards.Thereare4honoursofeachunit.ThereareKings,QueensandJacks.Theseareallcalledfacecards.SampleSpace:Whenweperformanexperiment,thenthesetSofallpossibleoutcomesiscalledthesamplespace.Examples:

1. Intossingacoin,S={H,T}2. Iftwocoinsaretossed,theS={HH,HT,TH,TT}.3. Inrollingadice,wehave,S={1,2,3,4,5,6}.

Event:Anysubsetofasamplespaceiscalledanevent.

Page 36: Maths book part 2 obtains Rs. 1100 after lending out Rs.x at 5% per annum for 2 years and obtains Rs. 1800 after lending out Rs.y at 10% per annum for 2 years. Find the value of x

36

ProbabilityofOccurrenceofanEvent:LetSbethesampleandletEbeanevent.Then,P(E)=n(E)/n(S)Ex.1.Ticketsnumbered1to20aremixedupandthenaticketisdrawnatrandom.Whatistheprobabilitythattheticketdrawnhasanumberwhichisamultipleof3or5?Sol.Here,S={1,2,3,4,....,19,20}.LetE=eventofgettingamultipleof3or5={3,6,9,12,15,18,5,10,20}.P(E)=n(E)/n(S)=9/20Ex.2.Abagcontains2red,3greenand2blueballs.Twoballsaredrawnatrandom.Whatistheprobabilitythatnoneoftheballsdrawnisblue?Sol.Totalnumberofballs=(2+3+2)=7.LetSbethesamplespace.Thenn(S)=numberofwaysofdrawing2ballsoutof7=7C2=21LetE=Eventofdrawing2balls,noneofwhichisblue.Thenn(E)=numberofwaysofdrawing2ballsoutof(2+3)balls=5C2=10SoP(E)=10/21Ex.3.Inabox,thereare8red,7blueand6greenballs.Oneballispickeduprandomly.Whatistheprobabilitythatitisneitherrednorgreen?Sol.Totalnumberofballs=(8+7+6)=21.LetE =eventthattheballdrawnisneitherrednorgreen

=eventthattheballdrawnisblue.

Son(E)=7P(E)=n(E)/n(S)=7/21=1/3Ex.4.Whatistheprobabilityofgettingasum9fromtwothrowsofadice?Sol.Intwothrowsofadie,n(S)=(6x6)=36.LetE=eventofgettingasum={(3,6),(4,5),(5,4),(6,3)}.P(E)=n(E)/n(S)=4/36=1/9Ex.5.Threeunbiasedcoinsaretossed.Whatistheprobabilityofgettingatmosttwoheads?Sol.HereS={TTT,TTH,THT,HTT,THH,HTH,HHT,HHH}LetE=eventofgettingatmosttwoheads.ThenE={TTT,TTH,THT,HTT,THH,HTH,HHT}.P(E)=n(E)/n(S)=7/8

Page 37: Maths book part 2 obtains Rs. 1100 after lending out Rs.x at 5% per annum for 2 years and obtains Rs. 1800 after lending out Rs.y at 10% per annum for 2 years. Find the value of x

37

Ex.6.Twodicearethrownsimultaneously.Whatistheprobabilityofgettingtwonumberswhoseproductiseven?Sol.Inasimultaneousthrowoftwodice,wehaven(S)=(6x6)=36.ThenE={(1,2),(1,4),(1,6),(2,1),(2,2),(2,3),(2,4),(2,5),(2,6),(3,2),(3,4),(3,6),(4,1),(4,2),(4,3),(4,4),(4,5),(4,6),(5,2),(5,4),(5,6),(6,1),(6,2),(6,3),(6,4),(6,5),(6,6)}P(E)=n(E)/n(S)=27/36=3/4Ex.7.Acardisdrawnfromapackof52cards.Theprobabilityofgettingaqueenofcluborakingofheartis:Sol.Here,n(S)=52.LetE=eventofgettingaqueenofcluborakingofheart.Then,n(E)=2.P(E)=n(E)/n(S)=2/52=1/26Ex.8.Abagcontains4white,5redand6blueballs.Threeballsaredrawnatrandomfromthebag.Theprobabilitythatallofthemarered,is:Sol.LetSbethesamplespace.Thenn(S)=numberofwaysofdrawing3ballsoutof15=15C3=

$?×$>×$CC×<×$

= 455LetE=eventofgettingallthe3redballs.Son(E)=5C3=5C2=10orP(E)=10/455=2/91Ex.9.Twocardsaredrawntogetherfromapackof52cards.Theprobabilitythatoneisaspadeandoneisaheart,is:Sol.LetSbethesamplespace.Thenn(S)=52C2=

?<×?$<×$

= 1326LetE=eventofgetting1spadeand1heart.Son(E)=numberofwaysofchoosing1spadeoutof13&1heartoutof13=13C1×13C1=13×13=169HenceP(E)=n(E)/n(S)=169/1326=13/102Ex.10.Onecardisdrawnatrandomfromapackof52cards.Whatistheprobabilitythatthecarddrawnisafacecard(Jack,QueenandKingonly)?Sol.Clearly,thereare52cards,outofwhichthereare12facecards.P(gettingafacecard)=12/52=3/13

Page 38: Maths book part 2 obtains Rs. 1100 after lending out Rs.x at 5% per annum for 2 years and obtains Rs. 1800 after lending out Rs.y at 10% per annum for 2 years. Find the value of x

38

Ex.11.Abagcontains6blackand8whiteballs.Oneballisdrawnatrandom.Whatistheprobabilitythattheballdrawniswhite?Sol.Letnumberofballs=(6+8)=14.Numberofwhiteballs=8.P(drawingawhiteball)=8/14=4/7Ex.12.Inaclass,thereare15boysand10girls.Threestudentsareselectedatrandom.Theprobabilitythat1girland2boysareselected,is:Sol.LetSbethesamplespaceandEbetheeventofselecting1girland2boys.Thenn(S)=numberofwaysofselceting3studentsoutof25=25C3=

<?×<>×<CC×<×$

= 2300n(E)=10C1×15C2=1050sop(E)=n(E)/n(S)=1050/2300=21/46Ex.13.Inalottery,thereare10prizesand25blanks.Alotteryisdrawnatrandom.Whatistheprobabilityofgettingaprize?Sol.p(gettingaprize)=n(E)/n(S)=10/35=2/7Ex.14.Fromapackof52cards,twocardsaredrawntogetheratrandom.Whatistheprobabilityofboththecardsbeingkings?Sol.LetSbethesamplespace.n(S)=52C2=

?<×?$<×$

= 1326letE=eventofgetting2kingsoutof4son(E)=4C2=

>×C<×$

= 6sop(E)=n(E)/n(S)=5/1326=1/221Ex.15.Twodicearetossed.Theprobabilitythatthetotalscoreisaprimenumberis:Sol.Clearly,n(S)=(6x6)=36.LetE=Eventthatthesumisaprimenumber.ThenE=={(1,1),(1,2),(1,4),(1,6),(2,1),(2,3),(2,5),(3,2),(3,4),(4,1),(4,3),(5,2),(5,6),(6,1),(6,5)}Son(E)=15sop(E)=n(E)/n(S)=15/36=5/12

Page 39: Maths book part 2 obtains Rs. 1100 after lending out Rs.x at 5% per annum for 2 years and obtains Rs. 1800 after lending out Rs.y at 10% per annum for 2 years. Find the value of x

39

Exercise1.Fromapackof52cards,twocardsaredrawnatrandom.Whatistheprobabilitythateitherbothareredorbotharequeensisa.55/221b.52/221c.52/243d.56/221e.Noneofthese2.Aboxcontains20ballsbearingnumbers1,2,3…..20.Aballisdrawnatrandomfromthebox.Whatistheprobabilitythatthenumberontheballisdivisibleby2or3?a.9/20b.11/20c.17/20d.13/20e.Noneofthese3.Abagcontains2red,5blueand7greenballs.Aballisdrawnatrandom.Whatistheprobabilitythatitiseitherablueoragreenball?a.5/7b.5/14c.6/7d.3/7e.Noneofthese4.Fromapackofcardstwocardsaredrawnatrandom.Whatistheprobabilitythateitherbotharekingsorbotharequeens?a.3/221b.4/221c.5/221d.2/221e.Noneofthese5.Thereare8blueand4whiteballsinabag.Aballisdrawnatrandom.Withoutreplacingitanotherballisdrawn.Findtheprobabilitythatboththeballsdrawnarebluea.12/35b.14/33c.15/33d.17/33e.Noneofthese6.Thereare5greenand6blackballsinabag.Aballisdrawnatrandomwithreplacement.Findtheprobabilitythatboththeballsdrawnareblacka.36/121b.14/121c.15/121d.12/121e.Noneofthese7.Onecardisdrawnatrandomfromapackofcards.Findtheprobabilitythatthecarddrawniseitherakingoraqueen?a.3/26b.13/51c.2/13d.1/26e.Noneofthese8.Alotof12bulbscontains4defectivebulbs.Threebulbsaredrawnatrandomfromthelot,oneaftertheother.Theprobabilitythatallthreearenon-defectiveisa.11/56b.11/28c.15/56d.17/28e.Noneofthese9.100studentsappearedfortwoexaminations,60passedthefirst,50passedthesecondand30passedboth.Theprobabilitythatastudentselectedatrandomhasfailedinbothexaminationsisa.1/5b.2/5c.3/5d.4/5e.Noneofthese10.6boysand6girlssitinarowrandomly,findtheprobabilitythatallthe‘6’girlssittogether.a.3/132b.1/132c.5/132d.7/132e.Noneofthese11.Twodicearethrownsimultaneously.Whatistheprobabilityofgettingtwonumberswhoseproductiseven?a.1/5b.3/4c.1/4d.4/15e.Noneofthese

Page 40: Maths book part 2 obtains Rs. 1100 after lending out Rs.x at 5% per annum for 2 years and obtains Rs. 1800 after lending out Rs.y at 10% per annum for 2 years. Find the value of x

40

12.Ina6faceddiewithnumbers1to6inscribed.Findtheprobabilityofgettinga2digitnumberwhichisprimeonadding2successivethrowsofthediea.2/5b.2/6c.5/6d.3/5e.Noneofthese13.Outof20consecutivepositiveintegers,twoarechosenatrandom.Theprobabilitythattheirsumisoddisa.11/19b.10/17c.13/19d.10/19e.Noneofthese14.Whentwodicearerolled,whatistheprobabilitythatthesumofthenumbersappearedonthemis11?a.1/18b.2/19c.3/17d.4/5e.Noneofthese15.Theprobabilityofdrawingaredcardfromadeckofplayingcards,isa.1/13b.2/13c.1/26d.1/2e.Noneofthese16.Fivecoinsaretossedatatime.Thentheprobabilityofobtainingatleastonetailisa.15/32b.31/32c.1/2d.15/16e.Noneofthese17.Ifa4digitnumberisformedatrandomusingthedigits1,3,5,7,9withoutrepetition,thentheprobabilitythatitisdivisibleby5isa.1/5b.2/5c.3/5d.4/5e.Noneofthese18.Theprobabilityofgettingacompositenumberwhenasix-facedunbiaseddieistossed,isa.1/5b.2/5c.2/3d.1/3e.Noneofthese19.Twounbiaseddicearethrownsimultaneously.Theprobabilityofgettingthesumdivisibleby3isa.2/3b.1/3c.5/12d.5/6e.Noneofthese20.Anumberisselectedatrandomfromtheset{1,2,3…..50}.Theprobabilitythatnisaprimeisa.1/5b.2/5c.3/10d.7/10e.Noneofthese21.Ifthreeunbiasedcoinsaretossedsimultaneously,thentheprobabilityofexactlytwoheadsisa.3/8b.5/8c.7/8d.1/2e.Noneofthese22.LetEbethesetofallintegerswith1intheirunitplace.Theprobabilitythatanumberchosen{2,3,4….50}isanelementofEisa.2/13b.2/7c.4/49d.5/49e.Noneofthese23.Anumberxischosenatrandomfrom{1,2,….10}.Theprobabilitythatxsatisfiestheequation(x-3)(x-6)(x-10)=0isa.1/5b.2/5c.3/10d.7/10e.Noneofthese

Page 41: Maths book part 2 obtains Rs. 1100 after lending out Rs.x at 5% per annum for 2 years and obtains Rs. 1800 after lending out Rs.y at 10% per annum for 2 years. Find the value of x

41

24.Apersongetsasmanyrupeesasthenumberhegetswhenanunbiasedsix-faceddieisthrown.IftwosuchdicearethrowntheprobabilityofgettingRs.10isa.1/12b.2/11c.3/13d.7/12e.Noneofthese25.Abagcontains3redballs,4whiteballsand7blackballs.Theprobabilityofdrawingaredorablackballisa.5/14b.5/7c.5/49d.25/49e.Noneofthese26.Ifadieisthrown,thentheprobabilityofgettinganevennumberoranumbergreaterthan3,isa.2/3b.2/7c.2/5d.3/5e.Noneofthese27.Iftwodicearethrownsimultaneously,thentheprobabilityofhaving6onfirstdieandanynumberotherthan6onotherdieisa.25/36b.5/6c.5/36d.7/36e.Noneofthese28.Inasinglethrowoftwodice,findtheprobabilityofgettingatotalof3or5a.1/6b.2/5c.7/10d.3/10e.Noneofthese29.Aboxcontains36ticketsnumbered1to36,oneticketdrawnatrandom.Findtheprobabilitythatthenumberontheticketiseitherdivisibleby3orisaperfectsquarea.5/9b.1/9c.2/9d.4/9e.Noneofthese30.ThelettersB,G,I,N,Rarerearrangedtoformtheword‘BRING’.Finditsprobabilitya.7/120b.1/120c.11/120d.119/120e.Noneofthese31.Aboxofelectronicdiodecontains120standardand80sub-standardones.Twodiodesaretakenatrandom.Whatistheprobabilitythatoneisstandardandtheotherissub-standard?a.97/199b.99/199c.96/199d.95/199e.Noneofthese32.Aboxcontains20white,30black,40blueand30redballs.Computetheprobabilitythatoneoftheballsextractedatrandomfromtheboxturnsouttobewhite,blackorreda.2/3b.2/7c.2/5d.3/5e.Noneofthese33.Ifonerollsafair-sideddietwice,whatistheprobabilitythatthediewilllandonthesamenumberonboththeoccasions?a.1/6b.2/5c.7/10d.3/10e.Noneofthese34.Ifsevencoinsaretossed,whatistheprobabilityofobtainingatleast2heads?a.1/16b.9/16c.7/16d.15/16e.Noneofthese35.Abagcontains4red,5blueand6blackballs.Ifoneballisdrawnatrandom,findtheprobabilitythatitisnotablackballa.2/3b.2/7c.2/5d.3/5e.Noneofthese

Page 42: Maths book part 2 obtains Rs. 1100 after lending out Rs.x at 5% per annum for 2 years and obtains Rs. 1800 after lending out Rs.y at 10% per annum for 2 years. Find the value of x

42

36.Acoinistossedtwice.Findtheprobabilityofgettingnoheada.3/4b.1/4c.4/5d.1/5e.Noneofthese37.Threeunbiasedcoinsaretossed.Findtheprobabilityofgettingatleasttwoheadsa.2/5b.2/7c.1/2d.3/5e.Noneofthese38.Iftwodicearethrowntogether,whatistheprobabilityofthesumbeinglessthan6,onthesidefacingup?a.3/18b.5/18c.1/18d.3/9e.Noneofthese39.Fromapackof52cards,whatistheprobabilityofpickingaspadecardifonlyonecardisdrawn?a.3/4b.1/4c.4/5d.1/5e.Noneofthese40.Ifabagcontains5yellowand6redballsand2ballsaredrawnouttogether,whataretheoddsagainstallballsbeingred?a.7/11b.6/11c.8/11d.9/11e.Noneofthese41.Whatistheprobabilitythatarandomlychosen2digitnumberisdivisibleby5?a.3/4b.1/4c.4/5d.1/5e.Noneofthese42.Thereare16candidatesforthreeopeningsinacompany.Iffiveofthemarewomen,whatistheprobabilitythatexactlyonewomanwouldbechosenforthejob?a.54/221b.52/225c.55/221d.55/224e.Noneofthese43.Aschoolhas4labsand10classrooms.Ifastudentcangotoanyoftheserooms,whatistheprobabilitythatthestudentsenteredaparticularroom?a.3/14b.1/14c.4/15d.1/15e.Noneofthese44.IfthelettersofthewordEnglisharerearranged,whatistheprobabilitythatawordwillstartwithavowel?a.2/7b.3/7c.5/7d.1/7e.Noneofthese45.Reshmadecidedtogetmarriedandwantedtoinform3outofher5friendsfirst,beforeconveyingtoeveryoneelse.IfSushmaisoneamongher5friends,whatistheprobabilitythatSushmawillbeamongthefirst3tohearaboutReshma’swedding?a.3/8b.5/8c.1/8d.3/5e.Noneofthese46.Whenfourcoinsaretossedsimultaneously,whatistheprobabilityofgettingexactlytwoheads?a.3/8b.5/8c.1/8d.3/5e.Noneofthese47.Fromapackofcards,if4randomcardsaredrawn,whatistheprobabilitythateachcardisfromadifferentsuit?a.132/52C2b.13/52C1c.133/52C3d.134/52C4e.Noneofthese

Page 43: Maths book part 2 obtains Rs. 1100 after lending out Rs.x at 5% per annum for 2 years and obtains Rs. 1800 after lending out Rs.y at 10% per annum for 2 years. Find the value of x

43

48.Findtheprobabilityofgettinganumberlessthanfivewhenadieisrolled?a.2/3b.2/7c.2/5d.3/5e.Noneofthese49.Twodicearethrown,findtheprobabilityofgettinganeventotalonthem?a.2/3b.1/3c.1/2d.3/5e.Noneofthese50.Threedicearethrown,findtheprobabilityofgettingasumgreaterthan17ontheirfaces?a.1/216b.1/108c.1/54d.5/216e.Noneofthese51.Ifthreecoinsaretossed,findtheprobabilityoftheeventshowingexactlyoneTailonthem?a.3/8b.5/8c.1/8d.3/5e.Noneofthese52.Twocoinsaretossed.Findtheprobabilitythattwotailsresult,giventhatthereisatleastonetail?a.1/3b.2/3c.1/6d.1/2e.Noneofthese53.Onecardisdrawnatrandomfromapackof52cards.Whatistheprobabilitythatthecarddrawnisafacecard?a.3/13b.2/13c.1/13d.5/13e.Noneofthese54.FindtheprobabilitythatvowelselectedatrandomfromanEnglishbookisa‘0’a.3/4b.1/4c.4/5d.1/5e.Noneofthese55.Inasinglethrowwithtwodice,findtheprobabilityofthrowinga10?a.1/13b.2/13c.1/16d.1/12e.Noneofthese56.When4coinsaretossedsimultaneouslyfindtheprobabilityofgettingexactlyonetail?a.1/4b.5/8c.1/8d.3/4e.Noneofthese57.Twodicearethrownsimultaneously.Findtheprobabilitythatthesumofthenumbersisatmost4?a.1/3b.2/3c.1/6d.1/2e.Noneofthese58.Findtheprobabilitythatanon-leapyearshouldhave52Tuesdays?a.2/3b.2/7c.6/7d.3/5e.Noneofthese59.Asingleletterisselectedatrandomfromtheword‘CONICS’.Findtheprobabilitythatitisavowel?a.1/3b.2/3c.1/6d.1/2e.Noneofthese60.If3dicearerolled,findthetotalnumberofexhaustiveoutcomes?a.53b.73c.43d.63e.Noneofthese61.Iffourstudentsarechosenatrandom,findtheprobabilitythatnotwoofthemwereborn

Page 44: Maths book part 2 obtains Rs. 1100 after lending out Rs.x at 5% per annum for 2 years and obtains Rs. 1800 after lending out Rs.y at 10% per annum for 2 years. Find the value of x

44

onthesamedayoftheweek?a.124/343b.120/343c.125/343d.130/343e.Noneofthese62.Anintegerischosenfrom3to17whatistheprobabilitythatitisprime?a.3/4b.1/4c.2/5d.1/5e.Noneofthese63.Acardisdrawnatrandomfromadeckofcards.Findtheprobabilityofgettingthe3ofdiamond?a.1/13b.2/13c.1/26d.1/52e.Noneofthese64.Findtheprobabilityofgettinganoddnumberinthrowofanunbiaseddie?a.1/3b.2/3c.1/6d.1/2e.Noneofthese65.Whatistheprobability,thataleapyearselectedatrandomwillcontain53Sundays?a.5/7b.1/7c.3/7d.2/7e.Noneofthese66.Findtheprobabilityofgettinga‘king’ora‘Queen’inasingledrawfromawellshuffledpackofplayingcards?a.1/13b.2/13c.1/16d.1/12e.NoneoftheseDirections(67-69):Onecardisdrawnfromapackof52cards,eachcardbeingequallylikelytobedrawn67.Findtheprobabilitythatthecarddrawnisreda.1/3b.2/3c.1/6d.1/2e.Noneofthese68.Findtheprobabilitythatthecarddrawnisakinga.1/13b.2/13c.1/16d.1/12e.Noneofthese69.Findtheprobabilitythatthecarddrawnisredandakinga.1/26b.2/13c.1/16d.1/12e.Noneofthese

Answers1.A 2.D 3.C 4.D 5.B 6.A 7.C 8.B 9.A 10.B11.B 12.E 13.D 14.A 15.D 16.B 17.A 18.D 19.B 20.C21.A 22.C 23.C 24.A 25.B 26.A 27.C 28.A 29.D 30.B31.C 32.A 33.A 34.D 35.D 36.B 37.C 38.B 39.B 40.C41.D 42.D 43.B 44.A 45.D 46.A 47.D 48.A 49.C 50.A51.A 52.A 53.A 54.D 55.D 56.A 57.C 58.C 59.A 60.D61.B 62.C 63.D 64.D 65.D 66.B 67.D 68.A 69.A

Page 45: Maths book part 2 obtains Rs. 1100 after lending out Rs.x at 5% per annum for 2 years and obtains Rs. 1800 after lending out Rs.y at 10% per annum for 2 years. Find the value of x

45

DatainterpretationGraphsWhiletablesexpressactualnumbers,graphsareadiagrammaticrepresentationofdata.Theybringouttherelationshipbetweendatamoreclearlythannumbersinatable.Forexample,apie-chartcanbringoutclearlythepercentagethatAnilAmbaniownsofRelianceCommunicationsLtdandthefactthatheisthelargestshareholder,whileatablewouldrequireyoutoactuallycalculatethepercentageofeachshareholder'sownershiptofindoutthelargestshareholder.Graphsarefarbettertounderstandchangesinvariables-whetheraparticularvaluehasrisenorfallenoverthepastfewyearsandhenceanalyzethetrends.PieChartsTheyderivetheirnamefromitsshape,likethatofapiedividedintovariousportions.Theyalwaysrepresentdataintheformofapercentageofthetotal,withthetotalpercentagebeing100.Insuchachart,thelengthofthearc(andthereforetheangleeachsectorsubtendsatthecentre)isproportionaltothequantityitrepresents.Suchchartsareoftenusedinthecorporateworldandinnewspapers.Sinceacirclecomprises360degrees,eachpercentofapie-chartisequalto360dividedby100,or3.6degrees.Thisfactwillbeimportantforthecalculationsyouareexpectedtoperform.BarGraphsBargraphsrepresentdataintheformofcolumnsorbars.Bargraphscanbehorizontalorvertical.Thelengthofthebarisproportionaltothedatavaluerepresentedbyit.LineGraphsLinegraphrepresentsdataintheformofstraightlinesthatconnectvariousdatavalues.Bothlinegraphsandbargraphsareusedtoconveysamethingsandhencecanbeusedinter-changeably.Forexample,alinegraphcanbegeneratedbyjoiningthetipofthebargraph.

Page 46: Maths book part 2 obtains Rs. 1100 after lending out Rs.x at 5% per annum for 2 years and obtains Rs. 1800 after lending out Rs.y at 10% per annum for 2 years. Find the value of x

46

DirectionforQ1-Q5:Followingtableshowsthescoresofstudentsin8BANKPO(sectionwise).AssumeallBANKPOEXAMcontain3sections,eachsectionshave50questions.Thetotalofthescoresin3sectionsiscomparedwith“cutoff”andifhistotalscoreishigherthanthe“cutoff”,itissaidthathehasclearedthe“cutoff”. English Quant Rasoning Cut-off TotalscoreEXAM1 23 12 19 56 54EXAM2 24 17 18 62 59EXAM3 29 11 23 60 63EXAM4 21 14 21 57 56EXAM5 19 13 25 59 57EXAM6 26 17 21 60 64EXAM7 25 19 23 58 67EXAM8 27 20 21 65 68Q1.InhowmanyBANKPOEXAMSdidheclearthecutoff?Solution:asclearfromthetable,thereare4BANKEXAMSinwhichtotalmarksaregreaterthancutoffsoin4heclearthecutoff.Soansweris4Q2.InwhichBANKEXAMSdidhescorethemaximumtotalmarks?Solution:asclearfromthetable,inBANKEXAMS,hegotmaximumscorei.e.68soanswerisBANKEXAM8Q3.Whatishisimproving,ifimprovementisdefinedas=(maximumscore–minimumscore)?Solution:Hismaximumscoreis68andminimumscoreis54sotheimprovementis(68-54)=14soansweris14Q4.Inwhichsectionhasheshownthemaximumimprovementamongthese8BANKEXAMS?Solution:ImprovementinEnglish=29-19=10ImprovementinQuant=20-11=9Improvementinreasoning=25-18=7 HencemaximumimprovementisinEnglishQ5.Ifthesection-wisecut-offinallBANKEXAMSis15forQuantand20eachforEnglishandReasoning,theninhowmanyBANKEXAMSdidheclearallthecut-offs?Solution:Thereareonly3BANKEXAMS,inwhichsectionwisecutoffhecleartheseareBANKEXAMS6,7,8DirectionsforQ6-Q8:Studythegraphcarefullyandanswerthedatainterpretationsquestionsthatfollow.Thegraphshowstheimportsandexportsofcottoninrupeescroresfrom1990-91to1994-94.

Page 47: Maths book part 2 obtains Rs. 1100 after lending out Rs.x at 5% per annum for 2 years and obtains Rs. 1800 after lending out Rs.y at 10% per annum for 2 years. Find the value of x

47

Q6.Whatistheratioofthenumberofyearshavingaboveaverageexportstothosehavingbelowaverageexportsinthegivenperiod?Solution:Averageexports=638+1226+1661+1538+1305/5=1273.6Years1990-91&1991-92haveexportslessthanaverageexportwhileothershavemorethanthat.Sorequiredratiois3:2.Q7.Inwhichyearwasthegapinimportsandexportstheleast?Solution:Thegapinimportsandexportsfortheyear1990-91is186,1991-92is212,1992-93is524,1993-94is25and1994-95is353.Soyear1993-94hastheminimumgap.Alsoit’scanbedeterminedbylookingatthegraph.Q8.Theimportsin1994-95wereapproximatelyhowmuchpercentmorethantheimportsof1990-91?Solution:Importin1994-95is1,658crore.Importin1990-91is824croreExcess=1658=1658–824=834croretherefore,Excesspercentage=834/824X100=101.2%.DirectionsforQ9toQ13.Studythefollowinglinegraphwhichgivesthenumberofstudentswhojoinedandlefttheschoolinthebeginningofyearforsixyears,from1996to2001.InitialStrengthofschoolin1995=3000.

Page 48: Maths book part 2 obtains Rs. 1100 after lending out Rs.x at 5% per annum for 2 years and obtains Rs. 1800 after lending out Rs.y at 10% per annum for 2 years. Find the value of x

48

Q9.Thestrengthofschoolincreased/decreasedfrom1997to1998byapproximatelywhatpercent?Solution:Importantdatanotedfromthegivengraph:In1996:Numberofstudentsleft=250andnumberofstudentsjoined=350.In1997:Numberofstudentsleft=450andnumberofstudentsjoined=300.In1998:Numberofstudentsleft=400andnumberofstudentsjoined=450.In1999:Numberofstudentsleft=350andnumberofstudentsjoined=500.In2000:Numberofstudentsleft=450andnumberofstudentsjoined=400.In2001:Numberofstudentsleft=450andnumberofstudentsjoined=550Therefore,thenumbersofstudentsstudyingintheschool(i.e.,strengthoftheschool)invariousyears:In1995=3000(given).In1996=3000-250+350=3100.In1997=3100-450+300=2950.In1998=2950-400+450=3000.In1999=3000-350+500=3150. In2000=3150-450+400=3100.In2001=3100-450+550=3200.Percentageincreaseinthestrengthoftheschoolfrom1997to1998= C%%%W<_?%

C<%%×100 %=1.7%

Q10.Thenumberofstudentsstudyingintheschoolduring1999was?Solution:Ascalculatedabove,thenumberofstudentsstudyingintheschoolduring1999=3150.Q11.Duringwhichofthefollowingpairsofyears,thestrengthoftheschoolwassame?Solution:Ascalculatedabove,intheyears1996and2000thestrengthoftheschoolwassamei.e.,3100.

Page 49: Maths book part 2 obtains Rs. 1100 after lending out Rs.x at 5% per annum for 2 years and obtains Rs. 1800 after lending out Rs.y at 10% per annum for 2 years. Find the value of x

49

Q12.Thenumberofstudentsstudyingintheschoolin1998waswhatpercentofthenumberofstudentsstudyingintheschoolin2001?Solution:Requiredpercentage= C%%%

C<%%×100 %=93.75%

Q13.Amongthegivenyears,thelargestnumberofstudentsjoinedtheschoolintheyear?Solution:Ascalculatedabove,thelargestnumberofstudents(i.e.,550)joinedtheschoolintheyear2001.DirectionforQ14-Q18:Thecircle-graphgivenhereshowsthespendingsofacountryonvarioussportsduringaparticularyear.Studythegraphcarefullyandanswerthequestionsgivenbelowit.

Q14.WhatpercentoftotalspendingisspentonTennis?Solution:PercentageofmoneyspendonTennis=45/360*100=12.5%Q15.HowmuchpercentmoreisspentonHockeythanthatonGolf?Solution:LetthetotalspendingsonsportsbeRs.x.Then,AmountspentonGolf= C`

C`%×𝑥%=x/10%

Amountspentonhockey= `CC`%

×𝑥%=7x/40%Difference=Rs.XD

>%− D

$%=Rs.CD

>%

Requiredpercentage=Rs.nAFon=F×100=75%

Q16.HowmuchpercentlessisspentonFootballthanthatonCricket?Solution:LetthetotalspendingsonsportsbeRs.x.Then,AmountspentonCricket=Rs. [$

C`%×𝑥%=_D

>%

AmountspentonFootball=Rs. ?>C`%

×𝑥%=CD<%

Difference=Rs._D>%−CD

<%=CD

>%

Requiredpercentage=Rs.Hn=Fpn=F×100=33.33%

Page 50: Maths book part 2 obtains Rs. 1100 after lending out Rs.x at 5% per annum for 2 years and obtains Rs. 1800 after lending out Rs.y at 10% per annum for 2 years. Find the value of x

50

Q17.IfthetotalamountspentonsportsduringtheyearbeRs.1,80,00,000,theamountspentonBasketballexceedsonTennisby:Solution:AmountspentonBasketballexceedsthatonTennisby=?%W>?

C`%×1800000

=Rs.250000Q18.IfthetotalamountspentonsportsduringtheyearwasRs.2crores,theamountspentonCricketandHockeytogetherwas:Solution:AmountspentonCricketandHockeytogether=[$E`C

C`%×2=Rs.8000000

Q19.Ifthereareatotalof200items,inastationaryshop,findthedifferenceinthenumberofpencilstopens

Solution:Totalno.ofitems=200,No.ofpens= $`

$%%×200=32

No.ofpencils= C[$%%

×200=76So,Difference=76-32=4420.Abagcontainsballsof4colorsasshowninthefigure,ifaballispickedatrandomwhichcoloredballhasthemaximumprobabilitytobepicked.

Solution:As,clearlyshowninfigure,blackareaismaximumsoprobabilitytobepickedanyballwillbemaximumforblack21.Thegivenpie-chart,showsthepercentageofchildreninterestedineachfield,findthenumberofstudentswhoareinterestedinmusicfromagroupof120students.

Solution:%ofstudentsinterestedinmusic=30%Totalnumberofstudents=120Numberofstudentsinterestedinmusic=30%of120=36

Page 51: Maths book part 2 obtains Rs. 1100 after lending out Rs.x at 5% per annum for 2 years and obtains Rs. 1800 after lending out Rs.y at 10% per annum for 2 years. Find the value of x

51

Directions(Q22-):Studythefollowingtablecarefullyandanswerthequestionthatfollows:PRODUCTIONOFMACHINES(inunits)

Q22.Theproductionofmachinesin1985isapproximatelyhowmanytimesthatoftheproductionin1983?Solution:Youshould,firstofall,writedownthetotalproductionofmachinesforeachofthegivenyears.430/390=1.1.Q23.InwhichofthefollowingyearsdidthetypeVmachinesregisterthehighestproductionascomparedtothetotalproductionforthatyear?Solution:In1980,theratioworksoutto60/217=0.27.Q24.InwhichofthefollowingyearswasthereamaximumincreaseintheproductionoftypeIIImachinesoverthepreviousyear?Solution:In1983,Increase=70–53=17.Q25.Whatwastheapproximate%increaseintheproductionofalltypesofmachinestogether,fromtheyear1981totheyear1984?Solution:Therequired%growthrate=(420–280)/280=50%

TYPE 1980 1981 1982 1983 1984 1985I 60 80 75 85 90 94II 28 40 40 60 65 71III 39 45 53 70 75 78IV 30 60 120 90 100 92V 60 55 52 85 90 95

Page 52: Maths book part 2 obtains Rs. 1100 after lending out Rs.x at 5% per annum for 2 years and obtains Rs. 1800 after lending out Rs.y at 10% per annum for 2 years. Find the value of x

52

Exercise

DirectionforQ1-Q3

1.Howmanyboysattendedthe1995convention?a.358 b.390 c.407 d.540 e.7162.Whichyeardidthesamenumberofboysandgirlsattendtheconference?a.1995 b.1996 c.1997 d.1998 e.None3.Whichtwoyearsdidtheleastnumberofboysattendtheconvention?a.1995and1996 b.1995and1998 c.1996and1997d.1997and1994 e.1997and1998DirectionforQ4-Q5

Page 53: Maths book part 2 obtains Rs. 1100 after lending out Rs.x at 5% per annum for 2 years and obtains Rs. 1800 after lending out Rs.y at 10% per annum for 2 years. Find the value of x

53

4.Whichvacationdestinationismostcommonforthestudents?a.Beach b.HistoricalSites c.Cruises d.Mountains e.Other5.If500studentsattendWashingtonMiddleSchool,howmanyaregoingtothemountainsforvacation?a.25 b.60 c.75 d.100 e.125DirectionforQ6-Q8

6.Whichtwoyearsweretheleastnumberoftiressold?a.1998and1999 b.1998and2000 c.1998and2001d.1999and2000 e.2000and20017.Whichyeardidthestoresell1/3moretiresthantheyearbefore?a.1998 b.1999 c.2000d.2001 e.Thisdidnotoccurduringthe4yearspan.8.Whatwastheaveragenumberoftiressoldbythestorefrom1998to2001?a.9,000 b.9,375 c.9,545 d.9,770 e.9,995

Page 54: Maths book part 2 obtains Rs. 1100 after lending out Rs.x at 5% per annum for 2 years and obtains Rs. 1800 after lending out Rs.y at 10% per annum for 2 years. Find the value of x

54

DirectionforQ9-Q11

9.WhatwastheaveragenumberofbabiesthatDr.Jonesdeliveredeachyearfrom1995to1998?a.35 b.40 c.45 d.50 e.5510.HowmanybabiesdidDr.Jonesdeliverin1998?a.25 b.35 c.45 d.55 e.6511.IfDr.Jonesdelivered85babiesin1999,howmanyrattleswouldrepresentthisnumber?a.6½ b.7 c.7½D.8 e.8½DirectionforQ12-14

Page 55: Maths book part 2 obtains Rs. 1100 after lending out Rs.x at 5% per annum for 2 years and obtains Rs. 1800 after lending out Rs.y at 10% per annum for 2 years. Find the value of x

55

12.IfXYZAutoCompanysold23,000vehiclesin1999,howmanywereSUV's?a.2,990 b.3,030 c.3,450 d.4,760 e.4,77513.If7,650trucksweresoldin1999,howmanytotalvehiclesweresoldin1999byXYZAutoCompany?a.35,000 b.40,000 c.45,000 d.50,000 e.55,00014.If3,7502-doorsedansweresoldin1999,thenhowmany4-doorsedansweresoldin1999byXYZAutoCompany?a.3,578 b.4,950 c.5,120 d.5,670 e.5,000DirectionforQ15-Q17.

15.Howmuchdidtheinfantgaininthefirstmonthoflife?a.0.5pounds b.1.2pounds c.1.5pounds d.0.25poundse.0.75pounds16.WhatwastheaverageweightoftheinfantfromApriltoOctober,roundedtothenearestounce?a.10pounds b.10.5pounds c.10.9poundsd.11.5pounds e.11.9pounds17.Betweenwhichtwomonthsdidtheinfantgainthemostweight?a.AprilandMay b.JuneandJuly c.JulyandAugustd.AugustandSeptember e.SeptemberandOctoberDirections(Q18-Q22):Studythefollowinginformationandanswerthefollowingquestion:Mr.Mohanhastodecidewhetherornottotestabatchof1000crank–shaftsbeforesendingthemtothebuyeroverseas.Incasehedecidedtotest,hehastwooptions:(1)Usetest1(2)Usetest2.Test1costsRs.2percrankshaft.Howeverthistestisnotperfect.Itsends20%ofthebadonestothebuyerasgood.Test2willcostRs.3percrank–shaft.Itindeedbringsout

Page 56: Maths book part 2 obtains Rs. 1100 after lending out Rs.x at 5% per annum for 2 years and obtains Rs. 1800 after lending out Rs.y at 10% per annum for 2 years. Find the value of x

56

allthebadones.AdefectivecrankshaftidentifiedbeforesendingcanbecorrectedatacostofRs.25percrank–shift.Alldefectivecrank–shaftsareidentifiedandthebuyer'sendandapenaltyofRs.50perdefectivecrankshafthastobepaidbyMr.Mohanasperthecontractsigned.Note:OnceadefectivepieceisidentifiedbyMr.Mohan,thenecessarilysendsitforcorrection.Mr.Mohanhasappointedyou,abuddingmanagementconsultant,toadvisehim.Youhavetoadvisehimpurelyonacommercialbasis18.Mr.Mohanshouldnottestiftheno.ofdefectivecrank–shaftsinthelotis:a.lessthan100b.morethan200c.between120&190d.indeterminatee.Noneofthese19.Iftheno.ofdefectivecrank–shaftsinthelotisbetween200and400,Mohan:a.shoulduseeithertest1ortest2b.shouldusetest1onlyc.shouldusetest2onlyd.cannotdecidee.Noneofthese20.Ifthereare200defectivecrank–shaftsinthelot,Mohan:a.shoulduseeithertest1ortest2b.shouldusetest1ornotuseanytestc.shouldusetest2ornotuseanytestd.cannotdecidee.Noneofthese21.IfMohanistoldthatthelothas160defectivecrank–shafts,heshould:a.usetest1onlyb.usetest2onlyc.donotestingd.eitherusetest1ordonoteste.Noneofthese22.Thetotalcostoftest1whenthenumberofdefectivepiecesis600is:a.Rs.16680b.Rs.20000c.Rs.30000d.Rs.22600e.NoneoftheseDirections(Q.23–27):Studythefollowinggraphandanswerthequestionsthatfollow:No.ofstudents(inthousands)whooptedforthreedifferentspecializationsduringthegivenfiveyearsinauniversity

Page 57: Maths book part 2 obtains Rs. 1100 after lending out Rs.x at 5% per annum for 2 years and obtains Rs. 1800 after lending out Rs.y at 10% per annum for 2 years. Find the value of x

57

23.Outofthetotalnumberofstudentswhooptedforthegiventhreesubjects,intheyear2009,38%weregirls.HowmanyboysoptedforMathematicsinthesameyear?a.3100b.3200c.3000d.Cannotbedeterminede.Noneofthese24.Ifthetotalnumberofstudentsintheuniversityintheyear2007was455030,thetotalnumberofstudentswhooptedforthegiventhreesubjectswasapproximatelywhatpercentofthestudents?a.19b.9c.12d.5e.2325.WhatisthetotalnumberofstudentswhooptedforHindiandMathematicsintheyears2006,2007and2009together?a.97000b.93000c.85000d.96000e.Noneofthese26.ThetotalnumberofstudentswhooptedforMathematicsintheyears2005and2008togetherisapproximatelywhatpercentofthetotalnumberofstudentswhooptedforallthreesubjectsinthesameyear?a.38b.28c.42d.32e.4827.WhatistheratioofthenumberofstudentswhooptedforEnglishintheyear2006and2008togethertothenumberofstudentswhooptedforHindiintheyear2005and2009together?a.11:5b.12:7c.11:7d.12:5e.Noneofthese

Page 58: Maths book part 2 obtains Rs. 1100 after lending out Rs.x at 5% per annum for 2 years and obtains Rs. 1800 after lending out Rs.y at 10% per annum for 2 years. Find the value of x

58

DirectionsforQ28–32:Studythefollowinglinegraphwhichgivesthenumberofstudentswhojoinedandlefttheschoolinthebeginningofyearforsixyears,from1996to2001.InitialStrengthofschoolin1995=3000

28.Thestrengthofschoolincreased/decreasedfrom1997to1998byapproximatelywhatpercent?a.10%b.1.7%c.3%d.5%e.Noneofthese29.Thenumberofstudentsstudyingintheschoolduring1999was?a.3150b.3000c.3100d.3050e.Noneofthese30.Duringwhichofthefollowingpairsofyears,thestrengthoftheschoolwassame?a.1999&2000b.2000&2001c.1996&2000d.1998&2000e.Noneofthese31.Thenumberofstudentsstudyingintheschoolin1998waswhatpercentofthenumberofstudentsstudyingintheschoolin2001?a.93%b.93.75%c.94%d.92%e.Noneofthese32.Amongthegivenyears,thelargestnumberofstudentsjoinedtheschoolintheyear?a.1997b.1998c.1999d.2001e.2000Directions(33to35):-Studythebarchartandanswerthequestionbasedonit.ProductionofFertilizersbyaCompany(in1000tonnes)OvertheYears

Page 59: Maths book part 2 obtains Rs. 1100 after lending out Rs.x at 5% per annum for 2 years and obtains Rs. 1800 after lending out Rs.y at 10% per annum for 2 years. Find the value of x

59

33.Whatwasthepercentagedeclineintheproductionoffertilizersfrom1997to1998?a.33.33%b.25%c.20%d.50%e.Noneofthese34.Inhowmanyofthegivenyearswastheproductionoffertilizersmorethantheaverageproductionofthegivenyears?a.1b.2c.3d.4e.Noneofthese35.Theaverageproductionof1996and1997wasexactlyequaltotheaverageproductionofwhichofthefollowingpairsofyears?a.2000and2001b.1999and2000c.1998and2000d.1995and2001e.NoneoftheseDirections(36to38):-Thefollowingpie-chartshowsthepercentagedistributionoftheexpenditureincurredinpublishingabook.Studythepie-chartandtheanswerthequestionsbasedonit.VariousExpenditures(inpercentage)IncurredinPublishingaBook

36.WhatisthecentralangleofthesectorcorrespondingtotheexpenditureincurredonRoyalty?a.15°b.24°c.54°d.48°e.Noneofthese37.Royaltyonthebookislessthantheprintingcostby:a.25%b.20%c.30%d.15%e.Noneofthese

Page 60: Maths book part 2 obtains Rs. 1100 after lending out Rs.x at 5% per annum for 2 years and obtains Rs. 1800 after lending out Rs.y at 10% per annum for 2 years. Find the value of x

60

38.Ifforacertainquantityofbooks,thepublisherhastopayRs.30,600asprintingcost,thenwhatwillbeamountofroyaltytobepaidforthesebooks?a.Rs.19,450 b.Rs.21,200c.Rs.22,950d.Rs.26,150e.NoneoftheseDirectionsfor(Q39to41):RefertothefollowingBar-chartandanswerthequestionsthatfollow:

39.Whatistheaveragevalueofthecontractsecuredduringtheyearsshowninthediagram?a.Rs.103.48croreb.Rs.105crorec.Rs.100crored.Rs.125.2croree.Noneofthese40.Comparedtotheperformancein1985(i.e.takingitasthebase),whatcanyousayabouttheperformancesintheyears’84,’85,’86,’87,’88respectively,inpercentageterms?a.150,100,211,216,97b.100,67,141,144,65c.150,100,200,215,100d.120,100,220,230,68e.Noneofthese41.Whichistheyearinwhichthehighestpercentagedeclineisseeninthevalueofcontractsecuredcomparedtotheprecedingyear?a.1985b.1988c.1984d.1986e.Noneofthese

Page 61: Maths book part 2 obtains Rs. 1100 after lending out Rs.x at 5% per annum for 2 years and obtains Rs. 1800 after lending out Rs.y at 10% per annum for 2 years. Find the value of x

61

Directionsforthequestionsfrom42to46:Thefollowingtablegivesthenationalincomeandthepopulationofacountryfortheyears1984–85to1989–90.Foreachothefollowingquestionschoosethebestalternative:

42.Theincreaseinthepercapitaincomecomparedtothepreviousyearislowestfortheyear:a.1985-86b.1986-87c.1987-88d.1989-90e.Noneofthese43.Thepercapitaincomeishighestfortheyear:a.1984-85b.1985-86c.1987-88d.1989-90e.Noneofthese44.Thedifferencebetweenthepercentageincreaseinpercapitaincomeandthepercentageincreaseinthepopulationcomparedtothepreviousyearishighestfortheyear:a.1985-86b.1986-87c.1987-88d.1988-89e.Noneofthese45.Therateofincreaseinpopulationwaslowestintheyear:a.1985-86b.1987-88c.1989-90d.datainadequatee.Noneofthese46.Increaseinthepercapitaincomecomparedtothepreviousyearamongtheyearsgivenbelowwashighestfortheyear:a.1985-86b.1986-87

Page 62: Maths book part 2 obtains Rs. 1100 after lending out Rs.x at 5% per annum for 2 years and obtains Rs. 1800 after lending out Rs.y at 10% per annum for 2 years. Find the value of x

62

c.1987-88d.1989-90e.NoneoftheseDirection(Q47–51):Thegraphbelowshowstheendofthemonthmarketvaluesof4sharesfortheperiodfromJanuarytoJune.Answerthefollowingquestionsbasedonthisgraph.

47.Whichshareshowedthegreatestpercentageincreaseinmarketvalueinanymonthduringtheentireperiod?a.Ab.Bc.Cd.De.Noneofthese48.Inwhichmonthwasthegreatestabsolutechangeinmarketvalueforanysharerecorded?a.Marchb.Aprilc.Mayd.Junee.Noneofthese49.Inwhichmonthwasthegreatestpercentageincreaseinmarketvalueforanysharerecorded?a.Februaryb.Marchc.Aprild.Maye.Noneofthese50.Anindividualwishestosell1shareofCand1shareofDtobuy1shareofAattheendofa

Page 63: Maths book part 2 obtains Rs. 1100 after lending out Rs.x at 5% per annum for 2 years and obtains Rs. 1800 after lending out Rs.y at 10% per annum for 2 years. Find the value of x

63

month.Atwhichmonth-endwouldtheindividual’slossfromthisdecision,duetosharevaluechanges,bethemost?a.Februaryb.Marchc.Aprild.Junee.Noneofthese51.Anindividualdecidestosell1shareofCand1shareofDtobuy1shareofAattheendofthemonth.Whatcanbetheindividual’sgreatestgainfromthisdecision,duetosharevaluechanges?a.5b.10c.15d.datainadequatee.NoneoftheseDirection(Q52–56):Answerthequestionsbasedonthefollowingtable.

52.Themaximumpercentagedecreaseinmarketshareisa.60%b.50%c.33.3%d.20%e.Noneofthese53.Thecityinwhichminimumnumberofproductsincreasedtheirmarketsharesin1993-94isa.Mumbaib.Delhic.Kolkatad.Chennai

Page 64: Maths book part 2 obtains Rs. 1100 after lending out Rs.x at 5% per annum for 2 years and obtains Rs. 1800 after lending out Rs.y at 10% per annum for 2 years. Find the value of x

64

e.Noneofthese54.Themarketsharesofwhichproductsdidnotdecreasedbetween1993-94inanycity?a.HDb.COc.BNd.datainadequatee.Noneofthese55.Thenumberofproductswhichhad100%marketshareinfourmetropolitancitiesisa.0b.1c.2d.3e.Noneofthese56.Thenumberofproductswhichdoubledtheirmarketsharesinoneormorecitiesisa.0b.1c.2d.3e.NoneoftheseDirectionfor(Q57to60):Analyzethetableandanswerthequestionscarefully

Page 65: Maths book part 2 obtains Rs. 1100 after lending out Rs.x at 5% per annum for 2 years and obtains Rs. 1800 after lending out Rs.y at 10% per annum for 2 years. Find the value of x

65

57.Whatwasthetotalnumberofengineeringstudentsin1989–90?a.28500b.4400c.4200d.42000e.Noneofthese58.ThegrowthrateinstudentsofGovt.Engg.CollegescomparedtothatofPrivateEngg.Collegesbetween1988–89and1989–90isa.moreb.lessc.equald.3/2e.Cannotbedetermined59.ThetotalnumberofEngg.Studentsin1991–92,assuminga10%reductioninthenumberoverthepreviousyear,isa.5700b.57000c.44800d.noneofthesee.Cannotbedetermined60.In1990–91,whatpercentofEngg.StudentswerestudyingatIIT’s?a.16b.15c.14d.12e.NoneoftheseDirectionfor61to64:Refertothepie-chartgivenbelow:

Page 66: Maths book part 2 obtains Rs. 1100 after lending out Rs.x at 5% per annum for 2 years and obtains Rs. 1800 after lending out Rs.y at 10% per annum for 2 years. Find the value of x

66

61.WhatfractionofGhoshbabu’sweightconsistsofmuscularandskinprotein?a.1/13b.1/30c.1/20d.Cannotbedeterminede.Noneofthese62.Ratioofdistributionofproteininmuscletothedistributionofproteininskinisa.3:1b.3:10c.1:3d.31/2:1e.Noneofthese63.WhatpercentofGhoshBabu’sbodyweightismadeupofskin?a.0.15b.10c.1.2d.Cannotbedeterminede.Noneofthese64.Intermsoftotalbodyweight,theportionofmaterialotherthanwaterandproteinisclosesttoa.3/20b.1/15c.85/100d.1/20e.Noneofthese

Answers

1.A 2.A 3.A 4.A 5.B 6.B 7.B 8.B 9.C 10.D11.E 12.A 13.C 14.E 15.A 16.A 17.D 18.A 19.C 20.A21.A 22.B 23.D 24.B 25.E 26.D 27.A 28.B 29.A 30.C31.B 32.D 33.B 34.D 35.D 36.C 37.A 38.C 39.A 40.A41.B 42.B 43.D 44.B 45.C 46.E 47.D 48.A 49.A 50.A51.B 52.C 53.B 54.C 55.A 56.B 57.E 58.B 59.D 60.C61.C 62.E 63.D 64.A

Page 67: Maths book part 2 obtains Rs. 1100 after lending out Rs.x at 5% per annum for 2 years and obtains Rs. 1800 after lending out Rs.y at 10% per annum for 2 years. Find the value of x

67

Mixture&AlligationMixture:Mixingoftwoormorethantwotypeofquantitiesgivesusamixure.Quantitiesoftheseelementscanbeexpressedaspercentageorratio.i.e.Percentage(20%ofsugarinwater)Fraction(Asolutionofsugarandwatersuchthatsugar:water=1:4)Alligation:Alligationisarulewhichisusedtosolvetheproblemsrelatedtomixtureanditsingredient.Itistherulethatenablesustofindtheratioinwhichtwoormoreingredientsatthegivenpricemustbemixedtoproduceamixtureofdesiredprice.AlligationRuleWhentwoelementsaremixedtomakeamixtureandoneoftheelementsischeaperandotheroneiscostlierthen,

𝑄𝑢𝑎𝑛𝑡𝑖𝑡𝑦𝑜𝑓𝑐ℎ𝑒𝑎𝑝𝑒𝑟𝑄𝑢𝑎𝑛𝑡𝑖𝑡𝑦𝑜𝑓𝑐𝑜𝑠𝑡𝑙𝑖𝑒𝑟 =

𝐶. 𝑃𝑜𝑓𝑐𝑜𝑠𝑡𝑙𝑖𝑒𝑟 − 𝑚𝑒𝑎𝑛𝑜𝑓𝑝𝑟𝑖𝑐𝑒𝑀𝑒𝑎𝑛𝑝𝑟𝑖𝑐𝑒 − 𝐶𝑃𝑜𝑓𝑐ℎ𝑒𝑎𝑝𝑒𝑟

HereMeanPriceisCPofmixtureperunitquantity.Aboverulecanbewrittenas,

Then,CheaperQuantity:CostlierQuantity=(D–M):(M–C)ExampleInwhatproportionmustsugaratRs40perkgbemixedwithsugaratRs60perkgsothatthemixturebeRs55perkg?Sol:Here,CPofCheaper(C)=40,

Page 68: Maths book part 2 obtains Rs. 1100 after lending out Rs.x at 5% per annum for 2 years and obtains Rs. 1800 after lending out Rs.y at 10% per annum for 2 years. Find the value of x

68

CPofCostlier(D)=60andMeanPrice(M)=55Sofromtheruleofalligationwecansaythat

Cheapersugar/costliersugar=5/15ProportionofCheaperSugarandCostlierSugaris1:3Mixtureofmorethantwoelements.Thismethodisabittrickyinitiallybutifyoupracticeitthenitbecomesquiteeasy.Ifthemixtureisofmorethantwoingredients,thenwritethepricesofeachingredientbelowoneanotherinascendingorder.Writethemeanpricetotheleftofthelist.Nowmakecouplesofpricesinsuchawaythatonepriceofthecoupleisbelowmeanpriceandanotherpriceofthecoupleisabovemeanprice.Nowfindthedifferencebetweeneachpriceandmeanpriceandwriteitoppositetothepricelinkedtoit.Thisdifferenceisrequiredanswer.Don’tworryifyoudon’tunderstandaboveparagraph.Trytounderstandaboveparagraphwiththeexamplesgivenbelow.ExampleHowmustashopownermix4typesofriceworthRs95,Rs60,Rs90andRs50perkgsothathecanmakethemixtureofthesesugarsworthRs80perkg?Sol:Herethepricesofsugarsare95,60,90and50.Andthemeanpriceis80.Nowreadtheaboveparagraphandtheimagegivenbelowtounderstandthismethod.

Sotheproportionofsugaris

Page 69: Maths book part 2 obtains Rs. 1100 after lending out Rs.x at 5% per annum for 2 years and obtains Rs. 1800 after lending out Rs.y at 10% per annum for 2 years. Find the value of x

69

50:60:90:95=15:10:20:30or50:60:90:95=3:2:4:6ExampleInwhatratiomustapersonmixthreekindofteaeachofwhichhasapriceof70,80and120rupeesperkg,insuchawaythatthemixturecostshim100rupeesperkg?Sol:Herethepricesofteaare70,80and120Andmeanpriceis100,so

Sotheproportionofteais70:80:120=20:20:50or70:80:120=2:2:5SomeShortcutFormulasRule1IfndifferentvesselsofequalsizearefilledwiththemixtureofPandQintheratiop1:q1,p2:q2,……,pn:qnandcontentofallthesevesselsaremixedinonelargevessel,then

wxy9zb{|}!wxy9bzb{|}w

= ~A

~A��AE ~;~;��;

E...…….. ~G~G��G

�A~A��A

E �;~;��;

E...…….. �G~G��G

ExampleThreeequalbucketscontainingthemixtureofmilkandwateraremixedintoabiggerbucket.Iftheproportionofmilkandwaterintheglassesare3:1,2:3and4:2thenfindtheproportionofmilkandwaterinthebiggerbucket.Sol:Let’ssayPstandsformilkandQstandsforwater,So,p1:q1=3:1p2:q2=2:3p3:q3=4:2

𝑄𝑢𝑎𝑛𝑖𝑡𝑦𝑜𝑓𝑃𝑄𝑢𝑎𝑛𝑡𝑖𝑡𝑦𝑜𝑓𝑄 =

𝑃$𝑃$ + 𝑄$

+ 𝑃<𝑃< + 𝑄<

+. . . …… . . 𝑃9𝑃9 + 𝑄9

𝑄$𝑃$ + 𝑄$

+ 𝑄<𝑃< + 𝑄<

+. . . …… . . 𝑄9𝑃9 + 𝑄9

Page 70: Maths book part 2 obtains Rs. 1100 after lending out Rs.x at 5% per annum for 2 years and obtains Rs. 1800 after lending out Rs.y at 10% per annum for 2 years. Find the value of x

70

wxy9zb{|}!wxy9bzb{|}w

= H

H�AE;

;�HE=

=�;A

H�AEH

;�HE;

=�;=109/71

Soinbiggerbucket,Milk:Water=109:71Rule2Ifndifferentvesselsofsizesx1,x2,…,xnarefilledwiththemixtureofPandQintheratiop1:q1,p2:q2,……,pn:qnandcontentofallthesevesselsaremixedinonelargevessel,then

𝑄𝑢𝑎𝑛𝑖𝑡𝑦𝑜𝑓𝑃𝑄𝑢𝑎𝑛𝑡𝑖𝑡𝑦𝑜𝑓𝑄 =

𝑃$𝑋$𝑃$ + 𝑄$

+ 𝑃<𝑋<𝑃< + 𝑄<

+. . . …… . . 𝑃9𝑋9𝑃9 + 𝑄9

𝑄$𝑋$𝑃$ + 𝑄$

+ 𝑄<𝑋<𝑃< + 𝑄<

+. . . …… . . 𝑄9𝑋9𝑃9 + 𝑄9

ExampleThreebucketsofsize2liter,4literand5litercontainingthemixtureofmilkandwateraremixedintoabiggerbucket.Iftheproportionofmilkandwaterintheglassesare3:1,2:3and4:2thenfindtheproportionofmilkandwaterinthebiggerbucket.Sol:Let’ssayPstandsformilkandQstandsforwater,So,p1:q1=3:1,x1=2p2:q2=2:3,x2=4p3:q3=4:2x3=5,so

𝑄𝑢𝑎𝑛𝑖𝑡𝑦𝑜𝑓𝑃𝑄𝑢𝑎𝑛𝑡𝑖𝑡𝑦𝑜𝑓𝑄 =

𝑃$𝑋$𝑃$ + 𝑄$

+ 𝑃<𝑋<𝑃< + 𝑄<

+. . . …… . . 𝑃9𝑋9𝑃9 + 𝑄9

𝑄$𝑋$𝑃$ + 𝑄$

+ 𝑄<𝑋<𝑃< + 𝑄<

+. . . …… . . 𝑄9𝑋9𝑃9 + 𝑄9

wxy9zb{|}!wxy9bzb{|}w

= H×;H�AE

;×=;�HE

=×B=�;

A×;H�AE

H×=;�HE

;×B=�;

=109/71

𝑄𝑢𝑎𝑛𝑖𝑡𝑦𝑜𝑓𝑀𝑖𝑙𝑘𝑄𝑢𝑎𝑛𝑡𝑖𝑡𝑦𝑜𝑓𝑤𝑎𝑡𝑒𝑟 = 193/137

Soinbiggerbucket,Milk:Water=193:137

Page 71: Maths book part 2 obtains Rs. 1100 after lending out Rs.x at 5% per annum for 2 years and obtains Rs. 1800 after lending out Rs.y at 10% per annum for 2 years. Find the value of x

71

Rule3:RemovalandReplacementIfavesselcontains“x”litresofliquidAandif“y”litresbewithdrawnandreplacedbyliquidB,thenif“y”litresofthemixturebewithdrawnandreplacedbyliquidB,andtheoperationisrepeated‘n’timesinall,then:

𝑄𝑢𝑎𝑛𝑡𝑖𝑡𝑦𝑜𝑓𝑙𝑖𝑞𝑢𝑖𝑑𝐴𝑎𝑓𝑡𝑒𝑟𝑛𝑡ℎ𝑜𝑝𝑒𝑟𝑎𝑡𝑖𝑜𝑛𝑖𝑛𝑖𝑡𝑖𝑎𝑙𝑞𝑢𝑎𝑛𝑖𝑡𝑦𝑜𝑓𝑙𝑖𝑞𝑢𝑖𝑑𝑜𝑓𝐴 = 1 −

𝑦𝑥

9

ExampleAcontaineriscontaining80literofwine.8literofwinewastakenoutfromthiscontainerandreplacedbywater.Thisprocesswasfurtherrepeatedtwotimes.Howmuchwineisthereinthecontainernow?Sol:Herex=80,y=8andn=3,so

𝑄𝑢𝑎𝑛𝑡𝑖𝑡𝑦𝑜𝑓𝑙𝑖𝑞𝑢𝑖𝑑𝐴𝑎𝑓𝑡𝑒𝑟3𝑟𝑑𝑜𝑝𝑒𝑟𝑎𝑡𝑖𝑜𝑛80 = 1 −

880

C

= 58.32𝑙𝑖𝑡𝑟𝑒𝑠Quantityofwineafter3rdoperation=58.32liters.Rule4:pgramofingredientsolutionhasa%ingredientinit.Toincreasetheingredientcontenttob%inthesolution?Quantityofingredientneedtobeadded=c(�Wy)

$%%W�

Example125literofmixtureofmilkandwatercontains25%ofwater.Howmuchwatermustbeaddedtoittomakewater30%inthenewmixture?Sol:Let’ssayp=125,b=30,a=25SofromtheequationQuantityofingredientneedtobeadded=$<?(C%W<?)

$%%WC%

Quantityofwaterneedtobeadded=8.92liter.

Page 72: Maths book part 2 obtains Rs. 1100 after lending out Rs.x at 5% per annum for 2 years and obtains Rs. 1800 after lending out Rs.y at 10% per annum for 2 years. Find the value of x

72

Ex.1.HowmanykgofRs.33akgcoffeehastobemixedwithRs.27akgcoffeetomakeamixtureof60kgworthRs.32akg?Sol. Bytheruleofallegation

Totalweightoftwotypeofcoffeeis=60kgTotalweightofcoffeeofRs.33akgis=?

`×60=50kg

Ex.2.Twovesselscontainmilkandwatermixedintheratioof2:3and3:4.Findtheratioinwhichthesetwoaretobemixedtogetanewmixtureinwhichtheratioofmilktowateris7:10Sol.Byalligationrule

Requiredratio= <

$$_∶ $[?=10:7.

Ex.3.Inacocktailtherewasamixtureofalcoholandjuice.Thepriceofthemixtureof30ml.ofalcoholand90ml.ofjuiceisRs.120.Thepriceofthealcoholincreasesby20%andthepriceofjuiceincreasesby30%.NowthesamecocktailcostRs.150.Whataretheoldpricesof100ml.ofalcoholand100mlofjuice?Sol.Letpriceof1mlofalcoholisRs.xandpriceof1mlofjuiceisRs.y.Therefore,30x+90y=120orx+3y=4...............(i)Aftertheincreaseinthepriceofalcoholby20%andofjuiceby30%,thenewpricesofcocktailis36x+117y=150...............(ii)Now,multiply(i)by36&subtracting(i)from(ii),wegety=2/3,putthisvalueofyin(i),wegetx=2Now,priceof1mlofalcoholisRs.2andpriceof1mlofjuiceisRs.2/3Therefore,priceof100mlofalcoholand100mlofjuiceisRs.200&Rs.66.67Ex.4.TheaveragemonthlysalaryoflabourersandsupervisorsinafactoryisRs.1,250permonth,whereastheaveragemonthlysalaryofallthe6supervisorsisRs.2,450.IftheaveragemonthlysalaryofthelabourersisRs.950,findthenumberoflabourers.

Page 73: Maths book part 2 obtains Rs. 1100 after lending out Rs.x at 5% per annum for 2 years and obtains Rs. 1800 after lending out Rs.y at 10% per annum for 2 years. Find the value of x

73

Sol.

ByAlligationSo,theratioof�xc�T�z�|T�

�y�|xT��=300/1200=$

>

Becausethereare6supervisors,Numberoflaboures=4 6=24.

Ex.5.TwoliquidsAandBareintheratio5:1incontainerXand1:3incontainerY.InwhatratioshouldthecontentsofthetwocontainersbemixedsoastoobtainamixtureofAandBintheratio1:1?Sol.

Wecansolvethisbyalligation.Butwhilewealligate,wehavetobecarefulthatithastobedonewithrespecttoanyoneofthetwoliquids,viz,eitherAorB.Wecanverifythatinbothcases,wegetthesameresult.WithrespecttoliquidA:TheproportionofAinthefirstvesselis?`andthatinthesecondvesselis$

>,andwefinallyrequire$

<partofA.Nowsolvewiththehelp

ofallegationEx.6.Adishonestmilkmanmixes20litresofwaterwith80litresofmilk.Aftersellingone-fourthofthismixture,headdswatertoreplenishthequantitythathehassold.Whatisthecurrentproportionofwatertomilk?Sol.100litresofmixturehave80lmilkand20lwater.When25litresofmixtureistakenoutthenitcontains20lmilkand5lwater.Somilkleft=60litresandwaterleft=15litresIf25litresofwaterisaddedthenratioofwatertomilk=40:60=2:3Ex.7.Agoldbiscuitof11kgcontainedonly82%goldandtherestcopper.Ifitwasalloyedwithanothergoldbiscuitandtheresultingalloyhasamassof28kgandcontained90%ofgold,findthepercentageofgoldinthesecondbiscuit.Sol.Weightof1stbiscuit=11kg%ageofgoldin1stbiscuit=82%Weightof2ndbiscuit=28–11=17kgLetthe%ageofgoldin2ndbiscuit=x%ByAlligation

Page 74: Maths book part 2 obtains Rs. 1100 after lending out Rs.x at 5% per annum for 2 years and obtains Rs. 1800 after lending out Rs.y at 10% per annum for 2 years. Find the value of x

74

Ratio= $�b�z��xzb

<9��z��xzb= $$

$X

$$$X= DW_%

[

17x–1530=8817x=1618x=95.17Ex.8.Therearetwoalloysofgold,silverandplatinum.Thefirstalloyisknowntocontain40percentofplatinumandthesecondalloy26percentofsilver.Thepercentageofgoldisthesameinbothalloys.Havingalloyed150kgofthefirstalloyand250kgofthesecond,wegetanewalloythatcontains30percentofgold.Howmanykilogramofplatinumisthereinthenewalloy?Sol.Sincethepercentageofgoldinbothalloysisthesame,anymixtureofthetwowillcontainthesamepercentageconcentrationofgold.Hence,wegetFirstalloy=Gold:Silver:Platinum=30:30:40Andsecondalloy=Gold:Silver:Platinum=30:26:44So,theweightofplatinuminthefirstalloy=150 >%

$%%=60kgAndtheweightofplatinumin

thesecondalloy=250 >>$%%

=110kgSo,thetotalweightofplatinuminthemixture=60+110kg=170kgEx.9.Avesselcontains40litresofmilkandamilkmandelivers10litrestothefirsthouseandaddsanequalquantityofwater.Hedoesexactlythesameatthesecondandthirdhouse.Whatistheratioofmilkandwater,whenhehasfinisheddeliveringatthethirdhouse?Sol.Totalamountofmilk=40litresAmountofmilkreplacedbywater=10litres

Now,accordingtothequestion:�z��T��yz9z9�y}b�TC|c�Tybz|9�z9zbzy��xy9bzb{|}�z��

= 1 − $%>%

C=<X

`>

Itmeanstheratioofmilktowater=27:37Ex.10.ThreecontainersA,BandCarehavingmixturesofmilkandwaterintheratioof1:5,3:5and5:7respectively.Thecapacitiesofthecontainersareintheratio5:4:5.Ifthecontentsofallthethreecontainersaremixedtogether,findtheratioofmilktowaterintheresultingmixture.Sol.Assumethatthereare500,400and500litresrespectivelyinthe3containers.Thenwehave,83.33,150and208.33litresofmilkineachofthethreecontainers.Thus,thetotalmilkis441.66litres.Hence,theamountofwaterinthemixtureis

Page 75: Maths book part 2 obtains Rs. 1100 after lending out Rs.x at 5% per annum for 2 years and obtains Rs. 1800 after lending out Rs.y at 10% per annum for 2 years. Find the value of x

75

1400–441.66=958.33litres.Hence,theratioofmilktowateris441.66:958.33=53:115.Ex.11.TwoboxesAandBwerefilledwithcoffeeandchicorymixedinAintheratioof5:3andinBintheratioof7:3.Whatquantitymustbetakenfromeachtoformamixturewhichshallcontain6kgofcoffeeand3kgofchicory(inkgrespectively)?Sol.SupposeXkgaretakenfromA;then(9–X)kgaretakenfromB.5/8ofthemixtureinAand7/10ofthemixtureinBiscoffee.5X/8+7/10(9–X)=6,HenceX=44kgmustbetakenfromAand5kgfromBEx.12.Kalicharangetssomecoinsmadeofanalloyofgoldandsilver.Thealloywithaweightof100gmcontains20%ofgold.Whatpieceofanothergold-silveralloycontaining60%ofsilvermustbealloyedwiththefirstpieceofalloyinordertoobtainanewalloywiththe32%ofgold?Sol.Goldisthefirstalloy=20%Goldisthesecondalloy=40%Byruledalligation:

Theratioofthetwoalloys=2:3Thequantityofsecondalloys=100×C

<=150gm.

Ex.13.Atoyweighing24gramsisanalloyoftwometalsandisworthRs.174,butiftheweightsofmetalsinalloybeinterchanged,thetoywouldbeworthRs.162.IfthepriceofonemetalbeRs.8pergram,findthepriceoftheothermetalinthealloyusedtomakethetoy.Sol.Let’ssayonemetalisxgrams.Sotheotheroneis[24–x]grams.Nowaccordingtothequestion8x+(24–x)y=1748(24–x)+xy=1628x+24y–xy=174192–8x+xy=16224y+192=33624y=144y=6.Rs.6pergram.

Page 76: Maths book part 2 obtains Rs. 1100 after lending out Rs.x at 5% per annum for 2 years and obtains Rs. 1800 after lending out Rs.y at 10% per annum for 2 years. Find the value of x

76

Ex.14.Johnstolesomewhiskyfromabottlewhichhisfatherhadkeptawayinthecupboard.Thisbottleofwhiskycontained50%alcohol.Johnreplacedwhathehaddrunkbyanotherbrandofwhiskywhichcontainedonly25%alcohol.Thewhiskyinthebottlenowhasonly35%alcohol.HowmuchofthebottledidJohnsteal?Sol.

��z��{�zb�?%%y��|�|���z��{�zb�<?%y��|�|�

= $%$?= <

C

Thesetwospiritsmustbemixedintheratio2:3inordertogetthewhiskyof35%alcohol.Thismeansthatonly2/5oftheoriginalbottleofwhiskyisleftandsoJohnhasstolen3/5ofthebottle.Ex.15. Alumpoftwometalsweighing18gramsisworthRs.74butiftheirweightsbeinterchanged,itwouldbeworthRs.60.10.IfthepriceofthegoldbeRs.7.20pergram,findtheweightoftheothermetalinthemixture.Sol.Lettheweightofthegoldbexgm&Weightoftheothermetal=(18–x)gmLetthepriceoftheothermetalbeRs.y/gm.Now,accordingtothequestion7.20x+y(18–x)=747.20(18–x)+yx=60.10Solvingthesetwoequationssimultaneously,x=10,y=0.25Quantityoftheothermetal=18–10=8gramsEx.16.If20litersofwaterisaddedtoatubalreadyfilledwith120litersofdilutedmilk,theconcentrationofmilkinthetubwillbe80%.Whatistheconcentrationofmilkinthetubbeforeaddingwatertothetub?Sol.$<%�E<%×%

$>%=80

120k=80×140k=<[%

C=93.33%.

Ex.17.Theratioofmilktowaterinamixtureis2:3andtheratioofwatertomilkinanothermixtureis3:4.If70litresofeachtypeofmilkisaddedto100litresofpuremilk,whatwillbetheratioofmilktowaternow?Sol.70litresoffirsttype

Page 77: Maths book part 2 obtains Rs. 1100 after lending out Rs.x at 5% per annum for 2 years and obtains Rs. 1800 after lending out Rs.y at 10% per annum for 2 years. Find the value of x

77

<?×70=28litresofmilk

70litresofsecondtype=>X×70=40litresofmilk

100litresofpuremilk=100litresofmilkTotaloutof240litres,thereis168litresofmilk.So,theremaining72litresiswater.So,requiredratio=$`[

X<= X

C

Ex.18.Analloycontainszincandtininratio3:4.Anotheralloycontainszincandsilverinratio4:3.Ifboththealloysaremeltedandmixedinequalratio,whatwillbetheratioofzincandtininthenewalloy?Sol.7kgofthefirstalloycontains3kgofzincand4kgofTin.7kgofthesecondalloycontains4kgofzincand3kgofsilver,Ifboththesealloysareadded,wewillget7kgofzincand4kgoftin.So,7:4.Ex.19.Analloycontainszinc,copperandIronintheratio4:3:2.If20kgofcopperisaddedto90kgofthisalloy,whatwillbetheratioofzincandcopperinthealloy?Sol.Initially,Zinc=40kgCopper=30kgIron=20kgIf20kgofcopperisadded,thetotalquantityofcopperwillbe50kg,andtheratioofzincandcopper=4:5.Ex.20.From200litersof80%concentratedmilk,30litersistakenoutandwaterisadded.ThenewconcentrationofmilkinthemixtureisSol.If30litersistakenoutfrom200litersitwillbe170ltsoutofthis;Themilkpercentis80%.Sothevolumeofmilkintheremainingsolution=170× [%

$%%=136lts

Percentageofmilkinthefinalsolution=$C`<%%

×100=68.1Ex.21.InwhatproportionmustwheatatRs.1.80perkgbemixedwithwheatatRs.2.50perkgtogetamixtureworthRs.2.0perkg?Sol.ApplytheRuleofAlligation:⇒requiredratio=5:2

Page 78: Maths book part 2 obtains Rs. 1100 after lending out Rs.x at 5% per annum for 2 years and obtains Rs. 1800 after lending out Rs.y at 10% per annum for 2 years. Find the value of x

78

Ex.22.20litresofamixturecontains40%alcoholandtherestiswater.If5litresofwaterisadded,thepercentageofalcoholinthenewmixturewillbeSol.Totalamountofmixture=20lt&amountofAlcohol=40%=8ltNow,AccordingtotheQuestion,Newmixture=20+5=25lits.Percentageofalcohol=(8/25)×100=32%Ex.23.TwovesselsA,Bcontainmilkandwatermixedintheratioof5:2and8:5respectively.Findtheratioinwhichthesemixturesaretobemixedtogetanewmixturecontainingmilkandwaterintheratio9:4.Sol.Ratioofmilk&waterinAis5:2&inBis8:5.Thenewmixtureshouldhavemilk&waterintheratio9:4.RatioofmilkinAis?

X&inBis [

$C.Theresultingmixtureshouldcontainmilk&

waterintheratio9:4.Henceratioofmilkshouldbe9/13Nowbyapplyingtheruleofalligationwegettheratio $

$C× _$

<= X

<

HenceA&Bshouldbemixedintheratio7:2sothattheresultingmixturecontainMilk&waterintheratio9:4.

Ex.24.McGrathearnedaprofitofRs.300byselling100kgofamixtureofAandBtypesofrice,atatotalpriceofRs.1,100.WhatwastheproportionofAandBtypesofriceinthemixtureifthecostpricesofAandBareRs.10andRs.5perkgrespectively?Sol.TotalamountwhichMcgrathgetaftersellingthe100kgofrice=Rs.1,100&Profit=Rs.300∴C.P.of100kg=1,100-300=Rs.800C.P.of1kg=Rs.8Now,ApplytheRuleofAlligation:

𝑄y𝑄�

= 8 − 510 − 8 =

32

Page 79: Maths book part 2 obtains Rs. 1100 after lending out Rs.x at 5% per annum for 2 years and obtains Rs. 1800 after lending out Rs.y at 10% per annum for 2 years. Find the value of x

79

Exercise

1.InwhatproportionmustagrocermixwheatatRs.2.04perkgandRs.2.88perkgsoastomakeamixtureofworthRs.2.57perkg?a.4:3b.3:4c.2:3d.3:2e.Noneofthese2.Amixtureofcertainquantityofmilkwith8Lofwaterisworth45paiseperlitre.Ifpuremilkbeworth54paiseperlitre,howmuchmilkisthereinthemixture?a.50Lb.20Lc.30Ld.60Le.Noneofthese3.Atraderhas50kgofpulses,partofwhichhesellsat8%profitandrestat18%profit.Hegains14%onthewhole.Whatisthequantitysoldat18%profit?a.50kgb.20kgc.30kgd.40kge.Noneofthese4.Amerchanthas2000kgofrice,partofwhichhesellsat36%profitandtherestat16%profit.Hegains28%onthewhole.Findthequantitysoldat16%a.500kgb.900kgc.800kgd.400kge.Noneofthese5.Abutlerstolewinefromabuttofsherrywhichcontained80%ofspiritandhereplaceditbywinecontainingonly32%spirit.Thenthebuttwasof48%strengthonly.Howmuchofthebuttdidhesteal?a.2/3b.3/2c.1/3d.4/5e.Noneofthese6.TwovesselsPandQcontainwineandwaterintheratiosof5:2and8:5,respectively.Findtheratioinwhichthesemixturesaretobemixedtogetanewmixturecontainingwineandwaterintheratioof9:4a.4:3b.7:2c.2:7d.3:4e.Noneofthese7.Acontainerisfilledwithliquid,6partofwhicharewaterand10partmilk.Howmuchofthemixturemustbedrawnoffandreplacedwithwatersothatthemixturemaybehalfwaterandhalfmilk?a.2/3b.3/2c.1/5d.4/5e.Noneofthese8.Abutlerstolewinefromabuttofsherrywhichcontains15%ofspiritandhereplacedwhathehadstolenbywinecontaining6%ofspirit.Thebuttwasthen9%strongonly.Howmuchofthebuttdidhestole?a.2/3b.3/2c.1/5d.4/5e.Noneofthese9.Ravina’ssavingsandexpenditureareintheratioof2:3.Herincomeincreasesby10%.Herexpenditurealsoincreasesby12%.Byhowmanypercentdoeshersavingincrease?a.6%b.7%c.8%d.6%e.Noneofthese10.Inazoo,therearelionsandparrots.Ifcounted,thereare100headsand290legs.Howmanyparrotsarethere?a.45b.50c.55d.60e.Noneofthese

Page 80: Maths book part 2 obtains Rs. 1100 after lending out Rs.x at 5% per annum for 2 years and obtains Rs. 1800 after lending out Rs.y at 10% per annum for 2 years. Find the value of x

80

11.4Laredrawnfromacontainerfullofmilkandisthenfilledwithwater.Thisoperationisperformedthreemoretimes.Theratioofthequantityofmilkleftinthecontainerandthatofwateris16:65.Howmuchmilkdidthecontainerholdinitially?a.11Lb.13Lc.14Ld.20Le.Noneofthese12.AmixtureworthRs.3.25akgisformedbymixingtwotypesofflour,onecostingRs.3.10perkgwhiletheotherRs.3.60perkg.Inwhatproportionmusttheyhavebeenmixed?a.4:3b.7:3c.2:5d.3:4e.Noneofthese13.Inwhatproportionmustwaterbemixedwithmilksoastogain20%bysellingthemixtureatthecostpriceofthemilk?(Assumethatwaterisfreelyavailable)a.1:5b.5:1c.2:3d.3:2e.Noneofthese14.Prabodhbought30kgofriceattherateofRs.8.50perkgand20kgofriceattherateofRs.9.00perkg.Hemixedthetwo.Atwhatprice(approx)perkgshouldhesellthemixtureinordertoget20%profita.Rs.10.25b.Rs.10c.Rs.10.5d.11e.Noneofthese15.Prabhupurchased30kgofriceattherateofRs.17.50perkgandanother30kgriceatacertainrate.HemixedthetwoandsoldtheentirequantityattherateofRs.18.60perkgandmade20percentoverallprofit.Atwhatpriceperkgdidhepurchasethelotofanother30kgrice?a.Rs.13.25b.Rs.13c.Rs.13.5d.12e.Noneofthese16.Inwhatproportionmustwaterbemixedwithmilksoastogain50%bysellingthemixtureatthecostpriceofthemilk?(Assumethatwaterisfreelyavailable)a.1:2b.2:1c.2:3d.3:2e.Noneofthese17.Amixturecontainsspiritandwaterintheratio3:2.Ifitcontains3litresmorespiritthanwater,thequantityofspiritinthemixtureisa.10Lb.13Lc.14Ld.9Le.Noneofthese18.Avesselcontains50litresmilk.Themilkmandelivers10litrestothefirsthouseandaddsanequalquantityofwater.Hedoesexactlythesameatthesecondandthirdhouse.Whatistheratioofmilkandwaterwhenhehasfinisheddeliveringatthethirdhouse?a.61:64b.64:61c.23:29d.29:23e.Noneofthese19.Severallitresofacidweredrawnofffroma54litresvesselfullofacidandanequalamountofwaterisadded.Againthesamevolumeofthemixturewasdrawnoffandreplacedbywater.Asaresult,thevesselcontained24litresofpureacid.Howmuchacidwasdrawnoffinitially?a.10Lb.13Lc.18Ld.9Le.Noneofthese20.Amixtureofacertainquantityofmilkwith16litresofwaterisworth90paisaperlitre.Ifpuremilkbeworth108paiseperlitre,howmuchmilkinthereisthemixture?a.100Lb.90Lc.80Ld.110Le.Noneofthese

Page 81: Maths book part 2 obtains Rs. 1100 after lending out Rs.x at 5% per annum for 2 years and obtains Rs. 1800 after lending out Rs.y at 10% per annum for 2 years. Find the value of x

81

21.Amixtureofacertainquantityofmilkwith32litresofwaterisworthRs.1.50perlitre.IfpuremilkbeworthRs.4.50perlitre,howmuchmilkisthereinthemixture?a.10Lb.16Lc.18Ld.9Le.Noneofthese22.Theamountofwheat@Rs.610perquintalwhichshouldbeaddedto126quintalsofwheatcostingRs.285perquintalsothat20%maybegainedbysellingthemixtureatRs.480perquintalwillbea.50kgb.20kgc.30kgd.40kge.Noneofthese23.Threevesselscontainequalmixturesofmilk&waterintheratio6:1,5:2,&3:1respectively.Ifallthesolutionsaremixedtogether,theratioofmilktowaterinthefinalmixturewillbea.19:65b.65:19c.23:29d.29:23e.Noneofthese24.TomixspiritsworthRs.8,Rs.6andRs.3pergallonformakingamixtureworthRs.5pergallon,howmuchofeachquantityistobetaken?a.2:1:2b.1:2:2c.2:2:1d.1:1:2e.Noneofthese25.Twoequalcontainersarefilledwithamixtureofwaterandalcohol.Oneofthemcontainsthreetimesasmuchalcoholastheother.Themixturesinthetwocontainersarethenmixedanditisfoundthattheratioofwatertoalcoholis3:2.Findtheratioofwatertoalcoholineachoftheoriginalcontainersa.2:3,1:4b.3:2.4:1c.2:3,4:1d.3:2,1:4e.Noneofthese26.A100litresolutionofmilkandwatercontainswaterandmilkintheratio1:4.10%ofthesolutionisremovedandreplacedbymilktwice,insuccession.Whatwouldbethequantityofwaterintheresultantsolution?a.16Lb.16.2Lc.15Ld.17Le.Noneofthese27.Acontainercontains240litresofwine.80litresistakenoutofthecontainereverydayandanequalquantityofwaterisputintoit.Findthequantityofthewinethatremainsinthecontainerattheendofthefourthdaya.47.4Lb.46Lc.48Ld.47Le.Noneofthese28.AcancontainsamixtureoftwoliquidsAandBintheratio7:5.When9litresofmixturearedrawnoffandcanisfilledwithB,theratioofAandBbecomes7:9.HowmanylitresofliquidAwascontainedbythecaninitially?a.16Lb.20Lc.21Ld.25Le.Noneofthese29. Analloyofaluminumandleadcontains37%aluminumbyweight.Theweightofaluminumwhichmustbeaddedto400poundsofthisalloytomakethepercentageofaluminum70isa.440poundsb.400poundsc.370pounds d.380poundse.Noneofthese30.Amixtureof20kgsofspiritandwatercontains10%water.Howmuchwatermustbeaddedtothismixturetoraisethepercentageofwaterto25%?

Page 82: Maths book part 2 obtains Rs. 1100 after lending out Rs.x at 5% per annum for 2 years and obtains Rs. 1800 after lending out Rs.y at 10% per annum for 2 years. Find the value of x

82

a.4kgb.5kgc.7kgd.3kge.Noneofthese31.InwhatratiomusttwokindsofteaworthRs.18andRs.28perkgbemixedsothatbysellingthemixtureatRs.32perkgtheremaybeagainof20%?a.2:15b.15:2c.13:2d.2:13e.Noneofthese32.Agrocermixes26kgofteawhichcostsRs.20akgwith30kgofteawhichcostsRs.36akgandsellsthemixtureatRs.30akg.Hisprofitpercentisa.4%b.6%c.5%d.4.5%e.Noneofthese33.TeaworthRs.126perkgandRs.135perkgaremixedwithathirdvarietyintheratio1:1:2.IfthemixtureisworthRs.153perkg,thepriceofthethirdvarietyperkgwilla.Rs.175.5b.Rs.177.5c.Rs.176.5d.Rs.174.5e.Noneofthese34.Nikitabought30kgofwheatattherateofRs.4.75perkg,40kgofwheatattherateofRs.4.25perkgandmixedthem.ShesoldthemixtureattherateofRs.4.45perkg.Hertotalprofitorlossinthetransactionwasa.Rs.1profitb.Rs.1lossc.Rs.2profitd.Rs.2losse.Noneofthese35.AsumofRs.41wasdividedamong50boysandgirls.Eachboygets90psandeachgirlgets65ps.Findthenumberofboys.a.15b.35c.16d.34e.Noneofthese36.Acontainercontains80litresofwine.8litresistakenoutofthecontainereverydayandanequalquantityofwaterisputintoit.Findthequantityofthewinethatremainsinthecontainerattheendofthe2nddaya.64.8Lb.64Lc.48Ld.60Le.Noneofthese37.Avesselcontains50litresmilk.Themilkmandelivers5litrestothefirsthouseandaddsanequalquantityofwater.Hedoesexactlythesameatthesecondandthirdhouse.Whatistheratioofmilkandwaterwhenhehasfinisheddeliveringatthethirdhouse?a.271:729b.729:271c.64:61d.61:64e.Noneofthese38. VesselsAandBcontainmilkandwaterintheratio4:5and5:1respectively.InwhatproportionshouldquantitiesbetakenfromthevesselsAandBrespectivelytoformamixtureinwhichmilk:waterisintheratio5:4?a.5:3b.3:5c.5:2d.2:5e.Noneofthese39.Twoequaltanksfilledwithmixtureofmilkandroohafzaintheproportionof2:1and1:1respectivelywereemptiedintoathirdtank.Whatistheproportionofroohafzaandmilkinthethirdtank?a.7:3b.3:7c.7:5d.5:7e.Noneofthese40.Twovesselscontainmilkandwatermixedintheratioof2:3and3:4.Findtheratioinwhichthesetwoaretobemixedtogetanewmixtureinwhichtheratioofmilktowateris

Page 83: Maths book part 2 obtains Rs. 1100 after lending out Rs.x at 5% per annum for 2 years and obtains Rs. 1800 after lending out Rs.y at 10% per annum for 2 years. Find the value of x

83

7:10a.7:10b.10:7c.3:7d.7:3e.Noneofthese41.ApersonhasachemicalofRs.50perlitre.InwhatratioshouldwaterbemixedinthatchemicalsothataftersellingthemixtureatRs.40perlitrehemaygetaprofitof50%?a.7:8b.15:2c.13:2d.8:7e.Noneofthese42.Amerchanthas1000kgofsugar,partofwhichhesellsat8%profitandtherestat18%profit.Hegains14%onthewhole.Thequantitysoldat18%profitis:a.400kgb.360kgc.600kgd.640kge.Noneofthese43.Alumpoftwometalsweighting18gramsisworthRs.87butiftheirweightsbeinterchanged,itwouldbeworthRs.78.60.IfthepriceofonemetalbeRs.6.70pergram,findtheweightoftheothermetalinthemixturea.8gb.6gc.4gd.10ge.Noneofthese44.Intwoalloys,copperandzincarerelatedintheratiosof4:1and1:3.10kgof1stalloy,16kgof2ndalloyandsomeofpurecopperaremeltedtogether.Analloywasobtainedinwhichtheratioofcoppertozincwas3:2.Findtheweightofthenewalloya.32kgb.35kgc.30kgd.40kge.Noneofthese45.Freshfruitcontains72%wateranddryfruitcontained20%water.Howmuchdryfruitfrom100kgoffreshfruitcanbeobtained?a.32kgb.35kgc.30kgd.40kge.Noneofthese46.Amixturecontainsacidandwaterintheratioof1:4andanothercontainsthemintheratio4:5.Ifwewantathirdmixturefromtheaboveoneswitharatio2:7,theproportioninwhichthetwovarietiesshouldbemixedisa.2:15b.15:2c.10:1d.1:10e.Noneofthese47.InwhatratiomustagrocermixtwovarietiesofteaworthRs.60akgandRs.65akgsothatbysellingthemixtureatRs.68.20akghemaygain10%?a.5:4b.4:5c.2:3d.3:2e.Noneofthese48.AcertainproductCismadeoftwoingredientsAandBintheproportionof2:5.ThepriceofAisthreetimesthatofB.TheoverallcostofCisRs.5.20perkgincludinglabourchargesof80paisaperkg.FindthecostofBperkg?a.Rs.2.80b.Rs.1.80c.Rs.3.80d.Rs.0.80e.Noneofthese49.TwovesselsAandBcontainmixtureofspiritandwater.Amixtureof3partsfromAand2partsfromBisfoundtocontain29%ofspiritandamixtureof1partfromAand9partsfromBisfoundtocontain34%ofspirit.FindthepercentageofspiritinAandBa.32,25b.35,25c.30,40d.40,30e.Noneofthese50.Twolumpscomposedofgold,silverandcoppertogetherweight20kg,onelumpcontains

Page 84: Maths book part 2 obtains Rs. 1100 after lending out Rs.x at 5% per annum for 2 years and obtains Rs. 1800 after lending out Rs.y at 10% per annum for 2 years. Find the value of x

84

gold75%andsilver31.25gramsperkg.Theothercontainsgold85%andsilver30gramsperkg.Thetotalquantityofsilverintwolumpsis617.5grams.Ifthetwolumpsaremeltedandformedintoone,whatpercentofgoldwillitcontain?a.82%b.80%c.74%d.78%e.Noneofthese51.Threevesselswhosecapacitiesareas5:3:2arecompletelyfilledwithmilkmixedwithwater.Theratioofmilkandwaterinthemixtureofvesselsareas3:2,2:1and3:1respectively.Findthepercentageofwaterinthenewmixtureobtainedwhen1/3rdoffirst,1/2ofsecondand2/3rdofthethirdvesselsistakenoutandmixedtogethera.25%b.33.33%c.20%d.30%e.Noneofthese52.HowmanykilogramofsugarcostingRs.9perkgmustbemixedwith27kgofsugarcostingRs.7perkgsothattheremaybeagainof10%bysellingthemixtureatRs.9.24perkg?a.63kgb.62kgc.60kgd.64kge.Noneofthese53.Whatwillbetheratioofpetrolandkeroseneinthefinalsolutionformedbymixingpetrolandkerosenethatarepresentinthreevesselsintheratio4:1,5:2and6:1respectively?a.22:83b.83:22c.10:1d.datainadequatee.Noneofthese54.AmixtureworthRs.3.25akgisformedbymixingtwotypesofflour,onecostingRs.3.10perkgwhiletheotherRs.3.60perkg.Inwhatproportionmusttheyhavebeenmixed?a.2:15b.15:2c.7:3d.3:7e.Noneofthese55.10gallonsaredrawnfromacaskfullofwine.Itisthenfilledwithwater.10gallonsofthemixturearedrawnandthecaskisagainfilledwithwater.Thequantityofwinenowleftinthecasktothatofthewaterinitis16:9.Howmuchdoesthecaskhold?a.50gallonsb.20gallonsc.30gallonsd.40gallonse.Noneofthese56.Theratioofkerosenetopetrolin100kgsofadulteratedpetrolnormallyusedbythreewheelers7:25.Theamountofkerosenetobeaddedto100kgsofadulteratedpetroltomaketheratio9:25isa.6.5kgb.6.25kgc.7kgd.7.25kge.Noneofthese57.Inwhatproportionmustwaterbemixedwithmilktogain20%bysellingitatthecostpricea.2:15b.15:2c.1:5d.5:1e.Noneofthese58.ThecontentsinthebeakersAandBare80cm3ofsugarand80cm3ofsandrespectively.Now20cm3ofsugaristakenoutfromAandputintoB.Afterthroughmixing,20cm3ofthemixtureistakenoutfromBandputintoA.FindthepercentageofsandinthemixtureinAa.25%b.33.33%c.20%d.30%e.Noneofthese59.Avesselisfullofamixtureofkeroseneandpetrolinwhichthereis18%kerosene.Eightlitresaredrawnoffandthenthevesselisfilledwithpetrol.Ifthekeroseneisnow15%,howmuchdoesthevesselhold?a.24Lb.48Lc.36Ld.42Le.Noneofthese

Page 85: Maths book part 2 obtains Rs. 1100 after lending out Rs.x at 5% per annum for 2 years and obtains Rs. 1800 after lending out Rs.y at 10% per annum for 2 years. Find the value of x

85

60.Twosolutionsof90%and97%purityaremixedresultingin21litresofmixtureof94%purity.Howmuchisthequantityofthefirstsolutionintheresultingmixture?a.10Lb.16Lc.18Ld.9Le.Noneofthese61.Alokbought25kgand35kgofacommodity@Rs.6perkgandRs.7perkgrespectively.Hemixedthetwoquantitiesandsoldthemixture@Rs.6.75perkg.Howmuchistheoverallgain/losstohim?a.10gainb.16lossc.18gaind.9losse.Noneofthese62.Sureshpurchased20kgofteaofonevarietyatRs.30perkgand30kgofteaofanothervarietyatRs.25perkg.Howmuchisthetotalprofit?a.Rs.20b.Rs.25c.Rs.30d.datainadequatee.Noneofthese63.HowmanylitresofRs.16perlitrewineshouldamerchantaddwith25litresofRs.20perlitrewine,sothatheearns25%bysellingthemixtureatRs.22?a.35Lb.32.5Lc.35Ld.37.5Le.Noneofthese64.Twovesselscontainmixturesofmilkandwater.Onecontains80%milkandtheothercontains60%milk.Theproportioninwhichtheyshouldbemixedtogetliquidwith75%milkand25%waterisa.3:1b.1:3c.2:3d.3:2e.Noneofthese65.Goldis19timesasheavyaswaterandcopperis9timesasheavyaswater.Inwhatratioshouldthesebemixedtogetanalloy15timesasheavyaswater?a.3:1b.1:3c.2:3d.3:2e.Noneofthese66.Twokindsofteawereprepared.Inthefirst,30gramsofsugarwasmixedwith190gramsoftea.Inthesecond,40gramsofsugarwasmixedwith270gramsoftea.Whichteawouldbesweeter?Ifthetwokindsofteaaremixedtogether,determinethepercentageofteainthemixturea.Ist,86><

?Cb.2nd,86><

?Cc.Ist,85><

?Cd.2nd,85><

?Ce.Noneofthese

67.Amixtureiscomposedof4partsofbrandyand1partofwater,1kgofwaterisadded,andtheresultingmixture3timesasmuchbrandyaswater.Findthequantityofbrandyintheoriginalmixturea.10kgb.12kgc.16kgd.8kge.Noneofthese68.Adishonestmilkmanmixed1litreofwaterforevery3litresofmilkandthusmadeup36litresofmilk.Ifhenowadds15litresofmilktothemixture,findtheratioofmilkandwaterinthenewmixturea.3:14b.14:3c.12:3d.3:12e.Noneofthese69.AdishonestmilkmanpurchasedmilkatRs.10perlitresandmixed5litresofwaterinit.BysellingthemixtureattherateofRs.10perlitreheearnsaprofitof25%.Thequantityofthemixturethathehadwas?

Page 86: Maths book part 2 obtains Rs. 1100 after lending out Rs.x at 5% per annum for 2 years and obtains Rs. 1800 after lending out Rs.y at 10% per annum for 2 years. Find the value of x

86

a.40Lb.20Lc.30Ld.10Le.Noneofthese70.Acisterncontains50litresofwater.5litresofwateristakenoutofitandreplacedbywine.Theprocessisrepeatedagain.Findtheproportionofwineandwaterintheresultingmixturea.19:81b.81:19c.81:100d.100:81e.Noneofthese71.Acontainerhasacapacityof20gallonsandisfullofspirit.4gallonsofspiritisdrawnoutandthecontainerisagainfilledwithwater.Thisprocessisrepeated5times.Findouthowmuchspiritisleftintheresultingmixturefinally?a.6C>`

C>Cgallonsb.6C>`

`<?gallonsc.5C>`

`<?gallonsd.6C>C

`<?gallonse.Noneofthese

72.Avesselisfullofrefinedoil.1/4oftherefinedoilistakenoutandthevesselisfilledwithmustardoil.Iftheprocessisrepeated4timesand10litresofrefinedoilisfinallyleftinthevessel,whatisthecapacityofthevessel?a.<?`%

[%litresb.<??_

[$litresc.<?`%

[$litresd.<>`%

[$litrese.Noneofthese

73.InwhatratioshouldtwoqualitiesofcoffeepowderhavingtheratesofRs.47perkgandRs.32perkgbemixedinordertogetamixturethatwouldhavearateofRs.37perkg?a.3:1b.1:3c.2:1d.1:2e.Noneofthese74.AsumofRs.36.90ismadeupof90coinsthatareeither20paisacoinsor50paisacoins.Findouthowmany20paisacoinsarethereinthetotalamount?a.25b.20c.27d.26e.Noneofthese75.Adiscountgrocerprofessestosellpurebutteratcostprice,buthemixesitwithadulteratedfatandtherebygains25%.Findthepercentageofadulteratedfatinthemixturesassumingthatadulteratedfatisfreelyavailablea.25%b.20%c.27%d.26%e.Noneofthese76.Athiefstealsfourgallonsofliquidsoapkeptinatraincompartment’sbathroomfromacontainerthatisfullofliquidsoap.Hethenfillsitwithwatertoavoiddetection.Unabletoresistthetemptationhesteals4gallonsofthemixtureagain,andfillsitwithwater.Whentheliquidsoapischeckedatastationitisfoundthattheratiooftheliquidsoapnowleftinthecontainertothatofthewaterinitis36:13.Whatwastheinitialamountoftheliquidsoapinthecontainerifitisknownthattheliquidsoapisneitherusednoraugmentedbyanybodyelseduringtheentireperiod?a.25gallonsb.20gallonsc.28gallonsd.26gallonse.Noneofthese77.Amixtureof70litresofalcoholandwatercontains10%ofwater.Howmuchwatermustbeaddedtotheabovemixturetomakethewater12.5%oftheresultingmixture?a.2Lb.4Lc.3Ld.1Le.Noneofthese78.Amixtureof20litresofbrandyandwatercontains10%water.Howmuchwatershouldbeaddedtoittoincreasethepercentageofwaterto25%?

Page 87: Maths book part 2 obtains Rs. 1100 after lending out Rs.x at 5% per annum for 2 years and obtains Rs. 1800 after lending out Rs.y at 10% per annum for 2 years. Find the value of x

87

a.2Lb.4Lc.3Ld.1Le.Noneofthese79.AmerchantpurchasedtwoqualitiesofpulsesattherateofRs.200perquintalandRs.260perquintal.In52quintalsofthesecondquality,howmuchpulseofthefirstqualityshouldbemixedsothatbysellingtheresultingmixtureatRs.300perquintal,hegainsaprofitof25%?a.20qb.22qc.24qd.26qe.Noneofthese80.AmanbuysmilkatRs.8.5perlitreanddilutesitwithwater.Hesellsthemixtureatthesamerateandthusgains11.11%.Findthequantityofwatermixedbyhimineverylitreofmilka.0.111Lb.1Lc.2Ld.0.5Le.Noneofthese81.Therearetwomixturesofhoneyandwater,thequantityofhoneyinthembeing25%and75%ofthemixture.If2gallonsofthefirstaremixedwith3gallonsofthesecond,whatwillbetheratioofhoneytowaterinthenewmixture?a.9:11b.11:9c.2:3d.3:2e.Noneofthese82.Therearetwokindsofalloysoftinandcopper.Thefirstalloycontainstinandcoppersuchthat93:33%ofitistin.Inthesecondalloythereis86.66%tin.Whatweightofthefirstalloyshouldbemixedwith25kgofthesecondalloysoastomakecontaining90%oftin?a.25kgb.20kgc.27kgd.26kge.Noneofthese83.Twocontainersofequalcapacityarefullofamixtureofoilandwater.Inthefirst,theratioofoiltowateris4:7andintheseconditis7:11.Nowboththemixturesaremixedinabiggercontainer.Whatistheresultingratioofoiltowater?a.149:247b.247:149c.243:144d.144:243e.Noneofthese84.Twovesselscontainspiritandwatermixedrespectivelyintheratioof1:3and3:5.Findtheratioinwhichthesearetobemixedtogetanewmixtureinwhichtheratioofspirittowateris1:2a.3:1b.1:3c.2:1d.1:2e.Noneofthese85.ApersonpurchasedacupboardandacotforRs.18,000.Hesoldthecupboardataprofitof20%andthecotataprofitof30%.Ifhistotalprofitwas25%,findthecostpriceofthecupboarda.Rs.9000b.Rs.15000c.Rs.3000d.Rs.5000e.Noneofthese

Page 88: Maths book part 2 obtains Rs. 1100 after lending out Rs.x at 5% per annum for 2 years and obtains Rs. 1800 after lending out Rs.y at 10% per annum for 2 years. Find the value of x

88

Answers

1.E 2.E 3.C 4.C 5.A 6.B 7.C 8.A 9.B 10.C11.E 12.B 13.A 14.C 15.C 16.A 17.D 18.B 19.C 20.C21.B 22.E 23.B 24.D 25.C 26.B 27.A 28.C 29.A 30.A31.D 32.C 33.A 34.B 35.D 36.A 37.B 38.C 39.C 40.B41.D 42.C 43.A 44.B 45.B 46.C 47.D 48.A 49.B 50.D51.B 52.A 53.D 54.C 55.A 56.B 57.C 58.C 59.B 60.D61.A 62.D 63.D 64.A 65.D 66.A 67.B 68.B 69.B 70.A71.B 72.C 73.D 74.C 75.B 76.C 77.A 78.B 79.D 80.A81.B 82.A 83.A 84.D 85.A

Page 89: Maths book part 2 obtains Rs. 1100 after lending out Rs.x at 5% per annum for 2 years and obtains Rs. 1800 after lending out Rs.y at 10% per annum for 2 years. Find the value of x

89

Simplification(Algebra)

IMPORTANTCONCEPTSI.'BODMAS'Rule:Thisruledepictsthecorrectsequenceinwhichtheoperationsaretobe executed,soastofindoutthevalueofagivenexpression. Here,'B'standsfor'bracket','O'for'of','D'for'division'and'M'for'multiplication','A' for'addition'and'S'for'subtraction'. Thus,insimplifyinganexpression,firstofallthebracketsmustberemoved,strictlyin theorder(),{}and[].Afterremovingthebrackets,wemustusethefollowingoperationsstrictlyintheorder:(1)of(2)division(3)multiplication(4)addition(5)subtraction.II.Modulusofarealnumber:Modulusofarealnumberaisdefinedas |a|={a,ifa>0 -a,ifa<0 Thus,|5|=5and|-5|=-(-5)=5.III.Virnaculum(orbar):WhenanexpressioncontainsVirnaculum,beforeapplyingthe 'BODMAS'rule,wesimplifytheexpressionundertheVirnaculum.SURDSANDINDICESIMPORTANTFACTSANDFORMULAE1.LAWSOFINDICES:(i)amxan=am+n(ii)am–an=am-n(iii)(am)n=amn(iv)𝑎�G = 𝑎 �G (v)(ab)n=anbn

(vi) y�

9=y

G

�G

(vii)a0=12.SURDS:Letabearationalnumberandnbeapositiveintegersuchthata1/n= 𝑥G isirrational.Then 𝑥G iscalledasurdofordern

Ex.1.𝟔𝟒𝟏𝟐𝟏W

𝟗𝟔𝟒

𝟖𝟏𝟏E

𝟑𝟖

[Bank2000]

Page 90: Maths book part 2 obtains Rs. 1100 after lending out Rs.x at 5% per annum for 2 years and obtains Rs. 1800 after lending out Rs.y at 10% per annum for 2 years. Find the value of x

90

Sol.

�=A;AW

p�=

�AAE

H�= `>

;W _×$<$$<$×`>

× [×$$[;E C×$$

= `>;W C×C×$$×$$[×[×$$×$$

× [×$$`>ECC

= `>;WCC;[×[×$$×$$

× [×$$`>ECC

= `>ECC (`>WCC)[×[×$$×$$

× [×$$`>ECC

=64 + 3388 =

3188

Ex.2.If𝟐𝒏 = 𝟒𝟓E𝟒𝟓E𝟒𝟓

𝟑𝟓E𝟑𝟓× 𝟔𝟓E𝟔𝟓E𝟔𝟓E𝟔𝟓

𝟐𝟓E𝟐𝟓E𝟐𝟓andn>0,thenthevalueofn2is[SSC2000]

Sol.>BE>BE>B

CBECB× `BE`BE`BE`B

<BE<BE<B= C(>B)

<(CB)× >(`B)C(<B)

>B

CB× <×`B

<B= >B

CB× `B

<=

(>;×>H)`B

CB×<== >H×`B

CB

>H×`×`×`×`×`C×C×C×C×C

4C×2? = 2`×2? = 2$$29 = 2$$So,n=11andn2=121

Ex.3.Simplify𝒂𝟏𝟐E𝒂O

𝟏𝟐

𝟏W𝒂+ (𝟏W𝒂

O𝟏𝟐)𝟏E 𝒂

[Bank2000]Sol.

yA;EyO

A;

$Wy+

$WyOA;

$E y= y

A;EyO

A;

$EyA; $Wy

A;+

$WyOA;

$EyA;

1 − 𝑎 = 1< − 𝑎A;<= 1 + 𝑎

A; 1 − 𝑎W

A;

yA;EyO

A; E $WyO

A; $WyO

A;

$EyA; $Wy

A;

yA;EyO

A;E$WyO

A;Wy

A;E$

$Wy

= <$Wy

Ex.4.Whichofthefollowingequationsareequivalent?[Bank2004]

(i)(𝟏𝟐𝑴 + 𝟑

𝟐𝑵)𝟐(ii)𝟒

𝟗N2+𝟏

𝟒M2+𝟐

𝟑MN(iii) (iv)

Sol.

Page 91: Maths book part 2 obtains Rs. 1100 after lending out Rs.x at 5% per annum for 2 years and obtains Rs. 1800 after lending out Rs.y at 10% per annum for 2 years. Find the value of x

91

$<𝑀 + C

<𝑁

<= >

_𝑁< + $

>𝑀< + <

C𝑀𝑁

So,(i)and(ii)aresame.$>

$`_𝑁< + 𝑀< + [

C𝑀𝑁

$>𝑀 + >

C𝑁

<----(iv)

Options(i),(ii)and(iv)areequal.Ex.5.Inaclassof63children,thechildrenareseatedinrowsandcolumnssuchthattherearetwochildrenineachcolumnthatthenumberofchildrenseatedineachrow.Howmanychildrenarethereineachrow?[Asst.Grade2005]Sol.Letthenumberofchildreninrow=xThenumberofchildreninthecolumn=(x+2)Accordingtothequestion,X(x+2)=63x2+2x–63=0x2+9x–7x–63=0x(x–9)–7(x–9)=0(x–9)(x–7)=0x=9or-7(Negativevalueisnotpossible)Thereare9childrenineachrow.Ex.6.If 𝒙 − 𝒚 = 𝟑and 𝒙 + 𝒚 = 𝟏𝟗,then 𝒙𝒚 =?[SSC2003]Sol. 𝑥 − 𝑦 = 3-----eq1𝑥 + 𝑦 = 19-----eq2

Addingboththeequations,2 𝒙 = 𝟐𝟐𝑥 = <<

<= 11

Subtractingboththeequations,-2 𝑦 = −16𝑦 = 8

So, 𝑥𝑦 = 11×8 = 88Ex.7.2 C

D×𝑦 $

<= 7 C

>[Bank2002]

Sol.Takingthequotients2,yand72y=7andy=3Substitutethevalueofy,2 C

D×3 $

<= 7 C

>

2 CD=

XH=CA;= 2 C

$>

Page 92: Maths book part 2 obtains Rs. 1100 after lending out Rs.x at 5% per annum for 2 years and obtains Rs. 1800 after lending out Rs.y at 10% per annum for 2 years. Find the value of x

92

So,comparingtheequations,x=14andy=3Ex.8.AmanhasRs.480inthedenominationsofone-rupeenotes,five-rupeenotesandten-rupeenotes.Thenumberofnotesofeachdenominationisequal.Whatisthetotalnumberofnotesthathehas?Sol.Letnumberofnotesofeachdenominationbex.Thenx+5x+10x=48016x=480x=30.Hence,totalnumberofnotes=3x=90.Ex.9.TherearetwoexaminationsroomsAandB.If10studentsaresentfromAtoB,thenthenumberofstudentsineachroomisthesame.If20candidatesaresentfromBtoA,thenthenumberofstudentsinAisdoublethenumberofstudentsinB.ThenumberofstudentsinroomAis:Sol.LetthenumberofstudentsinroomsAandBbexandyrespectively.Then,x-10=y+10x-y=20....(i)andx+20=2(y-20)x-2y=-60....(ii)Solving(i)and(ii)weget:x=100,y=80.TherequiredanswerA=100.Ex.10.a-b=3anda2+b2=29,findthevalueofab.Sol.2ab=(a2+b2)-(a-b)2=29-9=20ab=10.Ex.11.Inaregularweek,thereare5workingdaysandforeachday,theworkinghoursare8.AmangetsRs.2.40perhourforregularworkandRs.3.20perhoursforovertime.IfheearnsRs.432in4weeks,thenhowmanyhoursdoesheworkfor?Sol.Supposethemanworksovertimeforxhours.Now,workinghoursin4weeks=(5x8x4)=160.

160x2.40+xx3.20=4323.20x=432-384=48x=15.Hence,totalhoursofwork=(160+15)=175Ex.12.Amanhassomehensandcows.Ifthenumberofheadsbe48andthenumberoffeetequals140,thenthenumberofhenswillbe:Sol.Letthenumberofhensbexandthenumberofcowsbey.Then,x+y=48....(i)and2x+4y=140 x+2y=70....(ii)Solving(i)and(ii)weget:x=26,y=22.Therequiredanswer=26.

Page 93: Maths book part 2 obtains Rs. 1100 after lending out Rs.x at 5% per annum for 2 years and obtains Rs. 1800 after lending out Rs.y at 10% per annum for 2 years. Find the value of x

93

Ex.13.Thepriceof2sareesand4shirtsisRs.1600.Withthesamemoneyonecanbuy1sareeand6shirts.Ifonewantstobuy12shirts,howmuchshallhehavetopay?Sol.LetthepriceofasareeandashirtbeRs.xandRs.yrespectively.Then,2x+4y=1600....(i)andx+6y=1600....(ii)Divideequation(i)by2,wegetthebelowequation.x+2y=800.---(iii)Nowsubtract(iii)from(ii)x+6y=1600(-)x+2y=800----------------4y=800----------------Therefore,y=200.Nowapplyvalueofyin(iii)x+2x200=800x+400=800Thereforex=400Solving(i)and(ii)wegetx=400,y=200.Costof12shirts=Rs.(12x200)=Rs.2400.Ex.14.4/15of5/7ofanumberisgreaterthan4/9of2/5ofthesamenumberby8.Whatishalfofthatnumber?Sol.Letthenumberbex.then >

$?of?

Xofx–>

_of<

?ofx=8or >

<$x– [

>?x=8

or( ><$− [

>?)x=8or`%W?`

C$?x=8or >

C$?x=8

orx=[×C$?>

=630orx/2=315Hencerequirednumber=315.Ex.15.Therationinacampof500menisenoughtolastfor8weeks.Howlongwilltherationlastiftherewere400men?Sol.500menwilleattherationin8weeks.1manwilleattherationin8 500weeks=4000weeks.400menwilleattherationin4000/400=10weeks.Ex.16.If2x+3y+z=55,x–y=4andy–x+z=12,thenwhatarethevaluesofx,yandz?Sol.Thegivenequationsare:2x+3y+z=55…(i);x+z–y=4…(ii);y–x+z=12…(iii)Subtracting(ii)from(i),weget:x+4y=51…(iv)Subtracting(iii)from(i),weget:3x+2y=43…(v)Multiplying(v)by2andsubtracting(iv)fromit,weget:5x=35orx=7.Puttingx=7in(iv),weget:4y=44ory=11.Puttingx=7,y=11in(i),weget:z=8.

Page 94: Maths book part 2 obtains Rs. 1100 after lending out Rs.x at 5% per annum for 2 years and obtains Rs. 1800 after lending out Rs.y at 10% per annum for 2 years. Find the value of x

94

Ex.17.If4n+4n-1=20,thenthevalueofnnisSol.4x+4x-1=204x=42X=2xX=22=4Ex.18.Ifx+y=1,findthevalueofx3+y3+3xy.Sol.(x+y)3=x3+y3+3xy(x+y)puttingvalue

(1)3=x3+y3+3xy(1)x3+y3+3xy=1.

Ex.19.Divide17intotwopartssothatthedifferencebetweensquaresofnumbersis119.Sol. LetonenumberbexAndsecondnumber=17–xAccordingtothegivencondition(x)2–(17–x)2=119x2–(289+x2–34x)=119x2–289–x2+34x–119=034x–408=034x=408x=>%[

C>=12

Onenumber=12Secondnumber=12–7=5Ex.20.Theproductoftwoconsecutiveoddnumbersis575.Findthesumofthenumbers.Sol. Letfirstoddnumber=nSecondoddnumber=n+2Accordingtocondition,n(n+2)=575n2+2n–575=0n2+25n–23n–575=0n(n+25)–23(n+25)=0(n+25)(n–23)=0n=–25Rejected1stoddnumber=23Secondoddnumber=25.Sum=23+25=4Ex.21.Inagroupofchildren,everychildgivesagifttoeveryother.Ifthenumberofgiftsis272,findthenumberofchildren.Sol.Letnbethenumberofchildrenn(n–1)=272n2–n–272=0n2–17n+16n–272=0n(n–17)+16(n–17)=0

Page 95: Maths book part 2 obtains Rs. 1100 after lending out Rs.x at 5% per annum for 2 years and obtains Rs. 1800 after lending out Rs.y at 10% per annum for 2 years. Find the value of x

95

(n+16)(n–17)=0n=–16,17Numberofchildren=17Ex.22.Inacricketmatch,WarnetookonewicketmorethantwicethenumberofwicketstakenbyBrettLee.Iftheproductofthenumberofwicketstakenbythesetwois10,findthenumberofwicketstakenbyeach.Sol.LetthenumberofwicketstakenbyBrettLeebexNumberofwicketstakenbyWarne=2x+1Accordingtothequestion,x(2x+1)=102x2+x=102x2+x–10=02x2+5x–4x–10=0x(2x+5)–2(2x+5)=0Eitherx–2=0 x=2Or2x+5=0 x=W?

<(rejected)

NumberofwicketstakenbyBrettLee=2andnumberofwicketstakebyWarne=2(2)+1=5Ex.23.AandBhaveacertainnumberofstamps.AsaidtoB,“Ifyougivemeoneofyourstamps,weshallhaveequalnumberofstamps.”Breplied,“Ifyougivemeoneofyourstamps,Ishallhavetwiceasmanyasyouwillbeleftwith”.FindthetotalnumbersofstampsAandBhave.Sol.LetthenumberofstampsAhadbexThenumberofstampsBhad=yThen,x+1=y–1…(i)Andy+1=2(x–1)…(ii)Solving(i)and(ii),wegetx=5Andy=7Hence,thenumberofstampsAandBhaveis5+7=12.Ex.24.Anumberconsistsoftwodigits,thesumofthedigitsbeing10.If18issubtractedfromthenumber,thedigitsarereversed.Findthenumber.Sol. Letthedigitatunit’splacebeyandthedigitatten’splacebexOriginalnumber=10x+y

Whendigitsarereversedthenumberbecomes=10y+xAccordingtotheconditions,x+y=10…(i)10x+y–18=10y+x…(ii)Equation(ii)reducesto9x–9y=18x–y=2…(iii)Adding(i)&(iii),weget2x=12x=6

Page 96: Maths book part 2 obtains Rs. 1100 after lending out Rs.x at 5% per annum for 2 years and obtains Rs. 1800 after lending out Rs.y at 10% per annum for 2 years. Find the value of x

96

Putx=6inequation(iii);y=4Originalnumber=64Ex.25.Thedifferenceoftwodigitsofanumberis3.If4timesthenumberisequalto7timesthenumberobtainedbyreversingthedigits,findtheoriginalnumber.Sol.Letthedigitatunit’splacebeyAndthedigitatten’splacebexOriginalnumber=10x+yNumberobtainedbyreversingthedigits=10y+xAccordingtothecondition,x–y=3(i)4(10x+y)=7(10y+x)40x+4y=70y+7x33x=66yx=``

CC𝑦

x=2yPutx=2yin(i)weget2y–y=3y=3Puty=3ineqn(i);x=6Numberis63Ex.26.Abagcontains89coinsof50paisaand25paisa.IfthetotalworthofthesecoinsisRs28.50,findthenumberof25paisacoinsand50paisacoins.Sol.Letnumberof50paisacoinsbexAndnumberof25paisacoinsbeyx+y=89…(i)50x+25y=28502x+y=114…(ii)Solving(i)and(ii),wegetx=35Putvalueofxinequation(i)y=89–25y=64Ex.27.Solvethefollowingequationforx.2x2–14x+14=–10Sol.2x2–14x+14=–102x2–14x+24=02x2–8x–6x+24=02x(x–4)–6(x–4)=0(2x–6)(x–4)=0x=3,4Ex.28.Janehas5dollarsmorethanTom.Ifbothofthemhaveatotalof13dollars,findtheamountwithTom.Sol.LetJ–numberofdollarswithJane

Page 97: Maths book part 2 obtains Rs. 1100 after lending out Rs.x at 5% per annum for 2 years and obtains Rs. 1800 after lending out Rs.y at 10% per annum for 2 years. Find the value of x

97

T–numberofdollarswithTomJ=T+5andJ+T=13(T+5)+T=132T+5=132T=13–52T=8T=4Ex.29.Findx3+y3+z3–3xyz,ifx+y+z=9andxy+yz+zx=11.Sol.x3+y3+z3–3xyz=(x+y+z)(x2+y2+z2–xy–yz–zx)=(x+y+z)[(x+y+z)2–3(xy+yz+zx)]=9[81–3(11)]=9×[81–33]=9×48=432.Ex.30.Simplify:b–[b–(a+b)–{b–(b–a+b)+2a}]Sol.Givenexpression=b–[b–(a+b)–{b–(b–a+b)+2a}]=b–[b–a–b–{b–2b+a+2a}]=b–[–a–{b–2b+a+2a}]=b–[–a–{–b+3a}]=b–[–a+b–3a]=b–[–4a+b]=b+4a–b=4a.

Ex.31.Evaluate: (𝟐𝟒𝟖 +√(𝟓𝟏 + √(𝟏𝟔𝟗))).

Sol.Givenexpression= (248 +√(51 + 13)= (248 +√64)= (248 + 8)

= (256)=16.Ex.32.If𝒂 + 𝟏

𝒂W𝟐=4,thenthevalueof(𝒂 − 𝟐)𝟐 + 𝟏

(𝒂W𝟐)𝟐

Sol.weneedsquareof(a-2)and1/(a-2)directsquareofbothsidewillnotgivetheresult,weneed(𝑎 − 2) + $

yW<togetthedesireresult,𝑎 + $

yW<=4

so,(𝑎 − 2) + $yW<

=2andnowsquarebothsides,(𝑎 − 2)< + $

(yW<);+2=4

or(𝑎 − 2)< + $(yW<);

=2

Ex.33.Ifxy(x+y)=1Thenthevalueof 𝟏

𝒙𝟑𝒚𝟑− 𝒙𝟑 − 𝒚𝟑

Sol.𝑥𝑦 𝑥 + 𝑦 = 1or 𝑥 + 𝑦 = $D{

Page 98: Maths book part 2 obtains Rs. 1100 after lending out Rs.x at 5% per annum for 2 years and obtains Rs. 1800 after lending out Rs.y at 10% per annum for 2 years. Find the value of x

98

Sincewehavetoget $DH{H

− 𝑥C − 𝑦Cwhichisintermofx³andY³

so,wehavetotakecubeonboththesides

(𝑥 + 𝑦)C = 1

𝑥C𝑦C

X3+y3+3xy(x+y)= $DH{H

or3xy(x+y)= $DH{H

− 𝑥C − 𝑦C

or,3x1= $DH{H

− 𝑥C − 𝑦C(since𝑥𝑦 𝑥 + 𝑦 = 1)

so, $DH{H

− 𝑥C − 𝑦C = 3(answer)Ex.34.Ifa3−b3−c3=0,thenthevalueofa9−b9−c9−3a3b3c3is,Sol.wheneveryoufindcubeofthreevariablesand3abcor3xyzalwaysthinkabouttheformula,when,x+y+z=0then,x3+y3+z3–3xyz=0herex=a3,y=-b3,z=-c3therefore,a9–b9–c9–3a3b3c3=0Ex.35.If(x+7954×7956)beasquarenumber,thenthevalueof'x'isSol.Theproblemis,(x+7954×7956)wehavetoconvertinasquare,anysumissquareofanumberifwegetitintheabovetwoformsherefocuson7954x7956,theycanbecomeasquareformulax2+y2+2xyorx2+y2-2xyif,eitheryouwrite,7954=7956-2or7956=7954+2let’stake7956=7954+2then,(x+7954×7956)=x+7954x(7954+2)=x+79542+2x1x7954clearlyifyouputx=1,itwillgiveaperfectsquare,1+79542+2x1x7954(1+7954)2therefore,x=1Ex.36.If𝒙

𝒂= 𝟏

𝒂− 𝟏

𝒙,thenthevalueofx–x2is

Sol.youhavetofindx–x2andgiven,Dy= $

y− $

D

youneedxandx²,commondenominatorashouldbeononeside,therefore,Dy− $

y= − $

Dor,DW$

y= − $

D

crossmultiplication,x2–x=-aORx–x2=a

Page 99: Maths book part 2 obtains Rs. 1100 after lending out Rs.x at 5% per annum for 2 years and obtains Rs. 1800 after lending out Rs.y at 10% per annum for 2 years. Find the value of x

99

Exercise

1.Whensimplified,theproduct 1 −$<

1 −$C

1 −$>… . . 1 − $

9gives

a.$9b. $

9E$c.<

9d. $

9E<e.Noneofthese

2.IfD

{= C

>,thenfindthevalueof{WD

{ED

a.C>b.C

<c.$

Cd.<

?e.Noneofthese

3.Atotalof324coinsof20paiseand25paisemakeasumofRs.71.Thenumberof25paisecoinsisa.125b.123c.124d.130e.Noneofthese4.IfD

{=<

$thevalueofDE{

DW{is

a.2b.1c.3d.4e.Noneofthese5.IfaxbxcmeansyE�

�forallnumbersexcept0,then(axbxc)xaxbisequalto

a.yE�Ey���

b.yE�Wy���

c.yW�Ey���

d.yE�Ey�y�

e.Noneofthese6.Ifa=4.965,b=2.343,c=2.622,findthevalueofa3-b3-c3-3abca.2b.1c.3d.4e.Noneofthese7.Thevalueof$<%+ $

C%+ $

><+ $

?`+ $

X<+ $

_%+ $

$$%+ $

$C<is

a.$Xb.$

[c.<

?d.$

`e.Noneofthese

8.Thesquarerootof %.C><×%.`[>

%.%%%C><×%.%%%$X$is

a.4000b.40000c.400000d.4000000e.Noneofthese9.Simplify:$$

;W?;×<?×<

a.7$`b.7 C

$%c.7 $

$%d.6 $

$%e.Noneofthese

10.Thefluidcontainedinabucketcanfillfourlargebottlesorsevensmallbottles.Afulllargebottleisusedtofillandemptysmallbottle.Whatfractionofthefluidisleftoverinthelargebottlewhenthesmalloneis-full?a.$

Xb.$

[c.C

Xd.$

`e.Noneofthese

11.AmanearnsRs.20onthefirstdayandspendsRs.15onthenextday.HeagainearnsRs.20onthethirddayandspendsRs.15onthefourthday.Ifhecontinuestosavelikethis,howsoonwillhehaveRs.60inhand?

Page 100: Maths book part 2 obtains Rs. 1100 after lending out Rs.x at 5% per annum for 2 years and obtains Rs. 1800 after lending out Rs.y at 10% per annum for 2 years. Find the value of x

100

a.on17thdayb.on16thdayc.on15thdayd.on18thdaye.Noneofthese12.Afires5shotstoB’s3butAkillsonly1birdinthe3shotswhileBkillsonly1birdin2shots.WhenBhasmissed27times,Ahaskilledhowmanybirds?a.20birdsb.10birdsc.30birdsd.40birdse.Noneofthese13.Afterdistributingthesweetsequallyamong25children,8sweetsremain.Hadthenumberofchildrenbeen28,22sweetswouldhavebeenleftafterequallydistributing.Whatwasthetotalnumberofsweets?a.350b.356c.250d.358e.Noneofthese14.SanketearnstwiceasmuchinthemonthofMarchasineachoftheothermonthsoftheyear.WhatpartofhisentireannualearningwasearnedinMarch?a. <

$Cb. C

$Xc. >

$Cd. <

$Xe.Noneofthese

15.Thevalueof

C <CE `

− > C`E <

+ `<E C

a.2b.0c.3d.4e.Noneofthese16. %.%?;E%.>$;E%.%XC;

%.%%?;E%.%>$;E%.%%XC;is

a.200b.100c.300d.400e.Noneofthese17.Thevalueof(%.CCXE%.$<`)

;W(%.CCXW%.$<`);

%.CCX×%.$<`is

a.2b.1c.3d.4e.Noneofthese18.Thesimplifiedvalueof 900 − 0.09 − 0.000009isa.29.697b.29.197c.29.597d.29.797e.Noneofthese

19.Thesimplifiedvalueof 5 + 11 + 19 + 29 + 49

a.2b.1c.3d.4e.Noneofthese20.Thevalueof99 _?

__×99is

a.9986b.9896c.9976d.9996e.Noneofthese21. $

CW [− $

[W X+ $

XW `− $

`W ?+ $

?W<

a.2b.1c.3d.4e.Noneofthese

Page 101: Maths book part 2 obtains Rs. 1100 after lending out Rs.x at 5% per annum for 2 years and obtains Rs. 1800 after lending out Rs.y at 10% per annum for 2 years. Find the value of x

101

22.%.$<<<?E%.%<X%.<?W%.$?E%.%_

isequaltoa.0.2b.0.1c.0.7d.0.8e.Noneofthese23.Thesumoftheseries(1+0.6+0.06+.0006+.00006+…)isa.1 <

Cb.2 <

Cc.1 $

Cd.2 $

Ce.Noneofthese

24. %.%%_×%.%C`×%.%$`×%.%[%.%%<×%.%%%[×%.%%%<

isequalto

a.26b.18c.36d.42e.Noneofthese25. >.>$×%.$`

<.$×$.`×%.<$issimplifiedto

a.2b.1c.3d.4e.Noneofthese26.<?`×<?`W$>>×$>>

$$<isequalto

a.200b.100c.300d.400e.Noneofthese

27. CEDE CWDCEDW CWD

=2thenxisequalto

a.$C<b. <

$Cc. ?

$<d.$<

?e.Noneofthese

28.Simplifiedformof 𝑥WHB

BWBH

?

is

a.xb.(x+1)c.(x–1)d.x3e.Noneofthese29.4/15of5/7ofanumberisgreaterthan4/9of2/5ofthesamenumberby8.Whatishalfofthatnumber?a.320b.315c.420d.350e.Noneofthese

30.Simplify: C`

`× H=|}

;p÷

o=|}

;o

?

a.20b.18c.13d.14e.Noneofthese31.If1.5a=0.04b,then�Wy

�Eyisequalto

a.XC

XXb.XC

X[c.X<

XXd.X$

XXe.Noneofthese

32. %.$

;W%.%$;

%.%%%$+ 1 isequalto

a.200b.100c.130d.140e.Noneofthese33. C.%`HW$._[H

C.%`;EC.%`×$._[E$._[;isequalto

Page 102: Maths book part 2 obtains Rs. 1100 after lending out Rs.x at 5% per annum for 2 years and obtains Rs. 1800 after lending out Rs.y at 10% per annum for 2 years. Find the value of x

102

a.1.8b.1.5c.1.3d.1.08e.Noneofthese34. ?.`<>HE>.CX`H

?.`<>×?.`<>W ?.`<>×>.CX` E>.CX`×>.CX`?

a.20b.10c.13d.14e.Noneofthese35.Giventhat3.718= $

%.<`[_then $

%.%%%CX$[isequalto

a.2679b.2789c.2689d.2537e.Noneofthese36.(53×87+159×21+106×25)isequaltoa.10500b.10550c.10400d.10600e.Noneofthese37. $

$.>+ >

>.X+ $

X.$+ $

$%.$C+ $

$C.$`isequalto

a. ?$`b. <

$Cc. $

$<d. ?

$<e.Noneofthese

38. >

$?𝑜𝑓 ?

[×6 + 15 − 10is

a.2b.1c.3d.4e.Noneofthese39.Whatis1/6thof3?a.0.5b.1.5c.0.33333d.1.333e.Noneofthese40.Multiply0.932by100a.90.1b.92.4c.93.2d.89e.Noneofthese41.Divide0.045by100a.0.45b.0.045c.0.0045d.0.00045e.Noneofthese42.If2x=5and3y=8,then>D

_{isequalto

a. ?$`b. <

$Cc. $

$<d. ?

$<e.Noneofthese

43.Thesumoffirst50positiveintegersis1275.Whatisthesumoftheintegersfrom51to100?a.3770b.3789c.3775d.3540e.Noneofthese44. $

<− $

C+ $

C− $

>+ $

>+ $

<isequalto

a.2b.1c.3d.4e.Noneofthese

45. $.?<.?

<isequalto

a.0.45b.0.36c.0.49d.0.16e.Noneofthese46.Find3+0.3+0.03+0.003a.3.5b.3.666c.3.33333d.3.333e.Noneofthese

Page 103: Maths book part 2 obtains Rs. 1100 after lending out Rs.x at 5% per annum for 2 years and obtains Rs. 1800 after lending out Rs.y at 10% per annum for 2 years. Find the value of x

103

47.Simplify15.876-(2.49+4.056)÷$

<

a.18<?b.18<

Cc.18$

?d.18$

Ce.Noneofthese

48.Simplify%.>[÷%.$<E%.%>×<?

%.%?

a.200b.100c.130d.140e.Noneofthese49.Simplify<C

$_× $_$X×85

a.115b.100c.125d.120e.Noneofthese50.FindPintheexpression,if !

$E A

A� ~AO~

= 1

a.2b.1c.3d.4e.Noneofthese

51.SimplifyH;÷

A;×

H;

H;÷

A;|}

H;÷ $

[

a.20b.18c.13d.24e.Noneofthese

52.Findthevalueof(CECECEC)÷C?E?E?E?÷?

a.C

>b.?

>c.$

>d.X

>e.Noneofthese

53.Findthevalueof?E?×$_W$?WX

$C×$CW$?`

a.2b.1c.3d.4e.Noneofthese54.Simplify1÷[1+1÷{1+1÷(1÷1)}]a.$

>b.<

Cc.$

Cd.<

?e.Noneofthese

55.Simplify$>;B+ $

<=p

a. X

$$b. X

$Cc.X

_d. X

$?e.Noneofthese

56.If217×15=3255,then2.17×0.15isa.1.3255b.0.3255c.0.03255d.0.003255e.Noneofthese57.24.315×256.2×0.00019isthesameasa.243.15×2.562×0.019b.243.15×2.562c.2430.15×2.562×0.019d.243.15×2.562×0.00019e.Noneofthese

Page 104: Maths book part 2 obtains Rs. 1100 after lending out Rs.x at 5% per annum for 2 years and obtains Rs. 1800 after lending out Rs.y at 10% per annum for 2 years. Find the value of x

104

58.If13+23+33+…..+93=2025,thenfindthevalueof(0.1)3+(0.2)3+…..+(0.9)3a.2.0025b.2.025c.20.025d.2.00025e.Noneofthese59.If12+22+32+….+102=385,thenfindthevalueof(0.11)2+(0.22)2+….+(0.99)2a.4.6585b.4.06585c.4.006585d.4.0006585e.Noneofthese60.Ifx#y=x+y,thenfindthevalueof(3#4)#3a.20b.10c.30d.40e.Noneofthese61.Ifa+bbedefinedbytherelationy

;

�,whereb≠0.Findthevalueof(24+16)+4

a.322b.314c.236d.324e.Noneofthese62.If $

%.`<?=1.6.Findthevalueof $

%.%%%`<?

a.150b.170c.160d.180e.Noneofthese63.Ifx=2yandy=2z/3,whatisthevalueofzintermsofx?a.CD

?b.CD

>c.?D

>d.CD

Xe.Noneofthese

64.%.[C

%.[Coisapproximatelyequalto

a.2b.1c.3d.4e.Noneofthese65.Ifxispositiveand6 −𝑥< = $?

$`, 𝑡ℎ𝑒𝑛 𝑥 =?

a.C>b.C

<c.$

Cd.<

?e.Noneofthese

66.Findthevalueof$?

$E ?<

− $W ?<

a.2b.1c.3d.4e.Noneofthese67.$[

`%(0.1254)isequalto

a.0.3762b.0.03762c.0.3765d.0.003762e.Noneofthese68.Simplify[1-2(3-4)-1]-1a.4b.3c.1d.2e.Noneofthese69.Simplify $

<+ $

<C>− $

<X[− C

>

a.<XC<b.<?

CCc.<X

CCd.<?

C<e.Noneofthese

Page 105: Maths book part 2 obtains Rs. 1100 after lending out Rs.x at 5% per annum for 2 years and obtains Rs. 1800 after lending out Rs.y at 10% per annum for 2 years. Find the value of x

105

70.Simplify1 + $<E A

H�AH

a.CC<?b.C?

<Cc.CC

<Cd.C?

<?e.Noneofthese

71.Findx,if6 − 5 − 𝑥 − 2 − C

<= 3

a.C>b.C

<c.$

Cd.<

?e.Noneofthese

72.Simplify0.00175÷0.025÷0.07a.2b.1c.3d.4e.Noneofthese73.Whatleastfractionshouldbeaddedto $

<×C+ $

C×>+ $

>×?+ ⋯+ $

<$×<<,sothattheresultis

equalto1a.C

>b.C

<c.$

Cd.<

?e.Noneofthese

Answers

1.A 2.E 3.C 4.C 5.A 6.E 7.D 8.D 9.C 10.C11.A 12.C 13.D 14.A 15.B 16.B 17.D 18.A 19.C 20.B21.E 22.D 23.A 24.C 25.B 26.D 27.D 28.A 29.B 30.B31.A 32.B 33.D 34.B 35.C 36.D 37.A 38.E 39.B 40.C41.D 42.D 43.C 44.B 45.B 46.D 47.B 48.B 49.A 50.B51.B 52.C 53.E 54.B 55.A 56.B 57.E 58.B 59.A 60.B61.D 62.C 63.B 64.B 65.B 66.B 67.B 68.C 69.A 70.C71.E 72.B 73.E

Page 106: Maths book part 2 obtains Rs. 1100 after lending out Rs.x at 5% per annum for 2 years and obtains Rs. 1800 after lending out Rs.y at 10% per annum for 2 years. Find the value of x

106

GeometryAngles:-Anangleisafigureformedbytworayswithacommoninitialpoint,sayO.ThispointiscalledthevertexTypesofAngles:-1)Arightangleisanangleof900.e.g.AngleAOB

2)Ifanangleislessthan900,itiscalledacute.3)Ifanangleisgreaterthan900butlessthan1800,itiscalledobtuse.4)Ifanangleisof1800,itiscalledastraightangle,ananglegreaterthan1800butlessthan3600iscalledareflexangle.5)Twoangleswhosesumis1800arecalledsupplementaryangles,eachoneisasupplementoftheother.6)Twoangleswhosesumis900arecalledcomplementaryangles,eachoneisacomplementoftheother.7)Twoadjacentangleswhosesumis1800aretheanglesofalinearpair.AnglesandIntersectinglines:Whentwolinesintersect,twopairsofverticallyoppositeanglesareformed.Verticallyoppositeanglesareequal.Thus,∠𝑐&∠𝑑areequal.∠a&∠bareequal.Also,sumofalltheanglesatapoint=3600i.e.

AnglesandParallelLines:Ifatransversal(cuttingline)cutstwoparallellinescorrespondinganglesareequali.e.∠a=∠e,∠d=∠f,∠b=∠h,∠c=∠g.Alternateanglesareequal

Page 107: Maths book part 2 obtains Rs. 1100 after lending out Rs.x at 5% per annum for 2 years and obtains Rs. 1800 after lending out Rs.y at 10% per annum for 2 years. Find the value of x

107

i.e.∠c=∠f,∠d=∠e.Interioranglesonthesamesideofthetransversalaresupplementary,i.e.∠c+∠e=∠d+∠f=180°

Triangles:-Atriangleisapolygonwiththreeverticesandthreesides(edges).Ithasthreeinternalangles.Whenweaddalltheinternalanglestogether,wewilldefinitelyget180°.1.Acutetriangle:Thetrianglewhichhasallacuteangles(i.elessthan900).2.Obtusetriangle:Thetrianglewhichhasoneobtuseangle(i.egreaterthat900)3.Rightangletriangle:Thetrianglewhichhasonerightanglei.e900.4.Scalenetriangle:Ascalenetriangleisatrianglethathasnoequalsides5.Isoscelestriangle:Anisoscelestriangleisatrianglethathastwoequalsides.6.Equilateraltriangle:Thetriangleofwhichallsidesareequalisknownasequilateraltriangle&allanglesareequalto600.PropertiesofTriangles1.Sumoftheanglesofatriangleare180°(anglesofatrianglearesupplementary)

2.Theexteriorangleofatriangleisequaltothesumoftheinterioroppositeangles.

Page 108: Maths book part 2 obtains Rs. 1100 after lending out Rs.x at 5% per annum for 2 years and obtains Rs. 1800 after lending out Rs.y at 10% per annum for 2 years. Find the value of x

108

Here,∠1=∠2+∠33.Anglesoppositetotwoequalsidesofatriangleareequal&viceversa.

Here,∠1=∠24.Iftwosidesofatriangleareunequalthenthegreaterangleisoppositetogreaterside&viceversa

5.Twotrianglesarecongruentiftwoangles&includedsideofonetriangleisequaltothecorrespondingtwoangles&includedsideoftheothertriangles.

∠1=∠3&∠2=∠4,BC=EF

or∆ABC≅ ∆DEF

6.Iftwosides&includedangleofatriangleareequaltocorrespondingtwosides&includedangleofanothertrianglethenthetwotrianglesarecongruent.

Page 109: Maths book part 2 obtains Rs. 1100 after lending out Rs.x at 5% per annum for 2 years and obtains Rs. 1800 after lending out Rs.y at 10% per annum for 2 years. Find the value of x

109

AB=DE,AC=DF,∠1=∠2∆ABC≅ ∆DEF7.Iftwoangles&nonincludedsideofonetriangleareequaltocorrespondingtwoangles&nonincludedsideofanothertrianglethenthetwotrianglesarecongruent.

8.Ifthreesidesofatriangleareequaltothreesidesofanothertriangleeachtoeachthenthetrianglesarecongruent

AB=DE,AC=DF,BC=EF9.Trianglesonthesamebase&betweenthesameparallelsareequalinArea

Here,areaofΔABC=areaofΔBDCifADisparalleltoBC10.Tworighttrianglesarecongruentifthehypotenuse&onesideofonetrianglearerespectivelyequaltohypotenuse&onesideoftheothertriangle.

Page 110: Maths book part 2 obtains Rs. 1100 after lending out Rs.x at 5% per annum for 2 years and obtains Rs. 1800 after lending out Rs.y at 10% per annum for 2 years. Find the value of x

110

AB=DE,AC=DFor∆ABC≅ ∆DEF

11.Sumofanytwosidesofatriangleisgreaterthanthethird.SimilarTriangles1.Ifalineisdrawnparalleltoonesideofatriangletheothertwosidesaredividedproportionally&viceversa

ifDE||BCso¸¹

¹º= ¸»

»¼

ifso¸¹

¹º= ¸»

»¼thenDE||BC

2.Iftwotrianglesareequiangular(i.e.thecorrespondinganglesareequal)thenthetrianglesaresimilar&hencetheirsidesareproportional

1 4,∠ =∠ 2 5,∠ =∠ 3 6,∠ =∠

Page 111: Maths book part 2 obtains Rs. 1100 after lending out Rs.x at 5% per annum for 2 years and obtains Rs. 1800 after lending out Rs.y at 10% per annum for 2 years. Find the value of x

111

𝐴𝐵𝐷𝐸 =

𝐵𝐶𝐸𝐹 =

𝐴𝐶𝐷𝐹

3.Ifthecorrespondingsidesoftwotrianglesareproportionalthenthetrianglesareequiangular/similar.4.Ratioofareasoftwosimilartrianglesisequaltotheratioofthesquaresoftwocorrespondingsides.5.Inarighttrianglethesquareofthehypotenuseisequaltothesumofthesquaresontheothertwosides

6.Inthegivenfigure(obtuseangledtriangle)

𝐴𝐶<=𝐴𝐵< +𝐵𝐶<+2AB.BDif∠B>90°7.Inthefigure(Acuteanglestriangle)

𝐴𝐶<=𝐴𝐵<+𝐵𝐷<-2BC.BDif∠B<90°

Page 112: Maths book part 2 obtains Rs. 1100 after lending out Rs.x at 5% per annum for 2 years and obtains Rs. 1800 after lending out Rs.y at 10% per annum for 2 years. Find the value of x

112

Areaoftriangle1)Whenlengthsofthesidesaregiven:-

Area= 𝑠 𝑠 − 𝑎 𝑠 − 𝑏 (𝑠 − 𝑐)where,semiperimeter(s)=yE�E�

<

2)Whenlengthsofthebaseandaltitude(height)aregiven:-

Area= 𝑏ℎ<

$ 3)Whenlengthsoftwosidesandtheincludedanglearegiven:-

Area= 𝑎𝑏 sin 𝜃<

$

Page 113: Maths book part 2 obtains Rs. 1100 after lending out Rs.x at 5% per annum for 2 years and obtains Rs. 1800 after lending out Rs.y at 10% per annum for 2 years. Find the value of x

113

4)ForEquilateralTriangle:-

Area=C>𝑎<

5)ForIsoscelesTriangle

Area=�>× 4𝑎< −𝑏<

6)WhenthreemedianaregivenTheareaofatrianglecanbeexpressedintermsofthemediansby:A=>

C 𝑆(𝑆 − 𝑀$)(𝑆 − 𝑀<)(𝑆 − 𝑀C)

Where,

S=ÉA�É;�ÉH

<

Page 114: Maths book part 2 obtains Rs. 1100 after lending out Rs.x at 5% per annum for 2 years and obtains Rs. 1800 after lending out Rs.y at 10% per annum for 2 years. Find the value of x

114

ApolloniusTheorem

IfADisthemedian,then:AB²+AC²=2(AD²+BD²)AngleBisectorTheorem

IfADistheanglebisectorforangleA,then:-¸ºº¹= ¸¼

¼¹

Inradiusandcircumradiusoftriangle#Inradius

IncaseofEqualilateraltriangle=𝒔𝒊𝒅𝒆𝟐 𝟑

Incaseofrightangletriangle=

𝒔𝒖𝒎𝒐𝒇𝒑𝒓𝒆𝒑𝒆𝒏𝒅𝒊𝒄𝒖𝒍𝒂𝒓𝒔𝒊𝒅𝒆𝒔W𝒉𝒚𝒑𝒐𝒕𝒆𝒏𝒖𝒔𝒆𝟐

Incaseofothertriangles= 𝟐×𝑨𝒓𝒆𝒂

𝒔𝒖𝒎𝒐𝒇𝒂𝒍𝒍𝒔𝒊𝒅𝒆𝒔

Page 115: Maths book part 2 obtains Rs. 1100 after lending out Rs.x at 5% per annum for 2 years and obtains Rs. 1800 after lending out Rs.y at 10% per annum for 2 years. Find the value of x

115

#Circumradius

Incaseofequilateraltriangle=𝒔𝒊𝒅𝒆𝟑

Incaseofrightangletriangle=𝒉𝒚𝒑𝒐𝒕𝒆𝒏𝒖𝒔𝒆

𝟐

Incaseofothertriangles=

𝒑𝒓𝒐𝒅𝒖𝒄𝒕𝒐𝒇𝒔𝒊𝒅𝒆𝒔𝟒×𝒂𝒓𝒆𝒂

Page 116: Maths book part 2 obtains Rs. 1100 after lending out Rs.x at 5% per annum for 2 years and obtains Rs. 1800 after lending out Rs.y at 10% per annum for 2 years. Find the value of x

116

Circles:-

Circleillustrationwithcircumference(C),diameter(D),radius(R),andcentreororigin(O)

• Arc:anyconnectedpartofthecircle.• Centre:thepointequidistantfromthepointsonthecircle.• Chord:alinesegmentwhoseendpointslieonthecircle.• Circularsector:aregionboundedbytworadiiandanarclyingbetweentheradii.• Circularsegment:aregion,notcontainingthecentre,boundedbyachordandanarc

lyingbetweenthechord'sendpoints.• Circumference:thelengthofonecircuitalongthecircle.• Diameter:alinesegmentwhoseendpointslieonthecircleandwhichpassesthrough

thecentre;orthelengthofsuchalinesegment,whichisthelargestdistancebetweenanytwopointsonthecircle.Itisaspecialcaseofachord,namelythelongestchord,anditistwicetheradius.

• Radius:alinesegmentjoiningthecentreofthecircletoanypointonthecircleitself;orthelengthofsuchasegment,whichishalfadiameter.

• Secant:anextendedchord,acoplanarstraightlinecuttingthecircleattwopoints.• Semicircle:aregionboundedbyadiameterandanarclyingbetweenthediameter's

endpoints.Itisaspecialcaseofacircularsegment,namelythelargestone.• Tangent:astraightlinethattouchestheboundaryofcircleatasinglepoint.

Page 117: Maths book part 2 obtains Rs. 1100 after lending out Rs.x at 5% per annum for 2 years and obtains Rs. 1800 after lending out Rs.y at 10% per annum for 2 years. Find the value of x

117

Importantformulas

Circumference=2𝜋𝑟

Area=𝜋𝑟²

Areaofsemicircle=ÚT²<

Circumferenceofsemicircle=𝜋𝑟 + 2𝑟

Lengthofarc(𝒍)=2𝜋𝑟 ÛC`%°

Areaofsector=𝜋𝑟< ÛC`%°

Propertiesofcircle

1.Theperpendicularfromthecentreofacircletoachordbisectsthechord.

Oiscentre,ABischord&OPisperpendiculartoAB,AP=PB2.Perpendicularbisectorsoftwochordsofacirclepassesthroughitscentre(i.e.intersectatcentre).3.Iftwochordsofacircledrawnfromthesamepointareequalthenthelinebisectingtheanglebetweenthempassesthroughcentre(oristhediameter)

Page 118: Maths book part 2 obtains Rs. 1100 after lending out Rs.x at 5% per annum for 2 years and obtains Rs. 1800 after lending out Rs.y at 10% per annum for 2 years. Find the value of x

118

AB=BC,∠1=∠2thenBPisdiameter,centreliesonBP4.Equalchordsofacircleareequidistantfromthecentre&viceversa.

AB=CDorOP=OQ(Oisthecentreofthecircle)5.Anglesinthesamesegmentofacircleareequal

∠1=∠2(beinginthesamesegment)6.Angleinasemicircleisarightangle

ABisthediameter∠1=∠2=9007.Anglewhichanarcsubtendsatthecentreisdoubletheanglesubtendedbythesamearcatanyotherpartofthecircumference.

Page 119: Maths book part 2 obtains Rs. 1100 after lending out Rs.x at 5% per annum for 2 years and obtains Rs. 1800 after lending out Rs.y at 10% per annum for 2 years. Find the value of x

119

ArcABsubtends∠1atcentre&∠2&∠3atcircumference 1 2 2 2 3⇒∠ = ∠ = ∠ 8.Fromtheabovewecometoknowthat∠2=∠3henceanglesinthesamesegmentofacircleareequal.9.Equalchordsofacirclesubtendequalanglesatthecentre

ChordAB=ChordCD(Oisthecentreofthecircle)10.Oppositeanglesofacyclicquadrilateralaresupplementary

ABCDiscyclicquadrilateralthen∠1+∠2=∠3+∠4=180011.Atangenttoacircleisperpendiculartotheradiusthroughthepointofcontact

ABistangenttothecircleatPwithcentreO

Page 120: Maths book part 2 obtains Rs. 1100 after lending out Rs.x at 5% per annum for 2 years and obtains Rs. 1800 after lending out Rs.y at 10% per annum for 2 years. Find the value of x

120

12.Thelengthoftwotangentsfromanexternalpointareequal

OA=OB13.Iftwochordsofacircleintersectinsideoroutsidethecirclewhenproduced,therectangleformedbythetwosegmentsofonechordisequalinareatotherectangleformedbythetwosegmentsoftheother.

CirclewithcentreOchordsAB&CDintersectatP(fig.1inside&fig.2outside)

ð PA.PB=PC.PD&ifPTistangentthenPT2=PA.PB14.Ifachordisdrawnthroughthepointofcontactofatangenttoacircle,thentheangleswhichthischordmakeswiththegiventangentareequalrespectivelytotheanglesformedinthecorrespondingalternatesegments.

PQistangenttocirclethroughpt.A&ABisachord&∠1=∠2&∠3=∠4

Page 121: Maths book part 2 obtains Rs. 1100 after lending out Rs.x at 5% per annum for 2 years and obtains Rs. 1800 after lending out Rs.y at 10% per annum for 2 years. Find the value of x

121

CommonTangentToAPairOfCircles

Commontangentsarelinesorsegmentsthataretangenttomorethanonecircleatthesametime.Thepossibilityofcommontangentsiscloselylinkedtothemutualpositionofcircles.1.Iftwocirclestouchinside,thetwointernaltangentsvanishandthetwoexternalonesbecomeasingletangent.

2.Iftwocirclesintersect,thecommontangentisreplacedbyacommonsecant,whencethereareonlytwoexternaltangents.

3.Iftwocirclestoucheachotheroutside,thetwointernaltangentscoincideinacommontangent,thustherearethreecommontangents.

4.Iftwocirclesareseparate,therearefourcommontangents,twoinsideandtwooutside.

Lengthofcommontangent.Ifr1,r2aretheradiioftwocirclesanddisdistancebetweentheircenters,then

(i)thelengthofadirectcommontangent= 𝑑< − (𝑟$< − 𝑟<<)(ii)thelengthofatransversecommontangent= 𝑑< − (𝑟$< + 𝑟<<)Lengthofthetangentoftwocircleswhichtoucheachotheratanexternalpointwithradiusr1&r2,thenthelengthofdirectcommontangent= 𝟒𝒓𝟏𝒓𝟐

Page 122: Maths book part 2 obtains Rs. 1100 after lending out Rs.x at 5% per annum for 2 years and obtains Rs. 1800 after lending out Rs.y at 10% per annum for 2 years. Find the value of x

122

Quadrilaterals:-

Quadrilateral Shape Properties ImportantResultsSquare

1.Allsidesareequal2.allanglesare90°3. Diagonals are equaland Bisect each other at90°

Area=a²=$<×D²

(D=diagonal)Perimeter=4aDiagonal= 2𝑎

Rectangle

1.Oppositesidesareequalandparallel2.Allanglesare90°3.Diagonalsareequalandbisecteachother

Area=𝑙×𝑏Perimeter=2𝑙 + 2𝑏Diagonal= 𝑙< + 𝑏<

Rhombus

1.Allsidesareequalandoppositesidesareparallel2.Oppositeanglesareequal3.Diagonalsarenotequal4.Diagonalsbisecteachotherat90°

Area=$<×𝑑$𝑑<

Perimeter=sumofallsides

side²=2 2

1 2d d2 2

⎛ ⎞ ⎛ ⎞+⎜ ⎟ ⎜ ⎟⎝ ⎠ ⎝ ⎠

Parallelogram

1.oppositesidesareparallelandequal2.Oppositeanglesareequal3.Diagonalsofparallelogrambisecteachother

Area=base×heightOrb×hPerimeter=sumofallsides

Trapezium

1.onlyonepairofoppositesidesareparallel2.Thediagonalscutthequadrilateralintofourtrianglesofwhichoneoppositepairaresimilar3.Thediagonalscuteachotherinmutuallythesameratio(thisratioisthesameasthatbetweenthelengthsoftheparallelsides).

Area=𝟏𝟐(sumofparallelsides)×

(height)

Iflengthsofonediagonalandtwooffsetsaregivenofanyquadrilateral

Page 123: Maths book part 2 obtains Rs. 1100 after lending out Rs.x at 5% per annum for 2 years and obtains Rs. 1800 after lending out Rs.y at 10% per annum for 2 years. Find the value of x

123

Area=$<𝑑(ℎ$ +ℎ<)

Iflengthsoftwodiagonalsandtheincludedanglearegivenofanyquadrilateral

Area=$<𝑑$𝑑< sin 𝜃

Solids:-

Page 124: Maths book part 2 obtains Rs. 1100 after lending out Rs.x at 5% per annum for 2 years and obtains Rs. 1800 after lending out Rs.y at 10% per annum for 2 years. Find the value of x

124

Figure Shape Volume CSA/LSA Totalsurfacearea

Cube 𝑎C 4𝑎< 6𝑎²

Cuboids

𝑙𝑏ℎ 2 𝑙ℎ + 𝑏ℎ 2(𝑙ℎ + 𝑏ℎ + 𝑙𝑏)

Cylinder

𝜋𝑟²ℎ 2𝜋𝑟ℎ 2𝜋𝑟 ℎ + 𝑟

Cone 13𝜋𝑟

<ℎ 𝜋𝑟𝑙𝑙 = ℎ< + 𝑟<

𝜋𝑟(𝑙 + 𝑟)

Sphere 43𝜋𝑟

C 4πr< 4πr<

Hemisphere

23𝜋𝑟

C 2𝜋𝑟< 3𝜋𝑟<

Page 125: Maths book part 2 obtains Rs. 1100 after lending out Rs.x at 5% per annum for 2 years and obtains Rs. 1800 after lending out Rs.y at 10% per annum for 2 years. Find the value of x

125

Rightprism

𝑎𝑟𝑒𝑎𝑜𝑓𝑏𝑎𝑠𝑒×ℎ𝑒𝑖𝑔ℎ𝑡

𝑝𝑒𝑟𝑖𝑚𝑒𝑡𝑟𝑒𝑜𝑓𝑏𝑎𝑠𝑒×ℎ𝑒𝑖𝑔ℎ𝑡

𝐿𝑆𝐴 + 2×𝑎𝑟𝑒𝑎𝑜𝑓𝑏𝑎𝑠𝑒

Rightpyramid

13𝑎𝑟𝑒𝑎𝑜𝑓𝑏𝑎𝑠𝑒

×ℎ𝑒𝑖𝑔ℎ𝑡

12𝑝𝑒𝑟𝑖𝑚𝑒𝑡𝑟𝑒𝑜𝑓𝑏𝑎𝑠𝑒

×𝑠𝑙𝑎𝑛𝑡ℎ𝑒𝑖𝑔ℎ𝑡

𝐿𝑆𝐴 + 𝑎𝑟𝑒𝑎𝑜𝑓𝑏𝑎𝑠𝑒

Page 126: Maths book part 2 obtains Rs. 1100 after lending out Rs.x at 5% per annum for 2 years and obtains Rs. 1800 after lending out Rs.y at 10% per annum for 2 years. Find the value of x

126

Polygon:-Aclosedplanefiguremadeupofseverallinesegmentsthatarejoinedtogether.Thesidesdonotcrosseachother.Exactlytwosidesmeetateveryvertex.

Side-oneofthelinesegmentsthatmakeupthepolygon.Vertex-pointwheretwosidesmeet.Twoormoreofthesepointsarecalledvertices.Diagonal-alineconnectingtwoverticesthatisn'taside.InteriorAngle-Angleformedbytwoadjacentsidesinsidethepolygon.ExteriorAngle-Angleformedbytwoadjacentsidesoutsidethepolygon.Sumofinteriorangleandexteriorangleofanypolygonisequalto180°TypesofPolygonsRegular-allanglesareequalandallsidesarethesamelength.Regularpolygonsarebothequiangularandequilateral.Equiangular-allanglesareequal.Equilateral-allsidesarethesamelength.

Convex-astraightlinedrawnthroughaconvexpolygoncrossesatmosttwosides.Everyinteriorangleislessthan180°.

Concave-youcandrawatleastonestraightlinethroughaconcavepolygonthatcrossesmorethantwosides.Atleastoneinteriorangleismorethan180°.

Page 127: Maths book part 2 obtains Rs. 1100 after lending out Rs.x at 5% per annum for 2 years and obtains Rs. 1800 after lending out Rs.y at 10% per annum for 2 years. Find the value of x

127

Numberofsides Name Numberofsides NameN N–Gon 6 Hexagon3 Triangle 7 Heptagon4 Quadrilateral 8 Octagon5 Pentagon 10 Decagon

Exteriorangleofanypolygon=C`%9

Where𝑛 =Numberofsides

Interiorangleofanypolygon= 180 − C`%9

𝑤ℎ𝑒𝑟𝑒𝑛 =NumberofsidesNumberofdiagonals𝑛(𝑛 − 3)

2

𝑛 =NumberofsidesNumberoftrianglesinanypolygon𝑛 𝑛 − 4 𝑛 − 5

6

𝑛 =Numberofsides

Page 128: Maths book part 2 obtains Rs. 1100 after lending out Rs.x at 5% per annum for 2 years and obtains Rs. 1800 after lending out Rs.y at 10% per annum for 2 years. Find the value of x

128

Example1:-In the given figure, if AB || CD, find the measure of angle BGF.

Solution:-ItisgivenAB||CD70°=30°+∠ECD∠ECD=40°∠ECD+∠CEF=140°+40°=180°So,EF||CD∠EFG=∠FGBExample2:-In Δ ABC, ∠A = 40o. D is a point on AC such that BD = BC. If ∠ABD = 30o, find ∠ABC. Solution:-

∠BCD=∠BDC=θ(ΔDBCisisosceles)110°=𝜃 + ∅(exteriorangle)2𝜃 + ∅=180°(ΔCDB>Anglesumproperty)or𝜃 + ∅ + 𝜃=180°orθ=180°–110°=70°∅=110°–70°=40°∠ABC=30°+∅=30°+40°=70°.Example3:-Inthefollowingfigure,OAbisects∠A,∠ABO=∠OCAand∠BOC=100o.Findthemeasureof∠AOB.

Page 129: Maths book part 2 obtains Rs. 1100 after lending out Rs.x at 5% per annum for 2 years and obtains Rs. 1800 after lending out Rs.y at 10% per annum for 2 years. Find the value of x

129

Solution:-

IntriangleAOBandAOC,twoanglesarecorrespondinglyequal.⇒∠AOB=∠AOC=zNow,∠AOB+∠AOC+∠BOC=360°z+z+100o=360o

orz=130o

Example4:-A1,A2,A3,…,A15areequallyspacedpointsontheboundaryofacircle.WhatisthesizeofangleA1A3A7?

Solution:-SincethepointsA1,A2,A3,…,A15areevenlyspaced,theygenerateequalanglesatO,eachofmeasureC`%°

$?=24°

Now,joinA1,A3andA7toO,thecentreofthecircle,asshown.Thus,

Page 130: Maths book part 2 obtains Rs. 1100 after lending out Rs.x at 5% per annum for 2 years and obtains Rs. 1800 after lending out Rs.y at 10% per annum for 2 years. Find the value of x

130

thesizeofangleA1A3A7=(7-1)x24o=144oExample5:-Inthegivenfigure,∠A=36o,∠C=30o,∠D=45oand∠CED=xo.Findx.

Solution:-ΔABC;∠CBA=180°–(30°+36°)=180°–66°=114°∠EBD=180°–∠EBA=180°–114°=66°x°=66°+45°=111°(exteriorangle)Example6:-SupposethatAB=AC=CDandAD=BD.Whichofthefollowingoptionsisthemeasureof∠ABCindegrees?

Solution:-IntriangleABC,AB=AC,∠ABC=∠ACB=xIntriangleACD,AC=CD,∠ADC=∠CAD=yIntriangleABD,AD=BD,∠A=∠B=xSo,∠BAC=x–yweget3x-y=180°andfromtriangleACDweget,2x+y=180°solving,∠ABC=x=72°

Page 131: Maths book part 2 obtains Rs. 1100 after lending out Rs.x at 5% per annum for 2 years and obtains Rs. 1800 after lending out Rs.y at 10% per annum for 2 years. Find the value of x

131

Example7:-Inthegivenfigure,ifAB=NBandAC=CM,findthevaluesofx,yandz.

Solution:-

∠BNA=∠BAN=z40°=2zorz=20°y=180°–(50°+40°)=180°–90°=90°∠CMA=∠CAM=x50°=2xX=25°Example8:-Inthetrianglebelow,anglePORis60o.AssumethatthelinesegmentsOQandQRhavethesamelength,andlinesegmentsOPandPQhavethesamelength.FindtheangleQRO.

Solution:-

Page 132: Maths book part 2 obtains Rs. 1100 after lending out Rs.x at 5% per annum for 2 years and obtains Rs. 1800 after lending out Rs.y at 10% per annum for 2 years. Find the value of x

132

InΔOPQ,OP=PQ∠PQO=∠POQ=ySimilarlyinΔOQR,OQ=QR∠QRO=∠QOR=xy=2x(sumofinterioranglesisequaltoexteriorangle)Also,y+x=60o(Given∠POR=60o)3x=60oorx=20oExample9:-InΔABC,DE||BCanditdividesthetriangleintotwoequalparts.TheratioofareaΔADEtoareaΔABCis

Solution:-Area∆ADE=Areaquad.∆DBEC⇒Area∆ABC=2Area∆ADE

Area∆ADE/Area∆ABC=2Example10:-IntriangleAEF,CDisparalleltoEF,AD=DF,CD=4andDF=3units.WhatisthelengthofEF?

Solution:-InΔADCandΔAFE∠A=∠A{Common}AD=DF{given}∠ADC=∠AFE{correspondinganglesbetweentwo||lines}HenceΔABCissimilartoΔAFE.

Now,¸¹¸à=¼¹

ȈandweknowAF=AD+DF=3+3=6units

Page 133: Maths book part 2 obtains Rs. 1100 after lending out Rs.x at 5% per annum for 2 years and obtains Rs. 1800 after lending out Rs.y at 10% per annum for 2 years. Find the value of x

133

EF=`×>C

=8units Example11:- Inthegivenfigure,DF||AG,DE||AB,AB=15,CD=8,AD=a,DE=10,FG=bandCG=6.Theratioa:bisequalto

Solution:-

In ΔABC and ΔDEC

[Ey$?

= [$%

(By similar triangles).

⇒ a = 4 In ΔACG and ΔDCF [Ey[

= ``W�

(By similar triangle)

⇒ b = 2 ∴ a : b = 4 : 2 = 2 : 1 Example 12:- In the given figure, EF || CD, DE || BC, AF = 2FD and DE = 4 units. What is the measure of BC?

Solution:-ΔAFE~ΔADCAF/AD=AE/AC

Page 134: Maths book part 2 obtains Rs. 1100 after lending out Rs.x at 5% per annum for 2 years and obtains Rs. 1800 after lending out Rs.y at 10% per annum for 2 years. Find the value of x

134

⇒AF/(AF+FD)=AE/AC⇒2FD/3FD=AE/AC⇒2/3=AE/ACΔADE~ΔABC⇒DE/BC=AE/AC=2/3⇒BC=C

<×DE=C

<×4=6units

Example13:-GivenΔABCwithrightangleCandasecondrighttriangleABDsuchthatbothtrianglessharethesamehypotenuse.IfBC=1,AC=bandAD=2,thenBD=

Solution:-

InΔABCsincetherightangleisC,thehypotenuseisAB.SinceBC=1andAC=b,wegetthehypotenuseABas 𝑏< + 1 .SincethisisthehypotenuseoftheΔABDaswell,andsinceAD=2,wegetBD 𝑏< + 1 − 2<= 𝑏< − 3.

Example14:-MediansQXandRYofΔPQRareperpendicular,QX=8andRY=12.TheareaofΔPQRis

Solution:-ThemediansmeetatG.thenRG=(2/3)RY=8andtheareaof∆QRXis(1/2)xQXxRG=(1/2)x8x8=32.SinceQXisamedian,trianglesPQXandXQRhavethesamearea.HencetheareaofΔPQR=64.Example15:-Inthegivenfigure,therearethreecirclesofradius2cmtouchingeachother.ThetriangleABCisanequilateraltriangleofside4cm.Thelengthofarcs(MP+PN+NM)is

Page 135: Maths book part 2 obtains Rs. 1100 after lending out Rs.x at 5% per annum for 2 years and obtains Rs. 1800 after lending out Rs.y at 10% per annum for 2 years. Find the value of x

135

Solution:-Asthecirclesarecongruent(havingsameradius) AndangleA=angleB=angleC=60°each Arc(MP)=arc(PN)=arc(MN)= Û

C`%×2𝜋𝑟

Lengthsofarcs(MP+PN+NM)=3× `%C`%

×2𝜋𝑟=2πcmExample16:-Inthegivendiagram,ACBisanarcofacircleandCDistheperpendicularbisectorofthechordAB.IfAD=3andCD=9,theareaoftheentirecircleis

Solution:-Here,ACBisanisoscelesΔwithAC=CB= 90inscribedinacircle.

ADCisarightangle= 𝐴𝐷< +𝐷𝐶<=AC3< +9<=AC=BC

Now,inanyΔ,R=y��

>á(R=circumradius)

Here,a=b=AC=ABR= y;�

>×â= >y;W�; _%

>×_%W`;=90/18=5

R=5unitsandArea=π(5)²=25πsq.unitsExample17:-ABCisatriangleinwhich∠A=60o,AC=4cmandAB=3cm.IfBDistheperpendicularfromBtothesideAC,whatisthelengthofBD?Solution:-Area∆ABC=½×AC×AB×sinA=½×base×altitude∴½×3×4×sin60=½×AC×BD

Page 136: Maths book part 2 obtains Rs. 1100 after lending out Rs.x at 5% per annum for 2 years and obtains Rs. 1800 after lending out Rs.y at 10% per annum for 2 years. Find the value of x

136

i.e.12× C<=4×BD

⇒BD=1.5 3cm.Example18:-AC:AD::2:5andareaoftriangleAODis8 5squareunits.FindCDintermsofr,whereABistangenttothecirclewithcentreO.

Solution:-AC=2𝑥,AD=5𝑥

CD=AD–AC=3𝑥

AB2=AC×AD=2𝑥×5𝑥=10𝑥2

AB= 10𝑥AreaofΔAOD=½×AB×OB

8 5=$<× 10𝑥×r

⇒𝑥=[ <T

CD=3𝑥=3×[ <T=<> <

T

Example19:-In the given figure, O is the center of the circle and AE is a diameter. If AB = BC and ∠BFC = 25°, find the value of ∠ABC.

Solution:-anglessubtendedbyequalchordsareequal∠AFB=25°=∠BFCincyclicquadrilateralABCF∠AFC=25°+25°=50°

Page 137: Maths book part 2 obtains Rs. 1100 after lending out Rs.x at 5% per annum for 2 years and obtains Rs. 1800 after lending out Rs.y at 10% per annum for 2 years. Find the value of x

137

∠ABC=180°-50°=130°(becausesumofoppositeanglesofacyclicquadrilateralis1800)

Example20:-Atangenttoacircleofradius3cmfromanexternalpointPisoflength4cm.FindtheshortestdistanceofPfromapointonthecircumferenceofthecircle.Solution:-

In triangle OAP, OP2 = 32 + 42 or OP = 5 cm Also OP = OM + MP So, MP = 5 – 3 = 2 cm. Example21:-PMandPNaretangentstothecirclewithcentreQandradius7cm.ThelengthofPMis7cm.WhatisthelengthofPQ?

Solution:-PQ2=PM2+MQ2(ΔPQMisrightangledtriangle)=72+72=49+49orPQ=7√2cmExample22:-IfAB=30,PM=8andDC=16andPisthecentreofcirclethenfindthelengthofPN?

Solution:-AnyperpendicularfromcentretochordbisectthecordSo,AM=15,PM=8IntriangleAPM,AP2=PM2+AM2

So,AP=17=CP

Page 138: Maths book part 2 obtains Rs. 1100 after lending out Rs.x at 5% per annum for 2 years and obtains Rs. 1800 after lending out Rs.y at 10% per annum for 2 years. Find the value of x

138

IntriangleCPN,CP=17,CN=8So,byPythagorastheoremwewillhavePN=15Example23:-IntriangleABC,∠ABC=90o,DliesonACsuchthatBCDisanequilateraltriangle.IfBChaslength1,findthelengthofAB.Solution:-SinceΔBCDisanequilateraltriangle,themeasureof∠BCD=60oand∠ABD=30o.ABDisanisoscelestrianglewithAD=BD=1andAC=2Now,byPythagorastheorem,AB= 3Example24:-InatriangleABC,AB=6,BC=8andAC=10.AperpendicularfromBmeetsthesideACatD.AcircleisdrawnwithradiusBDandcentreB.IfthecirclecutsABandBCatPandQ,respectively,AP:QCisequaltoSolution:-

∆ABCisrightangledatBas(10,8,6)isaPythagoreantripletLetBP=BD=BQ=x(radiusofthesamecircle)AD=yForrightangledΔBDCx2+(10–y)2=8262–y2+(10–y)2=8236–y2=64–(100–20y+y2)36–y2=64–100+20y–y2y=C`

$%

BD2=36– C`$%

<

BD=4.8Hence,AP=6–4.8=1.2=`

?

QC=8–4.8=3.2AP:QC=1.2:3.2=3:8Example25:-InacyclicquadrilateralABCD,ABisparalleltoCDandCD=2ABthenAD:BCisequalto

Page 139: Maths book part 2 obtains Rs. 1100 after lending out Rs.x at 5% per annum for 2 years and obtains Rs. 1800 after lending out Rs.y at 10% per annum for 2 years. Find the value of x

139

Solution:-

Example26:-Inthefigure,ABisadiameterofthecircle,TDisatangent.If∠AHD=36o,∠CDTis______.

Solution:-∠ADB=90o(∠insemi-circle)∠DAH=180o–90o–36o=54o∠DAH=∠CDK=54o(∠inalt.segment)⇒∠CDT=180o-54o(adj.∠sonstraightline)=126oExample27:-Inthegivenfigure,PQ=RQ.∠RQP=72o,PCandQCaretangentstothecirclewithcentreO.Calculate∠PCQ.

Page 140: Maths book part 2 obtains Rs. 1100 after lending out Rs.x at 5% per annum for 2 years and obtains Rs. 1800 after lending out Rs.y at 10% per annum for 2 years. Find the value of x

140

Solution:-

Angle2=angle4=54o

Angle1=angle3=54oAnglesinalternatesegmentthereforeanglePCQ=72o Example28:-GiventwoconcentriccircleswithcentreP,ACisachordofthelargercircletangenttothesmalleratB.If|BC|=2units,findtheareainbetweentwocirclesinsquareunits.Solution:-

LetR,rbetheradiiofbigger&smallercirclesrespectively.InΔABPR2=r2+(AB)2…………(1)AsBC=2ACisachordPBisperpendiculartoit&mustbisectthechord.AB=BC=2From(1)R2–r2=4Areainbetweentwocircles=πR2–πr2=π(R2–r2)=4πExample29:-Inthegivenfigure,ABCQisaquadrantofacirclewithradius14cm.WithACasdiameter,asemicircleisdrawn.Findtheareaoftheshadedportion.

Page 141: Maths book part 2 obtains Rs. 1100 after lending out Rs.x at 5% per annum for 2 years and obtains Rs. 1800 after lending out Rs.y at 10% per annum for 2 years. Find the value of x

141

Solution:-Requiredarea=AreaAQCPA=AreaACPA–AreaACQA=Areaofsemi-circlewithACasdiameter–(AreaABCQA–AreaofΔABC)=Areaofsemi-circlewithACasdiameter–areaofaquadrantofacirclewithABasradius+AreaofΔABCNowAC= 14< + 14<=14 2cmAreaofsemi-circlewithACasdiameter=$

<𝜋(AC/2)2=$

<𝜋(7 2)2=49𝜋

(AreaABCQA–AreaofΔABC)=$>𝜋142−$

<×14×14=49𝜋−98

Requiredarea=49𝜋 − (49𝜋−98)=98cm2Example30:-Thefigureshowsthelengthsofthesidesofanequiangularpolygon.Whatistheareaofthepolygon?

Solution:-Asshowninthefigure.

Therearetwosquaresofside1,onerectanglewithdimensionsof3x1andfourrightangledisoscelestriangles.Therefore,Area=2(1x1)+(3x1)+4($

<x1x1)=7.

Example31:-PQisthediameterofacirclewithradius5units.ThelengthofRSis6units.WhatistheareaofthetrapeziumPQRS?

Solution:-

Page 142: Maths book part 2 obtains Rs. 1100 after lending out Rs.x at 5% per annum for 2 years and obtains Rs. 1800 after lending out Rs.y at 10% per annum for 2 years. Find the value of x

142

RT=3,QT=4.Area=$

<(6+10)×4=32

Example32:-Asolidconsistsofacircularcylinderfittedwitharightcircularconeplacedonthetop.Theheightoftheconeis9cm.Iftotalvolumeofthesolidis3timesthevolumeofcone,findtheheightofcircularcylinder.Solution:-

LetrbetheradiusofconeandcylinderHeightofcone=9cmVolumeofcone=$

Cπr2h=$

Cπr2×9=$

<volumeofcylinder=$

<πr2H=

Soheighth=6cmExample33:-Acylindricalcontainerwhosediameteris12cmandheightis15cm,isfilledwithice-cream.Thewholeice-creamisdistributedto10childreninequalconeswithhemisphericaltops.Iftheheightoftheconicalportionistwicethediameterofitsbase,findthediameteroftheice-creamcone.Solution:-Volumeofcylindricalcontainer=10×volumeofeachcone

Page 143: Maths book part 2 obtains Rs. 1100 after lending out Rs.x at 5% per annum for 2 years and obtains Rs. 1800 after lending out Rs.y at 10% per annum for 2 years. Find the value of x

143

⇒πr2h=10× $C𝜋𝑟$<ℎ$ +

<C𝜋𝑟$C

⇒π×62×15=$%ÚC[r12h1×2r13]

=$%Ú

C[4r13+2r13][ h1=2d1=2.2r1=4r1]

⇒`

;×$?×C$%

=6r13⇒r1=3Diameterofice-creamcone=6cmExample34:-ThecostofwhitewashingwallsofaroomisRs.56.Howmuchwillitcosttowhitewashanotherroomwhichistwiceinlength.Breadthandheightofthepreviousroom?Solution:Areaofwalls=2h(l+b)Areaofwallsforthebiggerroom=2(2h)(2l+2b)=4×2h(l+b)Thecostwillalsobe4timesthecostofsmallerroom.Totalcost=4×56=Rs.224.Example35:-Ametallicrightcircularconeofheight9cmandbaseradius7cmismeltedintoacuboidwithtwosidesas11cmand6cm.Whatisthethirdsideofthecuboid?Solution:-Letthethirdsideofcuboid=xcmNow,accordingtothequestion:1/3πr2h=l×b×h⇒66x=462⇒x=7cmExample36:-Asolidisintheformofacylinderwithhemisphericalends.Thetotalheightofthesolidis19cmandthediameterofthecylinderis7cm.Thesurfaceareaofthesolidis?Solution:-Radiusofthehemisphere=Radiusofthecylinder=HeightofthehemisphereHeightofcylinder=Totalheight-2x(radiusofthehemisphere).Heightofcylinder=12cmradiusofHemisphere=3.5cmSurfacearea=Surfaceareaofcylinder+2x(Surfaceareaofhemisphere)=(2πrh+2πr2)+2x(2πr3)=(2xπx3.5+2xπx3.5x3.5)+2x(2xπx3.5x3.5x3.5)=418Example37:-ABCDisarectanglewithAD=1.DPFandCQFaretwoequalarcsdrawnwithAandBascentersrespectively.EisthemidpointofCD.AnotherarcwithEascentretouchesthetwoarcsDPFandCQFatPandQrespectively.Whatistheareaoftheshadedportion?

Page 144: Maths book part 2 obtains Rs. 1100 after lending out Rs.x at 5% per annum for 2 years and obtains Rs. 1800 after lending out Rs.y at 10% per annum for 2 years. Find the value of x

144

Solution:-AsAFandBFarebothradiiofequalarcs,AF=BF=AD=1.

AB=2Areaoftherectangle=2.

NotethatBEisthediagonalofsquareFBCE,andtherefore,BE= 2EQ=BE–BQ= 2-1( BQ=BC=1)AreaofsemicirclewithcentreE=$

<π( 2-1)2.

Also,thetotalareaofthetwosegmentswithcentresAandB=2×$>×π×12=Ú

<

Areaofshadedregion=2-Ú

<[1+( 2-1)2]=2-Ú

<[2-2 2+2]=2-π(2- 2)

Example38:-Fiveequalsquaresareplacedsidebysidetomakeasinglerectanglewhoseperimeteris372cm.Whatistheareaofeachsquare?Solution:-Letxbethelengthofthesideofeachsquare.Arectangleformedbyplacingfivesquaressidebysidehasthelengthequalto5xandbreadthequaltox.Hence,theperimeteroftherectangleis12x.Therefore,12x=372cm.i.e.x=31cm.Hence,areaofeachofthesquare=x2=312=961cm2Example39:-Ifallthesidesofatriangleareincreasedby200%,whatwillbethepercentageincreaseinarea?Solution:-Newsides=3xoldsidesofΔ

Newarea=9xoldareaHence,theincrease=800%Example40:-Twocirclesofradius8cmeach,cutorthogonallyasshowninthefigure.Findthelengthoftheoutlineofthefigure(i.e.ABCDA).

Page 145: Maths book part 2 obtains Rs. 1100 after lending out Rs.x at 5% per annum for 2 years and obtains Rs. 1800 after lending out Rs.y at 10% per annum for 2 years. Find the value of x

145

Solution:-∠PCQ=90o∠CPQ=∠CQP=45o∠CPA=∠CQA=90oItmeans,¼thofperimeterofboththecirclesisoverlapping.orArcABCDA=2π(8)[1–¼]×2=24π.

Example41:-Arectangularsheetofpaperis30cm×20cm.ItisfoldedinsuchawaythatcornerBfallsonACatB'andPQ,alongwhichitisfoldedisparalleltoACasshowninthefigure.FindtheareaofAPQCDA.

Solution:-

NotethatPBQB'isakite.Since,BP=PB'andBQ=QB',BO=OB'and∠BOP=∠B'OP=90°AndBO=OB'andPQ||ACAP=PB=X=½AB=10andBQ=QC=Y=½BC=15AreaoftrianglePBQ=½×10×15=75cm2=>AreaofAPQCDA=600–75=525cmExample42:-Thefigureshowsthelengthsofarectangleininches.Howmuchistheshadedareainsquarein

Page 146: Maths book part 2 obtains Rs. 1100 after lending out Rs.x at 5% per annum for 2 years and obtains Rs. 1800 after lending out Rs.y at 10% per annum for 2 years. Find the value of x

146

Solution:-Theshadedregionistheareaofthe8by12rectangle,minus[theareaoftherighttrianglewithbase8andheight9]plus[theareaoftherighttrianglewithbase2andheight3]=3.SotheshadedareaA=8(12)–$

<(8)(9)–$

<(2)(3)=96–36–3=57.

Example43:-Onesideofarhombusis10cmandoneofitsdiagonalsis12cm.TheareaoftherhombusisSolution:-

AB2=AE2+EB2orEB2=AB2–AE2=100–36=64EB=8Areaofrhombus=2×AreaofΔABC

Example44:-Thediameterofawheelis63cm.Thedistancecoveredbythewheelin100revolutionsisSolution:-Circumference=2×<<

X× `C

<=198cm.

Thedistancetraveledinonerevolution=198cm.Hence,thedistancetraveledbythewheelin100revolutions=198×100=198mExample45:-Asphericalballofradius4cmistobedividedintoeightequalpartsbycuttingitalongtheaxisasshown.Findthesurfaceareaofeachpieceincm2.

Page 147: Maths book part 2 obtains Rs. 1100 after lending out Rs.x at 5% per annum for 2 years and obtains Rs. 1800 after lending out Rs.y at 10% per annum for 2 years. Find the value of x

147

Solution:-Theballafterbeingcutwillhaveeightparts,eachofthesamevolumeandsurfacearea.Thefigurewillbesomewhatlikethefigure(1)ifseenfromthetopbeforecutting.Aftercuttingitlookssomethinglikethefigure(2).

Now,Thesurfaceareaofeachpiece=Area(ACBD)+2(AreaofCODBC).Thedarkenedsurfaceisnothing,butarcABfromsideglance,whichmeansitssurfaceareaisoneeighththeareaofthesphere,thatis,$

[×4πr2=$

<πr2.Now,CODBCcanbeseenasasemicirclewithradius4

cm.Therefore,2(AreaCODB)=2×$<×πr2=πr2

⇒surfaceareaofeachpiece=$<×πr2+πr2= C

<×π×42=24π

Example46:-Fourcircularcoins,eachofradius1.4cmareplacedflatonatable;suchthattheircentersarethecornersofasquareandthateachcointouchestwooftheothers.Findthearealyingvacantbetweentheirrims.Solution:-

Arealyingbetweencoins=Areaofsquare–4×areaof1sector

=(2.8cm)2–4×_%°C`%

×𝜋× 1.4 <=(1.4)2[4–π]=(1.4)2×(4–3.14)=1.68cm2

Example47:-AparallelogramPTRUisdrawnwithinanotherparallelogramPQRS,suchthatTRbisects∠SRQand∠RTQ=500.Findthe∠TQR

Page 148: Maths book part 2 obtains Rs. 1100 after lending out Rs.x at 5% per annum for 2 years and obtains Rs. 1800 after lending out Rs.y at 10% per annum for 2 years. Find the value of x

148

Solution:-PU||RT,∠UPT=∠RTQ=50o(Correspondingangles)∠TRU=50o(Oppositeangleofparallelogram)RTbisectsangleURQ.∠TRQ=50o∠TQR=80o(SRQPisparallelogram)Example48:-Ifthesumofanglesofapolygonis1080,thenthenumberofsidesofthepolygonwillbe_________?Solution:-1080=180(n–2)ORn=8Example49:-Inthefiguregiven,ifx=120oandy=100o,thenz=?

Solution:-x+y+a=360°.120°+100°+a=360°.a=140°.z+a=180°.z+140°=180°.z=40°.Example50:-Ahallis15mlongand12mbroad.Ifthesumoftheareasofthefloorandtheceilingisequaltothesumoftheareasoffourwalls,thevolumeofthehallis:Solution:-2(15+12)xh=2(15x12)ORh=180/27m=20/3mOrvolume=15x12x20/3=1200m3Example51:-Ahollowironpipeis21cmlonganditsexternaldiameteris8cm.Ifthethicknessofthepipeis1cmandironweighs8g/cm3,thentheweightofthepipeis:Solution:-Externalradius=4cm,Internalradius=3cm.Volumeoftheiron=<<

X× 4< − 3< ×21=462cm3

Page 149: Maths book part 2 obtains Rs. 1100 after lending out Rs.x at 5% per annum for 2 years and obtains Rs. 1800 after lending out Rs.y at 10% per annum for 2 years. Find the value of x

149

SoweightoftheironWeightofiron=(462x8)gm=3696gm=3.696kg.Example52:-50mentookadipinawatertank40mlongand20mbroadonareligiousday.Iftheaveragedisplacementofwaterbyamanis4m3,thentheriseinthewaterlevelinthetankwillbe:Solution:-Totalvolumeofwaterdisplaced=(4x50)m3=200m3.Orriseinwaterlevel=200/40×20=0.25mor25cmExample53:-Thecurvedsurfaceareaofacylindricalpillaris264m2anditsvolumeis924m3.Findtheratioofitsdiametertoitsheight.Solution:-ÚT;�<ÚT�

= _<><`>

sor=7m&2𝜋𝑟ℎ=264Soh=6mSoreuiredratio=2r/h=14/6=7/3Example54:-Alargecubeisformedfromthematerialobtainedbymeltingthreesmallercubesof3,4and5cmside.Whatistheratioofthetotalsurfaceareasofthesmallercubesandthelargecube? Solution:-Volumeofthelargecube=(33+43+53)=216cm3.Lettheedgeofthelargecubebea.So,a3=216soa=6cm.Sorequiredratio=6(32+42+52)/6×62=50/36=25/18Example55:-Ahorizontalpipeforcarryingfloodwaterhasdiameter1m.Whenwaterinitis10cmdeep,thewidthofwatersurfaceABisequalto

Solution:-

Page 150: Maths book part 2 obtains Rs. 1100 after lending out Rs.x at 5% per annum for 2 years and obtains Rs. 1800 after lending out Rs.y at 10% per annum for 2 years. Find the value of x

150

LetObethecentreofcircle.InΔOAD502=402+x22500–1600=x2x2=900x=30AB=2x=60cm.Example56:-Theradiusofarightcircularcylinderisincreasedby25%.Bywhatpercentshoulditsheightbechangedsothatitsvolumeremainsthesame?Solution:-LetradiusberandheightbehVolumeofcylinder=r2hAsradiusisincreasedby25%Hence,thenewradius=1.25randletheightbeH.Asvolumeremainsthesame,so𝜋r2h=𝜋(1.25)2HH/h=1/1.252=(4/5)2=0.82=0.64æW��=0.64-1=-0.36

æW��x100=-36%

So,theheightshouldbedecreasedby36%.Example57:-Inthefigure,AB||CDandPQintersectsthematPandQrespectively.

IfPRandQRarethebisectorsofTypeequationhere. APQand PQCrespectively,findout PRQSolution:-Since AB || CD

Page 151: Maths book part 2 obtains Rs. 1100 after lending out Rs.x at 5% per annum for 2 years and obtains Rs. 1800 after lending out Rs.y at 10% per annum for 2 years. Find the value of x

151

Now, ∠1 + ∠2 + ∠4 + ∠5 = 180o … (1) Also ∠2 + ∠3 + ∠4 = 180o … (2) ∠1 = ∠2 and ∠4 = ∠5 [ given that PR & QR are bisectors] Now from equation (1)

Now from equation (1) 2∠2 + 2∠4 = 180o

∠2 + ∠4 = 90o From equation (2)

90 + ∠3 = 180o

or∠3 = 90o Example58:-Giventhat AOB=88ointhefigure,findthemeasureof OCB,whereOisthecentreofthecircle.

Solution:-

Asshowninthefig,JoinBC&AB∠AOB=880(given)∠BOC=180–880=920(angleoflinearpair)

Now OBCisanisoscelestriangleOB=OC=radius∠OCB=∠OBC=x

Nowx+x+92=18002x=880 x=440

Example59:-Inthefigure,TAisatangenttothecircle.

Page 152: Maths book part 2 obtains Rs. 1100 after lending out Rs.x at 5% per annum for 2 years and obtains Rs. 1800 after lending out Rs.y at 10% per annum for 2 years. Find the value of x

152

If∠BAT=120o,findthemeasureof∠ACBand∠CAT.Solution:-As∠TAC=∠ABC(anglesinthealternatesegment)Now∠TAB=1200Or∠TAC+ CAB=1200Or∠ABC+∠CAB=1200...(from(i))Nowin∆ABC,∠ABC+∠CAB+∠BCA=1800120o+∠BCA=1800Hence∠BCA=600Now∠BCA+∠TCA=1800∠TCA=1800–600=1200Nowin∆TAC,∠TAC+∠TCA+∠ATC=1800∠TAC=1800–1200-250=350∠ABC=350Example60:-Acylindricalcisternofradius42cmispartlyfilledwithwater.Ifarectangularblock24cm×21cm×11cmiswhollyimmersedinthewater,byhowmuchwillthewaterlevelrise?Solution:-Volumeofcylindricalcolumnofincreasedheight=volumeofrectangularblock⇒<<

X×42×42×h=24×21×11

orh=1cmExample61:-Fourhorsesaretetheredat4cornersofasquarefieldofside70metressothattheyjustcannotreachoneanother.Thearealeftungrazedbythehorsesinsq.m.is:Solution:Thelengthoftheropeinwhichthehorsestiedshouldbeequaltohalfofthesideofthesquareplotsothattheyjustcannotreachoneanother.Therefore,thelengthoftheropeis35m(70/2).Theareacoveredbyeachhorseshouldbeequaltotheareaofsectorwithradiusof70/2=35m(lengthoftherope).Totalareacoveredbythefourhorses=4×areaofsectorofradius35metres

=Arealeftungrazedbythehorses=Areaofsquarefield-Areacoveredbyfourhorses=702-(22/7)*35*35=4900-3850=1050sq.m.

Page 153: Maths book part 2 obtains Rs. 1100 after lending out Rs.x at 5% per annum for 2 years and obtains Rs. 1800 after lending out Rs.y at 10% per annum for 2 years. Find the value of x

153

Example62:-Acircustenthasacylindricalbasesurmountedbyaconicalroof.Theradiusofthecylindricalpartis30manditsheightis9m.Thetotalheightofthetentis24m.Findtheamountofairavailableperperson,if15personsareseatedinthetentSolution:-Radiusofcylindricalpart=30mHeightofcylinder=9mTotalheightoftent=24mHeightofconicalpart=24–9=15mVolumeofair=volumeofcylinder+volumeofcone.

=πr2h+$

Cπr2H

=πr2 ℎ +$C𝐻

=<<X

Example63:-Ifthesumoftheinterioranglesofaregularpolygonmeasuresupto1440degrees,howmanysidesdoesthepolygonhave?Solution:WeknowthatthesumofanexteriorangleandaninteriorangleofaPolygon=180o.Wealsoknowthatsumofalltheexterioranglesofapolygon=360o.TheQuestionstatesthatthesumofallinterioranglesofthegivenpolygon=1440o.Therefore,sumofalltheinteriorandexterioranglesofthepolygon=1440+360=1800.Ifthereare'n'sidestothispolygon,thenthesumofalltheexteriorandinteriorangles=180*n=1800.Therefore,n=10.Example64:-Theheightofacylinderis14cmanditscurvedsurfaceis264sq.cm.TheradiusofitsbaseisSolution:-Curvedsurfaceareaofacylinder=2πrhgivenh=14cmandcurvedsurfacearea=264cm22πrh=264cm2

Solvingwegetr=3cmExample65:-Ateapotisintheshapeofhemisphericalbase,aconicallidandacylindricalbody.Theoverallheightoftheteapotis20cm.Thebasediameteris3cmandthecylindricalpartis8cmlong.Whatisthevolumeoftheteapot?

Page 154: Maths book part 2 obtains Rs. 1100 after lending out Rs.x at 5% per annum for 2 years and obtains Rs. 1800 after lending out Rs.y at 10% per annum for 2 years. Find the value of x

154

Solution:-

Volumeofteapot=volumeofhemisphere+volumeofcylinder+volumeofcone=<

Cπr3+πr2h+$

Cπr2h=219π/8cubic.Cm

Example66:-Acylinderandaconehavethesameheightandthesameradiusofthebase.TheratiobetweenthevolumesofthecylinderandtheconeisSolution:-RatioofthevolumeofthecylinderandtheconeisÚT

;�AHÚT

;�=3:1

Example67:-Agoatistiedtoonecornerofasquareplotofside12mbyarope7mlong.Findtheareaitcannotgraze.Solution:-

Areathegoatcan’tgrazeistheshadedareashownwhichisequaltoareaofsquareminusareaofquadrantofcirclewithradiusequaltolengthofropei.e.7m.Areaitcan’tgraze=(12)2– _%

C`%× <<

X×7×7

144–38.5=105.5sq.mExample68:-:Ifaregularhexagonisinscribedinacircleofradiusr,thenitsperimeterisSolution:

Page 155: Maths book part 2 obtains Rs. 1100 after lending out Rs.x at 5% per annum for 2 years and obtains Rs. 1800 after lending out Rs.y at 10% per annum for 2 years. Find the value of x

155

Inthecirclewitheachsideofhexagonwewillget6congruentequilateraltrianglesHenceSidesofhexagon=radiusofcircle SoSide=rOr,Perimeter=6r.Example69:-Thediagonalsofarhombusare64cmand48cm.TheheightoftherhombusisSolution:Areaofrhombus=$

<×d1×d2

Areaofrhombus=$<×64×48=1536cm2

AB= 𝑂𝐴< + 𝑂𝐵<= 32< + 24<=40cmAreaofrhombus=base×heightor1536=40×hHence,h=$?C`

>%=38.4cm.

Example70:-Theareaofatrapeziumis384cm2.Ifitsparallelsidesareintheratioof3:5andtheperpendiculardistancebetweenthemis12cm.thenthesmalleroftheparallelsidesisSolution:Lettheparallelsidesbe3xand5xcmrespectivelyThen,Area=$

<(3x+5x)12=384

Or,8x=64x=8Hencetheparallelsidesare24cmand40cm.Example71:-Thebaseandtheheightofarightangledtrianglearethesameasthoseofatrapezium.Iftheotherparallellineofthetrapeziumis1.5timesofthefirstparallelline,thenwhatwillbetheratiooftheareasofthetriangleandthetrapezium?Solution:Letxisthebaseandhistheheightofthetrapezium.Areaoftriangle=$

<xh

Areaoftrapezium=$<𝑥 + C

<𝑥 ×h=?

>𝑥ℎ

Requiredratio=$<𝑥ℎ:?

>𝑥ℎ=2:5.

Example72:-SRisadirectcommontangenttothecirclesofradiiof8cmand3cmrespectively,theircentresbeing13cmapart.ifSandRaretherespectivepointsofcontactsthenfindthelengthofSR

Solution:

UsetheformulaD=√d2-(R-r)2

D=√169-25orD=√144orD=12cmisthelengthofSR.

Page 156: Maths book part 2 obtains Rs. 1100 after lending out Rs.x at 5% per annum for 2 years and obtains Rs. 1800 after lending out Rs.y at 10% per annum for 2 years. Find the value of x

156

Exercise

1.Anairjetflies10milessouth,then4mileseast,then7milesnorthandthen8mileswestwhereitfinallylanded.Findtheshortestdistancefromthestartingpointofthejourneyandthepointwhereitfinallyends.(a)7miles (b)6miles (c)8miles (d)5miles2.Theareaofacircleis154cm²,whichisequaltotheareaofarectanglewithonesideequivalenttotheradiusofthecircle.Findtheothersideoftherectangle.(a)22/7cm (b)11cm (c)22cm (d)11/7cm3.Findthesupplementofangle75°.(a)105° (b)90°(c)15° (d)125°4.Findtheanglewhosesupplement&thriceitscomplementareintheratioof5:6.(a)60° (b)30° (c)90° (d)120°5.Findthelargeranglemadebythehandsoftheclockat8:00.(a)120° (b)180° (c)240° (d)200°6.Theperimeterofarectangleis220meters,andthedifferencebetweenlengthandbreadthis30meters.Findtheareaoftherectangle.(a)2524m² (b)3200m² (c)2400m² (d)2800m²7.Findthediagonalofasquarewhosesideisof8m.(a)8 2m (b)16m (c)8m (d)18 2m8.Asolidmetalcylinderhavingaradiusof5cmandheightof18cmismelteddownandrecastasaconehavingradiusof3cm.Findtheheightofthecone.(a)150cm (b)100cm (c)120cm (d)125cm9.Given:RadiusofcircleAis2.5unitsandradiusofcircleBistwicetheradiusofA.ColumnA ColumnBAreaofcircleA CircumferenceofcircleB(a)ThequantityincolumnAisgreater.(b)ThequantityincolumnBisgreater.(c)Boththequantitiesareequal.(d)Therelationshipcannotbedetermined.10.Calculatetheareaofthesquarehavingperimeterequaltotheareaofarectangleas44cm².(a)120cm² (b)142cm² (c)121cm² (d)144cm²11.Calculatetheareaofarectanglewithlengthas(1-a)andbreadthas(1+a).

Page 157: Maths book part 2 obtains Rs. 1100 after lending out Rs.x at 5% per annum for 2 years and obtains Rs. 1800 after lending out Rs.y at 10% per annum for 2 years. Find the value of x

157

(a)a² (b)1/a² (c)1+a² (d)1-a²12.Calculatebothsidesofarectangle,giventheperimeterandareaoftherectangleas24mand36m²respecively.(a)10m,2m (b)12m,3m (c)6m,6m (d)18m,2m13.FindtheareaoftriangleABC,whereABisthediameterofacircle.Cliesontheperimeterofthatcircleatadistanceof5unitsfromAand12unitsfromB.(a)32units (b)35units (c)30units (d)31units14.Anisoscelesrighttrianglehashypotenuseof16inches.Findthelengthofotherside.(a)6inches (b)8 2inches(c)7 2inches (d)6 2inches15.Findthenumberofrevolutionsmadebythewheelperkilometer,with14cmradius.(a)App.1000revolutions(b)App.1245revolutions(c)App.1136revolutions(d)App.1263revolutions16.Iftheradiusofthecircleistripled,theareaismultipliedby:(a)8 (b)2 (c)4 (d)remainsunchanged17.Thelengthofawirefencearoundacirculargardenis44meters.Whatisthearea(insq.meters)ofthe2metersconcretepathlaidinsidethefence?(a)24𝜋m² (b)25𝜋m² (c)32𝜋m² (d)33𝜋m²18.Theareaofarhombusis154sq.m.Ifoneofitsdiagonalsis22m,findthelengthoftheotherdiagonal.(a)20m (b)22m (c)14m (d)27m19.Arectangularparkwithlengthandbreadthof11mand22m,issurroundedbyapathof3mwide.Findtheareaofthepath.(a)100m² (b)108m² (c)200m² (d)234m²20.AlinesegmentABis32mlong.ApointCislocatedonABsuchthatAC:CBis5:3.FindthelengthofCB.(a)10m (b)12m (c)20m (d)22m21.Iftheanglesofaquadrilateralareintheratioof3:4:5:6.Calculatethesmallestangle.(a)60° (b)80° (c)100° (d)120°22.Thebaseofrightangletriangleis'b'units.Iftheareaofthetriangleis'a'units,findtheheightofthetriangle.(a)2abunits (b)2a/bunits (c)2abunits (d)Cannotbedetermined

Page 158: Maths book part 2 obtains Rs. 1100 after lending out Rs.x at 5% per annum for 2 years and obtains Rs. 1800 after lending out Rs.y at 10% per annum for 2 years. Find the value of x

158

23.Theperimeteroftherectangleis28cmandthebreadthis6timesthelength.Findtheareaofrectangle.(a)20cm² (b)28cm² (c)14cm² (d)24cm²24.Thesidesofatriangleareintheratio5:6:7.Ifitsperimeteris36cm.Findthelongestsideofthetriangle.(a)10cm (b)14cm (c)12cm (d)16cm25.Arectanglehasanareaof36cmandperimeterof30cm.Findthelargersideofit.(a)15cm (b)18cm (c)10cm (d)12cm26.Theareasoftwocirclesare4:1,findtheratioofthecircumferencesofthecircles:(a)4:1 (b)1:2 (c)1:4 (d)2:127.Abuffaloistiedtothegroundwitharope.Whatshouldbethelengthoftherope,sothatthebuffalocangrazein616m²areaonly?(a)10m (b)12m (c)14m (d)15m28.Calculatethetotalsurfaceareaofacuboidwhosedimensionsare12m,10mand5m.(a)400m² (b)460m² (c)360m² (d)480m²29.Findthetotalsurfaceareaofaconewithheightas21cmandradiusofitsbasebeing28cm.(a)5042cm² (b)5544cm² (c)5142cm² (d)5000cm²30.Findtheinradiusofthetrianglewithsides5,12&13cm?(a)12 (b)11.5(c)2 (d)12.531.Atraintravels4milesnorthfromtheplatform,then4mileswest,then2milesagainnorthandthen4mileswest.Howfaristhetrainfromtheplatform?(a)14miles (b)10miles (c)12miles (d)12.5miles32.Findthecircumradiusofthetrianglewithsides5,12&13cm?(a)12 (b)11.5(c)3 (d)6.533.Aladderwhichis40mtshighisleaningagainstawallwhichis32mtshigh.Howfaristhewallfromthebaseoftheladder.(a)26 2mts (b)25mts (c)24mts (d)25 2mts34.Findthenumberofspokesinthewheelofacycle,giventheanglebetweentwoconsecutivespokesas20°.(a)18 (b)20 (c)36 (d)935.Anice-creamconehastheheightof7cmanddiameterof6cm.Calculatethevolumeoftheice-creamthatwillbefilledinthiscone.(a)164cm³ (b)66cm³ (c)124cm³ (d)98cm³

Page 159: Maths book part 2 obtains Rs. 1100 after lending out Rs.x at 5% per annum for 2 years and obtains Rs. 1800 after lending out Rs.y at 10% per annum for 2 years. Find the value of x

159

36.Anangleisequaltoone-fourthofitssupplement.Theangleis:(a)42° (b)37° (c)57° (d)36°37.Awirewasintheshapeofrectangle,withlengthas14cmandbreadthas11cm.Thewireisthenmoldedintoacircle.Findthecircumferenceofthecircle.(a)44cm (b)54cm (c)50cm (d)40cm38.Twosquareshaveeachsideas20cmand21cmrespectively.Findthesideofthirdsquarewhoseareaisequaltothesumoftheareasofothertwosquares.(a)28cm (b)29cm (c)30cm (d)32cm39.Findtheareaofatrianglehavingsides7m,8m,and9m.(a)12 5m² (b)30m² (c)12 3m² (d)8 5m²40.Findthecircumradiusofthetrianglewithsides3,4&5cm?(a)2.5(b)2 (c)3 (d)12.541.InatrapeziumABCD,AB+CD=24.ColumnX ColumnYLengthofAB LengthofCD(a)ThequantityincolumnXisgreater(b)ThequantityincolumnYisgreater.(c)Boththequantitiesareequal.(d)Therelationshipcannotbedetermined.42.Aplotoflandisintheshapeofatrapeziumwhosedimensionsaregiveninthefigurebelow:

Hencetheperimeterofthefieldis(a)50m(b)64m(c)72m(d)Noneoftheabove43.Findtheareaofthesectorcoveredbythehourhandafterithasmovedthrough3hoursandthelengthofthehourhandis7cm.(a)77sq.cm(b)38.5sq.cm (c)35sq.cm(d)70sq.cm44.Whatisthemeasureofthecircumradiusofatrianglewhosesidesare9,40and41?(a)6cm (b)4cm (c)24.5cm(d)20.5cmQ45.Ifthesumoftheinterioranglesofaregularpolygonmeasuresupto1440degrees,howmanysidesdoesthepolygonhave?

Page 160: Maths book part 2 obtains Rs. 1100 after lending out Rs.x at 5% per annum for 2 years and obtains Rs. 1800 after lending out Rs.y at 10% per annum for 2 years. Find the value of x

160

(a)10 (b)8 (c)12 (d)9Q46.Whatisthemeasureofinradiusofthetrianglewhosesidesare24,7and25?(a)12.5 (b)3 (c)6(d)noneoftheseQ47.Whatisthecircumradiusofatrianglewhosesidesare7,24and25respectively?(a)18 (b)12.5 (c)12(d)14Q48.

ABCDisaparallelogram.BD=2.TheanglesoftriangleBCDareallequal.Whatistheperimeteroftheparallelogram?(a)9 (b)10 (c)11(d)noneofthese49.

PQRSisaparallelogramandST=TR.WhatistheratiooftheareaoftriangleQSTtotheareaoftheparallelogram?(a)1:2(b)2:3 (c)5:6(d)noneofthese50.

Twoequalcirclesarecutoutofarectangleofcardofdimensions16by8.Thecircleshavethemaximumdiameterpossible.Whatistheapproximateareaofthepaperremainingafterthecircleshavebeencutout?(a)21 (b)23(c)25(d)noneofthese

Page 161: Maths book part 2 obtains Rs. 1100 after lending out Rs.x at 5% per annum for 2 years and obtains Rs. 1800 after lending out Rs.y at 10% per annum for 2 years. Find the value of x

161

51.

ASBisaquartercircle.PQRSisarectanglewithsidesPQ=8andPS=6.WhatisthelengthofthearcAQB?(a)5pi (b)6pi (c)7pi(d)noneofthese52.

RadiusofcirclecenterOis3timestheradiusofcirclecenterC.∠C=∠O.IftheshadedareaofcircleCis2thenwhatistheareaoftheshadedpartofcircleO?(a)6 (b)12 (c)18 (d)noneofthese53.

IntriangleABC,AD=DB,DEisparalleltoBC,andtheareaoftriangleABCis40.WhatistheareaoftriangleADE?(a)10 (b)15 (c)20 (d)30 54.

Page 162: Maths book part 2 obtains Rs. 1100 after lending out Rs.x at 5% per annum for 2 years and obtains Rs. 1800 after lending out Rs.y at 10% per annum for 2 years. Find the value of x

162

RectangleABCDhasaperimeterof26.ThehalfcirclewithdiameterADhasanareaof8π.Whatistheperimeterofthepartofthefigurethatisnotshaded?(a)26+4π (b)18+8π (c)18+4π(d)noneofthese55.Findthenumberoftrianglesinanoctagon.(a)326(b)120 (c)56 (d)cannotbedetermined56:Findtheareaofthesectorcoveredbythehourhandafterithasmovedthrough3hoursandthelengthofthehourhandis7cm.(a)77 (b)38.5 (c)35(d)70 57.IfmedianADofanequilateraltriangleABCis9cmandGiscentroid.FindAG?(a)3 (b)6 (c)12 (d)1158.Theratiooftheside&heightofanequilateraltriangleis(a)1:1(b)2: 3(c) 3:2(d)2:159.InarightangletrianglePQR,rightangleatQ,PS=SQ=SRandangleSPQ=540.FindangleRSQ?(a)720(b)1080(c)360(d)540

60.Theareaofatrianglewithbasexunitsisequaltoareaofasquarewithsidexunits.Thenthealtitudeofthetriangleis:(a)x(b)2x(c)3x(d)4x61.Theratiobetweenlength&perimeterofarectangleplotis1:3.Whatistheratiobetweenlength&breadthoftheplot?(a)2:1(b)3:2(c)datainadequate(d)1:262.ThediagonalofasquareAis(x+y).ThediagonalofasquareBwithtwicetheareaofAis:(a) 2(x+y)(b)2(x+y)(c)2x+4y(d)4x+2y63.Theareaofarightangledtriangleis20sq.cm.andoneofthesidescontainingrightangleis4cm.thealtitudeonthehypotenuseis(a)20/ 29(b)8cm(c)10(d) 40/41

Page 163: Maths book part 2 obtains Rs. 1100 after lending out Rs.x at 5% per annum for 2 years and obtains Rs. 1800 after lending out Rs.y at 10% per annum for 2 years. Find the value of x

163

64.Thewholesurfaceofacubeis216cm2.Thevolumeincm3ofcubeis:(a)108(b)54(c)432(d)21665.TheradiusofbaseofarightcircularconeisR&isheightis2H,thenitsvolumeis:(a)<

C𝜋𝑅<𝐻(b)𝜋𝑅<𝐻 (c)2𝜋𝑅<𝐻(d)$

C𝜋𝑅<𝐻

66.Ifthecurvedsurfaceofacylinderbedoubletheareaoftheendsthentheratioofitsheightandradiusis:(a)2:3 (b)1:1(c)2:1(d)1:267.Theradiusofthebaseofacylinderis2cm&itsheight7cm,thenitscurvedsurfaceincm2is:(a)44 (b)22 (c)88 (d)5668.Eachedgeofacubeisincreasedby50%.Thepercentofincreaseinthesurfaceareaofthecubeis:(a)50(b)100 (c)120 (d)12569.Thesidesofatriangleare3cm,4cm&5cm.Itsareaincm2is(a)12(b)15(c)20(d)670.Theradiusofacircleisdiminishedby10%,theareaisdiminishedby:(a)10%(b)21%(c)19%(d)20%71.IfthecostofwhitewashingthefourwallsofarectangularroomisRs.25,thenthecostofwhitewashingaroomtwicethelength,breadthandtheheightwillbeRs.:(a)50(b)100(c)150(d)20072.Thedifferencebetweenthelength&breadthofarectangleis23m.Iftheperimeteroftherectangleis206m,finditsareainsq.cm.(a)2420(b)2480(c)2520(d)152073.Thelength&breadthofarectangleisinratio3:2.Ifcostoffencingit@Rs.12.5permeterisRs.2000.Byhowmuchitslengthexceeditsbreadthinmeters?(a)16(b)32(c)80(d)16074.Thevolumeofacubeis216cm3,itssideis:(a)16(b)6(c)26(d)3275.Whenrecast,theradiusofanironrodismadeone-fourth.Ifitsvolumeremainsconstant,thenthenewlengthwillbecome(a)¼timesoforiginal(b)1/16timesoftheoriginal(c)16timesoforiginal(d)4timesoforiginal

Page 164: Maths book part 2 obtains Rs. 1100 after lending out Rs.x at 5% per annum for 2 years and obtains Rs. 1800 after lending out Rs.y at 10% per annum for 2 years. Find the value of x

164

76.Arightcircularcone&arightcircularcylinderhaveequalbase&equalheight.Iftheradiusofthebase&theheightareintheratio5:12,thenratiooftotalsurfaceareaofthecylindertothatoftheconeis(a)3:1(b)13:9(c)17:9(d)34:977.Eachedgeofacubeisincreasedby20%.Thepercentofincreaseinthesurfaceareaofthecubeis:(a)43(b)45 (c)41 (d)4478.Thesidesofatriangleare9cm,12cm&15cm.Itsareaincm2is(a)12(b)15(c)50(d)5479.Theradiusofacircleisdiminishedby20%,theareaisdiminishedby:(a)40%(b)44%(c)36%(d)20%80.IfthecostofwhitewashingthefourwallsofarectangularroomisRs.50,thenthecostofwhitewashingaroomtwicethelength,breadthandtheheightwillbeRs.:(a)50(b)100(c)150(d)20081.Asolidmetallicconeismelted&recastintoasolidcylinderofthesamebaseasthatofthecone.Iftheheightofcylinderis7cm,theheightoftheconewas(a)20cm (b)21cm (c)28cm (d)24cm82.Themeasures(incm)ofsidesofarightangledtrianglearegivenbyconsecutiveintegers.Itsarea(incm2)isgivenby(a)8 (b)9 (c)5 (d)683.Ifatrianglewithsamebase8cmhasthesameareaasacirclewithradius8cm,thecorrespondingaltitude(incm)ofthetriangleis(a)12π(b)20π(c)16π(d)32π84.Theradiusofthebase&heightofarightcircularconeareinratio5:12.Ifthevolumeoftheconeis314cm3,theslantheightincmis(take𝜋 = 3.14):(a)12 (b)13 (c)15 (d)1785.Thearea(inm2)ofthesquarewhichhasthesameperimeterasarectanglewhoselengthsis48m&is3timesitsbreadthis:(a)1000(b)1024 (c)1600 (d)102586.Theareaofanequilateraltriangleis400 3sqm.Itsperimeteris(a)120(b)150 (c)90 (d)13587.Diameterofawheelis3m.Thewheelrevolves28timesinaminute.Tocover5.280kmdistance,thewheelwilltake

Page 165: Maths book part 2 obtains Rs. 1100 after lending out Rs.x at 5% per annum for 2 years and obtains Rs. 1800 after lending out Rs.y at 10% per annum for 2 years. Find the value of x

165

(a)10min (b)20min (c)30min (d)40min88.Theperimeterofarhombusis40m&itsheight5m.Itsareainsq.mis:(a)60 (b)50(c)45 (d)5589.Theareaofthebiggestcircleinsq.cm.,whichcanbedrawninsideasquareofside21cmis(a)344.5 (b)364.5 (c)346.5 (d)366.590.Theareaofrhombusis150cm2.Thelengthofoneofitsdiagonalis10cm.Thelengthoftheotherdiagonalincmis(a)25 (b)30 (c)35 (d)3691.Acircularwireoftheradius42cmisbentintheformofarectanglewhosesidesarein6:5.Thesmallestsideoftherectangleis(a)60 (b)30 (c)25 (d)3692.Ifradiusofthebaseofaconebedoubled&heightleftunchangedthentheratioofthevolumeofthenewconetothatofoneoriginalconewillbe:(a)1:4 (b)2:1 (c)1:2 (d)4:193.Theareaoftheincircleofanequilateraltriangleofside42cmis(a)231 (b)462 (c)22 3 (d)92494.Thediagonalsoftherhombusare32cm&24cmrespectively.Theperimeteroftherhombusincmis(a)80 (b)72 (c)68 (d)6495.Arectangularwatertankis2.1mlong&1.5mbroad.If630litresofwaterarepouredintotank,howmuchwillthewaterlevelrise?(a)0.2m (b)2m (c)0.63m (d)1.5m96.Howmanysidesdoesaregularpolygonhavewhoseinteriorandexterioranglesareintheratio2:1?(a)3(b)5(c)6(d)1297.ABCisatrianglewithbaseAB.DisapointonABsuchthatAB=5andDB=3.Whatistheratiooftheareaof∆ADCTotheareaof∆ABC?a.3/2b.2/3c.3/5d.4/2598.Intwotriangles,theratiooftheareais4:3andratiooftheirheightsis3:4.Findtheratiooftheirbases?a.16:9b.9:16c.9:12d.16:1299.Thecircumradiusofanequilateraltriangleis8cm.theinradiusofthetriangleis?a.3.25cmb.3.50cmc.4cmd.4.25cm

Page 166: Maths book part 2 obtains Rs. 1100 after lending out Rs.x at 5% per annum for 2 years and obtains Rs. 1800 after lending out Rs.y at 10% per annum for 2 years. Find the value of x

166

100.Fourequalcircleseachofradius“A”unitstouchoneanother.Theareaenclosedbetweenthem(π=22/7)insquareunits,is?a.3A2b.6A2/7c.41A2/7d.A2/7101.ThelengthsoftheperpendicularsdrawnfromanypointintheinteriorofanequilateraltriangletotherespectivesidesareA,BandC.thelengthofeachsideofthetriangleis?a. <

C(A+B+C)b.$

C(A+B+C)c. $

C(A+B+C)d. >

C(A+B+C)

102.Theareaofaregularhexagonofside2 3cmis?a.18 3b.12 3c.36 3d.27 3103.Thediagonalsofarhombusare24m&10m,itsslantheightis(a)60/13 (b)120/13(c)45(d)55104.Theperimeterofarhombusis80m&itsheight5m.Itsareainsq.mis:(a)60 (b)100(c)45(d)55105.Thechordoflength16cmisatadistanceof15cmfromthecentreofthecirclethenthelengthofthechordofthesamecirclewhichisatthedistanceof8cmfromthecentreisEqualto?a.10cmb.20cmc.30cmd.40cm106.Theratiooftheareasoftwoisoscelestriangleshavingthesameverticalangle(anglebetweenequalsides)is1:4,theratiooftheirheightsis?a.1:4b.2:5c.1:2d.3:4107.Threecirclesofdiameter10cmeach,areboundtogetherbyarubberband,incmifitisstretchedasshown,is?a.30b.30+10πc.10πd.60+20π108.Thelengthoftheeachsideofanequilateraltriangleis14 3.Theareaofthein-circle,incm2isa.450b.308c.154d.77109.Eachinteriorangleofaregularpolygonis18ᵒmorethaneighttimesanexteriorangle.Thenumberofsidesofthepolygonis?a.10b.15c.20d.25110.Ifthesumofthreedimensionsandthetotalsurfaceareaofarectangularboxare12cmand94cm2respectively,thanthemaximumlengthofastickthatcanbeplacedinsidetheboxis?a.5 2cmb.5cmc.6cmd.2 5cmg111.Thediagonalofacubeis15 3cm.findthesideofcube?

Page 167: Maths book part 2 obtains Rs. 1100 after lending out Rs.x at 5% per annum for 2 years and obtains Rs. 1800 after lending out Rs.y at 10% per annum for 2 years. Find the value of x

167

a.120b.14c.15d.12112.Thetotalsurfaceareaofasolidrightcircularcylinderistwicethatofasolidsphere.Iftheyhavethesameradii,theratioofthevolumeofthecylindertothatofthesphereisgivenby?a.9:4b.2:1c.3:1d.4:9113.Theradiusoftheincircleofatriangleis2cm.iftheareaofthetriangleis6cm2,thenitsperimeteris?a.2cmb.3cmc.6cmd.9cm114.Thediagonalofacubeis15 3cm.findtheratioofitstotalsurfaceareaandvolume?a.2:5b.5:2c.3:5d.5:3115.Waterisflowingattherateof3km/hrthroughacircularpipeof20cminternaldiameterintoacircularcisternofdiameter10manddepth2m.inhowmuchtimewillthecisternbefilled?a.1hrb.1hr,40minc.1hr,20mind.2hr,40min116.Ifthesideofasquareisincreasedby50%,itsareaisincreasedby?a.125%b.100%c.75%d.50%117.Ifawireisbentintotheshapeofasquare.Thentheareaofthesquaresoformedis81cm2.Whenthewireisbentintoasemicircularshape.Thenthearea,(incm2)ofthesemicirclewillbe?a.22b.44c.77d.154118.Iftheradiusofacircleisincreasedby50%,itsareaisincreasedby?a.125%b.100%c.75%d.50%119.Abicyclewheelmakes5000revolutionsinmoving11km.thentheradiusofthewheelis(incm)?a.70b.35c.17.5d.140120.Ariver3mdeepand40mwideisflowingattherateof2km.hr.Howmuchwater(inliters)willfallintotheseainaminute?a.4,00,000b.40,00,000c.40,000d.4,000121.Theperimeterofatriangleis40cmanditsareais60cm2.Ifthelargestsidemeasures17cm,thenthelength(incm)ofthesmallestsideofthetriangleis?a.4b.6c.8d.15122.Thevolume(inm3)oftherainwaterthatcanbecollectedfrom1.5hectaresofgroundinarainfallof5cmis?a.75b.750c.7500d.75000

Page 168: Maths book part 2 obtains Rs. 1100 after lending out Rs.x at 5% per annum for 2 years and obtains Rs. 1800 after lending out Rs.y at 10% per annum for 2 years. Find the value of x

168

123.Theareaoftheeconsecutivefacesofacuboidare12cm2,20cm2and15cm2,thenthevolumeofthecuboidis?a.3600b.100c.80d.60124.Waterisflowingattherateof5km/hrthroughapipeofdiameter14cmintoaRectangulartankwhichis50mlong,44mwide.Thetaken,inhours,fortheriseinthelevelofwaterinthetanktobe7cmis?a.2b.3/2c.3d.5/2125.Thesidesofatriangleareintheratio2:3:4,theperimeterofthetriangleis18cm.Thearea(incm2)ofthetriangleis?a.9b.36c. 42d.3 15126.Acopperwireisbentintheformofanequilateraltriangleandhasarea121 3cm2.Ifthesamewireisbentintotheformofacircle,theareais(incm2)enclosedbythewireis?a.364.5b.693.5c.346.5d.639.5127.Meetingpointofallperpendicularbisectorsiscalledasa.centroidb.orthocenterc.circumcenterd.incenter128.Waterflowsintoatankwhichis200mlongand150mwide,throughapipeofcross-section0.3m×0.2mat20km/hr.thenthetime(inhr)forthewaterlevelinthetanktoreach8mis?a.50b.120c.150d.200129.InanequilateraltriangleABCofside10cm,thesideBCistrisectedatD.Thelength(incm)ofADis?a.3 7b.7 3c.10 7/3d.7 10/3130.Thefloorofaroomisofsize4m×3𝑚anditsheightis3m.Thewallsandceilingoftheroomrequirepainting.Theareatobepaintedis?a.66m2b.54m2c.43m2d.33m2131.Whatistheratioofcircumradius&inradiusofanequilateraltriangle?a.2:1b.1:2c.1:1d.2:3132.Thelength(incm)ofachordofacircleofradius13cmatadistanceof12cmformitscentreis?a.5b.8c.10d.12133.Theradiusofbaseandslantheightofaconeareintheratio4:7.ifitscurvedsurfaceareais792cm2,thentheradius(incm)ofitsbaseis?a.8b.12c.14d.16134.Theperimeterofarhombusis146cmandoneofitsdiagonalis55cm.Theotherdiagonalis?

Page 169: Maths book part 2 obtains Rs. 1100 after lending out Rs.x at 5% per annum for 2 years and obtains Rs. 1800 after lending out Rs.y at 10% per annum for 2 years. Find the value of x

169

a.92cmb.73cmc.48cmd.72cm135.Theratiooftheareasoftwoisoscelestriangleshavingequalverticalanglesis1:4.Theratiooftheirheightswillbe?a.1:2b.3:4c.2:3d.6:7136.Ifeachoftheradiusofbaseandheightofarightcircularconeisincreasedby10%,thenthepercentageofincreaseinthevolumeoftheconewillbe?a.20b.33.1c.44.2d.100137.Meetingpointofallthealtitudesinatriangleiscalledasa.centroidb.orthocenterc.circumcenterd.incenter138.Meetingpointofallanglebisectorsiscalledasa.centroidb.orthocenterc.circumcenterd.incenter139.Anequilateraltriangleofside6cmhasitscornerscutofftoformaregularhexagon.Area(incm2)ofthisregularhexagonwillbe?a.3 3b.3 6c.6 3d.5 3/2140.Thelength(inmeter)ofthelongestrodthatcanbeputinaroomofdimensions10m×10m×5mis?a.15 3b.15c.10 2d.5 3141.Thelateralsurfaceareaofacylinderis1056cm2anditsheightis16cm.finditsvolume?a.4545cm3b.4455cm3c.5445cm3d.5544cm3142.Thelargestsphereiscurvedoutofacubeofside7cm.Thevolumeofthesphere(incm3)willbe?a.718.66b.543.72c.481.34d.179.67143.TheradiusofcircleAistwicethatofcircleBandtheradiusofthecircleBistwicethatofcircleC.Theirareawillbeintheratio?a.16:4:1b.4:2:1c.1:2:4d.1:4:16144.Througheachvertexofatriangle,alineparalleltotheoppositesideisdrawn.Theratiooftheperimeterofthenewtriangle,thusformed,withthatoftheoriginaltriangleis?a.3:2b.4:1c.2:1d.2:3145.TheradiiofthebasesoftwocylinderAandBareintheratio3:2andtheirheightsintheration:1.IfthevolumeofcylinderAis3timesthatofcylinderB,thevalueofnis?a.4/3b.2/3c.¾d.3/2

Page 170: Maths book part 2 obtains Rs. 1100 after lending out Rs.x at 5% per annum for 2 years and obtains Rs. 1800 after lending out Rs.y at 10% per annum for 2 years. Find the value of x

170

146.Thevolumeofarightcircularcylinderandthatofasphereareequalandtheirradiiarealsoequal.IftheheightofthecylinderbeHandthediameterofthesphereD,thenwhichofthefollowingrelationiscorrect?a.H=Db.2H=Dc.2H=3Dd.3H=2D147.Thecircumferenceofacircleis100cm.Themeasureofasideofthesquareinscribedinthiscircleis?a.25 2πb.50 2/πc.50 2πd.25 2/π148.Theradiioftwocircleare5cmand3cm,thedistancebetweentheircentersis24cm.Thenthelengthofthetransversecommontangentis?a.16cmb.15 2cmc.16 2cmd.15cm149.Eachoftheradiusofthebaseandtheheightofarightcircularcylinderisincreasedby10%.Thevolumeofthecylinderisincreasedby?a.3.31%b.14.5%c.33.1%d.19.5%150.Theheightofacylinderandthatofaconeareintheratio2:3andtheradiioftheirbasesintheratio3:4.Theratiooftheirvolumeswillbe?a.1:9b.2:9c.9:8d.3:8151.Ifthelengthandtheperimeterofarectangleareintheratio5:16.Thenitslengthandbreadthwillbeintheratio?a.5:11b.5:8c.5:4d.5:3152.Iftheperimeterofasemicircularfieldis144m,thenthediameterofthefieldis?a.55mb.30mc.28md.56m153.Theperimeterofarightangledtriangleis30cm.ifitshypotenuseis13cm,andthenfindstheothertwosides(incm)?a.6,11b.5,12c.7,8d.6,9154.Twocircletouchexternally.Thesumoftheirareasis130πcm2andthedistancebetweentheircentersis14cm.findtheradiiofthecircles?a.11cm,15cmb.11cm,4cmc.11cm,6cmd.11cm,3cm155.Theminutehandofaclockis10cmlong.Findtheareaofthefaceoftheclockdescribedbytheminutehandbetween9AMand9:35AM?a.180.5cm2b.183.3cm2c.182.3cm2d.187.3cm2

156.Asolidmetallicsphereofradius3decimetersismeltedtoformacircularsheetof1millimeterthickness.Thediameterofthesheetsoformedis?a.26mb.24mc.12md.6m157.Theheightandtheradiusofthebaseofarightcircularconeare12cmand6cmrespectively.Theradiusofthecircularcross-sectionoftheconecutbyaplaneparalleltoits

Page 171: Maths book part 2 obtains Rs. 1100 after lending out Rs.x at 5% per annum for 2 years and obtains Rs. 1800 after lending out Rs.y at 10% per annum for 2 years. Find the value of x

171

baseatadistanceof3cmfromthebaseis?a.4cmb.5.5cmc.4.5cmd.3.5cm158.Waterflowsthroughacylindricalpipe,whoseradiusis7cm,at5m/sec.Thetime,ittakestofillanemptywatertankwithheight1.54mandareaofthebase(3×5)m2is?a.6minb.5minc.10mind.9min159.Ifthedifferencebetweenareasofthecircumcircleandthein-circleofanequilateraltriangleis44cm2,thentheareaofthetriangleis?a.28cm2b.7 3cm2c.14 3cm2d.21cm2160.Awire,whenbentintheformofasquare,enclosesaregionhavingarea121cm2.Ifthesamewireisbentintotheformofacircle,thentheareaofthecircleis?a.144cm2b.180cm2c.154cm2d.176cm2161.Iftheareaofacircleinscribedinasquareis9𝜋cm2,thentheareaofthesquareis?a.24cm2b.30cm2c.36cm2d.81cm2162.ABCisanequilateraltriangleofside2cm.withA,B,Cascentersandradius1cmthreearcsaredrawn.Theareaoftheregionwithinthetriangleboundedbythethreearcsis?a.(3 3-Ú

<)cm2b.( 3-CÚ

<)cm2c.( 3-Ú

<)cm2d.(Ú

<- 3)

163.Whatistheeachinteriorangleofadecagon?a.360 b.1080 c.1120 d.1440164.ABCisanisoscelestriangleinwhichAB=AC.IfDandEarethemid-pointsofABandACrespectively.ThepointB,C,D,Eare?a.collinearb.non-collinearc.concyclicd.noneofthese165.Whatisthecircumradiusofanequilateraltriangleofside6cm?A.2 2b.3 2c.2 3d.4 2166.Iftwocirclearesuchthatthecentreofoneliesonthecircumferenceoftheotherthentheratioofthecommonchordofthetwocirclestotheradiusofanyoneofthecircleis?a.2:1b. 3:1c. 5:1d.4:1167.Ifoneangleofacyclictrapeziumistripleoftheother,thenthegreateronemeasures?a.90ᵒb.105ᵒc.120ᵒd.135ᵒ 168.InacyclicquadrilateralABCD,if⦞B-⦞D=60ᵒthenthemeasureofthesmallerofthetwois?a.60ᵒb.40ᵒc.38ᵒd.30ᵒ169.Thenumberofthecommontangentsthatcanbedrawntotwogivencirclesisatthemost?

Page 172: Maths book part 2 obtains Rs. 1100 after lending out Rs.x at 5% per annum for 2 years and obtains Rs. 1800 after lending out Rs.y at 10% per annum for 2 years. Find the value of x

172

a.1b.2c.3d.4170.ACBisatangenttoacircleatC.CDandCEarechordssuchthatthat⦞ACE>⦞ACD.if⦞ACD=⦞BCE=50ᵒthen?a.CD=CEb.EDisnotparalleltoABc.EDpassesthroughthecentreofthecircled.∆CDEisarightangledtriangle171.Inacircleofradius17cm,twoparallelchordsaredrawnonoppositesidesofadiameter.Thedistancebetweenthechordsis23cm.ifthelengthofonechordis16cm,thenthelengthoftheotheris?a.23cmb.30cmc.15cmd.noneofthese172.Ifanglebetweentwosidesof3cm&4cmofatriangleis300..whatistheareaofthetriangle?a.6cmb.12cmc.15cmd.noneofthese173.ABCisarightangledtriangleAB=3cm,BC=5cmandAC=4cm,thentheinradiusofthecircle?a.1cmb.1.25cmc.1.5cmd.noneofthese174.Acirclehastwoparallelchordsoflengths6cmand8cm.ifthechordsare1cmapartandthecentreisonthesamesideofthechords,thenadiameterofthecircleisoflength?a.5cmb.6cmc.8cmd.10cm175.Apointwhichisequidistantfromallvertexofatriangleiscalledasa.centroidb.orthocenterc.circumcenterd.incenter176.InacircleOisthecentreandABisachord⦞AOB=50ᵒthenfind⦞OAB=?a.50ᵒb.60ᵒc.55ᵒd.65ᵒ177.INacircleOisthecentre,ADisthediameterandAB,BC,CDarethechord.∠A=50ᵒthen∠O=?a.130ᵒb.50ᵒc.100ᵒd.80ᵒ178.INacirclewithcentreOandradius5cm,ABisachordoflength8cm.ifOMisperpendicularonAB,whatisthelengthofOM?a.4cmb.5cmc.3cmd.noneofthese179.Anequilateral∆ABCisinscribedinacirclewithcentreO.then⦞BOOCisequalto?a.120ᵒb.75ᵒc.180ᵒd.60ᵒ180.Inwhichofthefollowingarethelengthsofdiagonalsequal?a.Rhombusb.Rectanglec.Parallelogramd.Trapezium

Page 173: Maths book part 2 obtains Rs. 1100 after lending out Rs.x at 5% per annum for 2 years and obtains Rs. 1800 after lending out Rs.y at 10% per annum for 2 years. Find the value of x

173

181.Inacircle,PQisthediameterofacirclewithcentreatO.OSisperpendiculartoPR.ThenOSisequalto?a.¼QRb.1/3QRc.½QRd.QR182.InacircleOMandONaretheperpendiculardrawnonthechordsPQandRsifOM=ON=6cm.then?a.PQ≥RSb.PQ<RSc.PQ≤RSd.PQ=RS183.DiameterABandCDofacircleintersectatO.ifangleBOD=50ᵒthenangleAOD?a.50ᵒb.180ᵒc.130ᵒd.310ᵒ184.INacirclewithcentreO,AOCisadiameterofthecircle,BDisachordandOBandCDarejoined.If⦞AOB=130ᵒthenangleBDC=?a.30ᵒb.25ᵒc.50ᵒd.60ᵒ185.ABandCDareequalchordsofacirclewhosecentreisO.WhenproducedthesechordsmeetatEthen?a.EB=EDb.EA=ECc.EA=EDd.both(a)and(b)186.∆ABCisinscribedinacircle⦞P,⦞Qand⦞RareanglesinscribedinthearcscutoffbysideBC,ACandABrespectively.Then⦞P+⦞Q+⦞Rareequalto?a.180ᵒb.360ᵒc.240ᵒd.noneofthese187.Meetingpointofallmediansiscalledasa.centroidb.orthocenterc.circumcenterd.incenter188.OisthecircumcentreofthetriangleABCwithcircumradius13cm.LetBC=24cmandODisperpendiculartoBC.ThenthelengthofODis.a.7cmb.3cmc.4cmd.5cm189.ABisadiameterofacirclewithcentreO.CDisachordequaltotheradiusofthecircle.ACandBDareproducedtomeetatP.thenthemeasureofangleAPBis?a.120ᵒb.30ᵒc.60ᵒd.90ᵒ190.Pisthepointoutsideacircleandis13cmawayfromitscentre.AsecantdrawnfromthepointPintersectsthecircleatpointsAandBinsuchawaythatPA=9cmandAB=7cm.theradiusofthecircleis?a.5.5cmb.5cmc.4cmd.4.5cm191.ABCDisacyclicquadrilateral.sidesABandDC,whenproducedmeetatthepointPandsidesADandBC,whenproducedmeetatthepointQ.ifangleADC=85ᵒandangleBPC=40ᵒthenangleCQDisequalto?a.30ᵒb.40ᵒc.55ᵒd.85ᵒ192.Twocircleofradii8cmand2cmrespectivelytoucheachotherexternallyatthepointA.

Page 174: Maths book part 2 obtains Rs. 1100 after lending out Rs.x at 5% per annum for 2 years and obtains Rs. 1800 after lending out Rs.y at 10% per annum for 2 years. Find the value of x

174

PQisthedirectcommontangentofthosetwocirclesofcentersXandYrespectively.ThenlengthofPQisequaltoa.2cmb.3cmc,4cmd.8cm193.A,B,Carethreepointsonacircle.ThetangentatAmeetBCproducedatT,angleBTA=40ᵒ,anglecat=44ᵒ.TheanglesubtendedbyBCatthecentreofthecircleis?a.84ᵒb.92ᵒc.96ᵒd.104ᵒ194.PQisdirectcommontangentoftwocirclesofradiiR1andR2touchingeachotherexternallyatA.thenthevalueofPQ2is?a.R1×R2b.2R1R2c.3R1R2d.4R1R2195.Twocirclewithradii5cmand8cmtoucheachotherexternallyatapointA.ifastraightlinethroughthepointAcutsthecirclesatpointPandQrespectively,thenAP:AQis?a.8:5b.5:8c.3:4d.4:5196.ABandCDaretwoparallelchordsdrownontwooppositesidesoftheirparalleldiametersuchthatAB=6cm,CD=8cm.iftheradiusofthecircleis5cm,thedistancebetweenthechords,incmis?a.2b.7c.5d.3197.Theradiusofacircleis6cm.anexternalpointisatadistanceof10cmfromthecentre.Thenthelengthofthetangentdrawntothecirclefromtheexternalpointuptothepointofcontactis?a.8cmb.10cmc.6cmd.12cm198.Twocircleofradii4cmand9cmrespectivelytoucheachotherexternallyatapointandacommontangenttouchesthematthepointsPandQrespectively.ThentheareaofasquarewithonesidePQ,isa.81cm2b.121cm2c.196cm2d.144cm2199.TwotangentaredrawnfromapointptoacircleatAandB.Oisthecentreofthecircle.ifangleAOP=60ᵒ,thenangleAPBis?a.120ᵒb.90ᵒc.60ᵒd.30ᵒ200.Ifthelengthofachordofacircle,whichmakesanangle45ᵒwiththetangentdrawnatoneendpointofthechord,is6cm,thentheradiusofthecircleis?a.6 2cmb.5cmc.3 2cmd.6cm201.Theradiusofacircleis13cmandXYisachordwhichisatadistanceof12cmfromthecentre.Thelengthofthechordis?a.15cmb.12cmc.10cmd.20cm202.SRisadirectcommontangenttothecirclesofradii8cmand3cmrespectively,theircentersbeing13cmapart.Ifthepointsofcontact,thanthelengthofSRis?a.12cmb.11cmc.17cmd.10cm

Page 175: Maths book part 2 obtains Rs. 1100 after lending out Rs.x at 5% per annum for 2 years and obtains Rs. 1800 after lending out Rs.y at 10% per annum for 2 years. Find the value of x

175

203.Theradiusoftwoconcentriccirclesare9cmand15cm.ifthechordofthegreatercirclebeatangenttothesmallercircle,thenthelengthofthatchordis?a.24cmb.12cmc.30cmd.18cm204.OandCarerespectivelytheorthocenterandthecircumcentreofanacute-angledtrianglePQR.ThepointPandOarejoinedandproducedtomeetthesideQRatS.ifanglePQS=60ᵒandangleQCR=130ᵒ,thenangleRPS=?a.30ᵒb.35ᵒc.100ᵒd.60ᵒ205.Theratioofcircumradiiandtheinradiiofanequilateraltriangleis?a.2:1b.4:1c.8:1d.1:2206.Theratiooftheareasofthecircumcircleandthein-circleofanequilateraltriangleis?a.2:1b.4:1c.8:1d.3:2207.AB=8cmandCD=6cmaretwoparallelchordsonthesamesideofthecentreofacircle.Thedistancebetweenthemis1cm.theradiusofthecircleis?a.5cmb.4cmc.3cmd.2cm208.Meetingpointofallprependicularsiscalledasa.centroidb.orthocenterc.circumcenterd.incenter209.TWOequalcirclesofradius4cmintersecteachothersuchthateachpassesthroughthecentreoftheother.Thelengthofthecommonchordis?a.2 3cmb.4 3cmc.2 2cmd.8cm210.Thelengthofeachsideofanequilateraltriangleis14 3𝑐𝑚.Theareaofthein-circle,incm2,isa.450b.308c.154d.77211.ThecircumcentreofatriangleABCisO.ifangleBAC=85ᵒandangle=75ᵒ,thenthevalueofangleOACis?a.40ᵒb.60ᵒc.70ᵒd.90ᵒ212.IfIistheincentreof∆ABCandangleA=50ᵒ,thenthevalueofangleBICis?a.25ᵒb.115ᵒc.105ᵒd.80ᵒ213.IfSisthecircumcentreof∆ABCandangleA=50ᵒ,thenthevalueofangleBCSis?a.20ᵒb.40ᵒc.60ᵒd.80ᵒ214.IfIisthein-centreof∆ABCandangleBIC=135ᵒthen∆ABCis?a.acuteangledb.equilateralc.rightangledd.obtuseangled215.IfOisthecircumcentreof∆ABCandangleOBC=35ᵒ,thentheangleBACisequalto?

Page 176: Maths book part 2 obtains Rs. 1100 after lending out Rs.x at 5% per annum for 2 years and obtains Rs. 1800 after lending out Rs.y at 10% per annum for 2 years. Find the value of x

176

a.55ᵒb.110ᵒc.70ᵒd.35ᵒ216.Circumcentreof∆ABCisO.ifangleBAC=85ᵒ,angleBCA=80ᵒ,thenangleOACis?a.80ᵒb.30ᵒc.60ᵒd.75ᵒ217.OisthecentreofacircleandarcABCsubtendsanangleof130ᵒatO.ABisextendedtoP.thenanglePBCis?a.75ᵒb.70ᵒc.65ᵒd.80ᵒ218.ChordsABandCDofacircleintersectexternallyatP.IfAB=6cm,CD=18cmandPD=40cm,thenthelengthofPAis?a.5cmb.6cmc.4cmd.5.5cm219.TwocirclestoucheachexternallyatpointAandPQisadirectcommontangentwhichtouchesthecirclesatPandQrespectively.ThenanglePAQ=?a.45ᵒb.90ᵒc.80ᵒd.100ᵒ220.ThelengthoftwochordsABandACofacircleare8cmand6cmandangleBAC=90ᵒ,thentheradiusofcircleis?a.25cmb.20cmc.4cmd.5cm221.InternalbisectorsofanglesBandCofatriangleABCmeetatO.IfangleBAC=80ᵒ,thenthevalueofangleBOCis?a.120ᵒb.140ᵒc.110ᵒd.130ᵒ222.Theangleofatriangleare(x+5)ᵒ,(2x-3)ᵒand(3x+4)ᵒ.Thevalueofxis?a.30b.31c.29d.28223.InatriangleABC,incentreisOandangle110ᵒ,thenthemeasureofangleBACis?a.20ᵒb.40ᵒc.55ᵒd.110ᵒ224.DisanypointonsideACof∆ABC.ifP,Q,x,Y,arethemid-pointsofAB,BC,AD,andDCrespectively,thentheratioofPXandQYis?a.1:2b.1:1c.2:1d.2:3225.LetObetheincentreofatriangleABCandDbeapointonthesideBCof∆ABC,suchthatODisperpendicularonBC.IfangleBOD=15ᵒ,thenangleABC=?a.75ᵒb.45ᵒc.150ᵒd.90ᵒ226.In∆ABC,PQisparalleltoBC.IfAP:PB=1:2andAQ=3cm,ACisequalto?a.6cmb.9cmc.12cmd.8cm227.Theratiobetweenthenumberofsidesoftwopolygonis2:1andtheratiobetweentheirinteriorangleis4:3.Thenumberofsidesofthesepolygonsisrespectively?a.8,4b.10,5c.12,6d.14,7

Page 177: Maths book part 2 obtains Rs. 1100 after lending out Rs.x at 5% per annum for 2 years and obtains Rs. 1800 after lending out Rs.y at 10% per annum for 2 years. Find the value of x

177

228.Oistheincentreof∆ABCandangleA=30ᵒ,thenangleBOCis?a.100ᵒb.105ᵒc.110ᵒd.90ᵒ229.In∆ABC,angleBAC=90ᵒandAB=½BC.ThenthemeasureofangleABCis?a.60ᵒb.30ᵒc.45ᵒd.15ᵒ230.AstraightlineparalleltothebaseBCofthetriangleABCintersectsABandAcatthepointDandErespectively.Iftheareaofthe∆ABEbe36cm2.Thentheareaofthe∆ACDis?a.18cm2b.36cm2c.18cm2d.36cm2231.In∆ABC,ADistheinternalbisectorofangleA.metingthesideBCatD.ifBD=5cm,BC=7.5cm.ThenAB:ACis?a.2:1b.1:2c.4:5d.3:5232.Gisthecentroidof∆ABC.IfAG=BC,thenangleBGCis?a.90ᵒb.30ᵒc.60ᵒd.120ᵒ233.In∆ABC,angleB=60ᵒ,anglec=40ᵒ.ifADbisectsangleBACandAEisperpendicularonBC,thenangleEADis?a.10ᵒb.20ᵒc.40ᵒd.80ᵒ234.Consider∆ABDsuchthatangleADB=20ᵒandCisapointonBDsuchthatAB=ACandCD=CA.ThenthemeasureofangleABCis?a.40ᵒb.45ᵒc.60ᵒd.30ᵒ235.The3mediansAD,BEandCFof∆ABCintersectatpointG.iftheareaof∆ABCis60cm2.thentheareaofthequadrilateralBDGFis?a.10cm2b.15cm2c.20cm2d.30cm2236.IfGisthecentroidandAD,BE,CFarethreemediansof∆ABCwitharea72cm2,,thentheareaof∆BDGis?a.12cm2b.16cm2c.24cm2d.8cm2237.In∆ABC,ADisthemedianandAD=½BC.IfangleBAD=30ᵒ,thenmeasureofangleACBis?a.90ᵒb.45ᵒc.30ᵒd.60ᵒ238.IfGbethecentroidof∆ABCandtheareaof∆GBDis6cm2.WhereDisthemid-pointofsideBC,thentheareaof∆ABCis?a.18cm2b.12cm2c.24cm2d.36cm2

239.Iftheratioofareasoftwosimilartrianglesis4:9,thentheratiooftheircorrespondingsidesis?a.2:3b.4:3c.4:5d.4:3

Page 178: Maths book part 2 obtains Rs. 1100 after lending out Rs.x at 5% per annum for 2 years and obtains Rs. 1800 after lending out Rs.y at 10% per annum for 2 years. Find the value of x

178

240.LetBEandCFbethe2mediansofa∆ABCandGbetheirintersection.AlsoletEFcutAGatO.ThenAO:OGis?a.1:1b1:2c.2:1d.3:1241.Iftheratioofareasoftwosimilartrianglesis9:16,thentheratiooftheircorrespond-dingsidesis?a.3:5b.3:4c.4:5d.4:3242.In∆ABC,PandQarethemiddlepointsofthesidesABandACrespectively.RisapointonthesegmentPQsuchthatPR:RQ=1:2.ifPR=2cm,thenBC=?a.4cmb.2cmc.12cmd.6cm243.ABCisatriangle.theinternalbisectoroftheanglesA,angleBandangleCintersectthecircumcircleatX,YandZrespectively.IfangleA=50ᵒ,angleCZY=30ᵒ,thenangleBYZ,willbe?a.45ᵒb.55ᵒc.35ᵒd.30ᵒ244.ABCisaright-angledtriangle.ADisperpendiculartothehypotenuseBC.IfAC=2AB,thenthevalueofBDis?a.BC/2b.BC/3c.BC/4d.BC/5245.IFGisthecentroidandADbeAmedianwithlength12cmof∆ABC.ThenthevalueofAGis?a.4cmb.8cmc.10cmd.6cm246.Theperimetersof2similartriangles∆ABCand∆PQRare36cmand24cmrespectively.IfPQ=10cm,thenABis?a.25cmb.10cmc.15cmd.20cm247.DandEarethemid-pointsofABandACof∆ABC,BCisproducedtoanypointPDE,DPandEParejoined.Then?a.∆PED=¼∆ABCb.∆PED=∆BECc.∆ADE=∆BECd.∆BDE=∆BEC248.IfGisthecentroidof∆ABCandAG=BC,thenangleBGCis?a.75ᵒb.45ᵒc.90ᵒd.60ᵒ249.∆ABCand∆DEFaresimilarandtheirareasarerespectively64cm2and121cm2.IfEF=15.4cmBCis?a.12.3cmb.11.2cmc.12.1cmd.11.0cm250.Whatistheratioofin-radiustothecircumradiusofarightangledtriangle?a.1:2b.1: 2c.2:5d.can’tbedetermined251.InABC,Gisthecentroid,AB=15cm,BC=18cmandAC=25cm,findGD,whereDisthemid-pointofBC?

Page 179: Maths book part 2 obtains Rs. 1100 after lending out Rs.x at 5% per annum for 2 years and obtains Rs. 1800 after lending out Rs.y at 10% per annum for 2 years. Find the value of x

179

a. 86/3cmb.2 86/3c.8 15/3cmd.noneofthese252.In∆ABC,AB2+AC2=2500cm2andmedianAD=25cm,findBC?a.25cmb.40cmc.50cmd.48cm253.Areaof∆ABC=30cm2.DandEarethemid-pointsofBCandABrespectively.Findareaof∆ADE?a.10cm2b.7.5cm2c.15cm2d.noneofthese254.ADisthemedianofatriangleABCandOisthecentroidsuchthatAO=10cm.thelengthofODincmis?a.4b.5c.6d.8255.Inanequilateraltriangleofside2a,calculatethelengthofitsaltitude?a.2a 3b.a 3c.a 3/2d.noneofthese256.TheinternalbisectorsofangleBandangleCof∆ABCmeetatO.ifangleA=80ᵒthenangleBOCis?a.50ᵒb.160ᵒc.100ᵒd.130ᵒ257.Onesideotherthanthehypotenuseofrightangleisoscelestriangleis6cm.Thelengthoftheperpendicularonthehypotenusefromtheoppositevertexis?a.6cmb.6 2cmc.4cmd.3 2cm258.Thetriangleisformedbyjoiningthemid-pointsofthesidesAB,BCandCAof∆ABCandtheareaof∆PQRis6cm2,thentheareaof∆ABCis?a.36cm2b.12cm2c.18cm2d.24cm2259.Whatistheratioofsideandheightofanequilateraltriangle?a.2:1b.1:1c.2: 3d. 3:2260.Thedifferencebetweenaltitudeandbaseofarightangledtriangleis17cmanditshypotenuseis25cm.whatisthesumofthebaseandaltitudeofthetriangle?a.24cmb.31cmc.34cmd.can’tbedetermined261.In∆ABC,AB=5cm,AD=7cm.IfADistheanglebisectorofangleA.thenBD:CDis?a.25:49b.49:25c.6:1d.5:7262.ABCDisarhombuswhosesideAB=4cmandangleABC=120ᵒ,thenthelengthofdiagonalBDisequalto?a.1cmb.2cmc.3cmd.4cm263.Ifanexteriorangleofacyclicquadrilateralbe50ᵒ,thentheinterioroppositeangleis?a.130ᵒb.40ᵒc.50ᵒd.90ᵒ

Page 180: Maths book part 2 obtains Rs. 1100 after lending out Rs.x at 5% per annum for 2 years and obtains Rs. 1800 after lending out Rs.y at 10% per annum for 2 years. Find the value of x

180

264.Measureofeachinteriorangleofaregularpolygoncanneverbe?a.150ᵒb.105ᵒc.108ᵒd.144ᵒ265.ABCDisacyclictrapeziumwhosesidesADandBCareparalleltoeachother.ifangleABC=72ᵒ,thenthemeasureoftheangleBCDis?a.162ᵒb.18ᵒc.108ᵒd.72ᵒ266.ThelengthofthediagonalBDoftheparallelogramABCDis18cm.ifPandQarethecentroidofthe∆ABCand∆ADCrespectivelythenthelengthofthelinesegmentPQis?a.4cmb.6cmc.9cmd.12cm267.Eachinteriorangleofaregularpolygonis18ᵒmorethaneighttimesanexteriorangle.thenumberofsideofthepolygonis?a.10b.15c.20d.25268.ThesideABofAparallelogramABCDisproducedtoEinsuchwaythatBE=AB.DEintersectsBCatQ.ThepointQdividesBCintheratio?a.1:2b.1:1c.2:3d.2:1269.Ifthemeasureofeachinteriorangleofaregularpolygonbe144ᵒ,thenumberofsidesofthepolygonis?a.10b.20c.24d.36270.Eachinteriorangleofaregularpolygonis144ᵒ.Thenumberofsidesofthepolygonis?a.8b.9c.10d.11271.IfthelengthofthesidePQoftherhombusPQRSis6cmandanglePQR=120ᵒ,thenthelengthofQSincm,is?a.4b.6c.3d.5272.Ifeachinteriorangleisdoubleofeachexteriorangleofaregularpolygonwithnsides,thenthevalueofnis?a.8b.10c.5d.6273.Ifaregularpolygonhaseachofitsanglesequalto3/5timesoftworightangles,thenthenumberofsidesis?a.3b.5c.6d.8274.Ifeachinteriorangleofaregularpolygonis150ᵒ,whatisthenumberofsidesofapolygon?a.4b.8c.12d.16275.InaquadrilateralABCD,IfAOandBOarethebisectorsofangleAandangleBrespectively,angleC=70ᵒandangleD=30ᵒ,thenangleAOB=?a.40ᵒb.50ᵒc.80ᵒd.100ᵒ

Page 181: Maths book part 2 obtains Rs. 1100 after lending out Rs.x at 5% per annum for 2 years and obtains Rs. 1800 after lending out Rs.y at 10% per annum for 2 years. Find the value of x

181

276.Howmanysidesdoesaregularpolygonhavewhoseinteriorandexterioranglesareintheratio2:1?a.3b.5c.6d.12277.Twoparallelogramsstandonequalbasesandbetweenthesameparallels.Theratiooftheirareasis?a.1:2b.2:1c.1:3d.1:1278.Anycyclicparallelogramisa?a.rhombusb.trapeziumc.quadrilaterald.rectangle279.Theexteriorangleofaregularpolygonisonethirdofitsinteriorangle,thenumberofsidesofthepolygonis?a.2b.4c.6d.8280.Apolygonhas35diagonals,thenumberofsidesofthepolygonis?a.4b.6c.8d.10281.InaparallelogramABCD,thebisectorofangleAandangleBmeetatO.thenangleAOBisequalto?a.85ᵒb.90ᵒc.110ᵒd.noneofthese282.Howmanydiagonalsarethereinanoctagon?a.8b.16c.18d.20283.Aregularpolygonisinscribedinacircle.ifasidesubtendsanangleof72ᵒatthecentre,thenthenumberofsidesofthepolygonis?a.5b.7c.6d.8284.AregularhexagonisinscribedinacirclewithcentreO.thentheanglesubtendedbyeachsideofthesquareatthecentreOis?a.80ᵒb.90ᵒc.60ᵒd.45ᵒ285.Ifanangleofaparallelogramis2/3ofitsadjacentangle,thesmallestangleoftheparallelogramis?a.108ᵒb.54ᵒc.72ᵒd.81ᵒ286.IfABCDisaparallelograminwhichPandQarethecentroidsof∆ABDand∆BCD,thenPQequals?a.AQb.APc.BPd.DQ287.ABCDisaparallelogramandBDisadiagonalangleBAD=65ᵒandangleDBC=45ᵒthenangleBDCis?a.65b.70c.20d.noneofthese288.Theratioofaninteriorangletotheexteriorangletotheexteriorangleofaregular

Page 182: Maths book part 2 obtains Rs. 1100 after lending out Rs.x at 5% per annum for 2 years and obtains Rs. 1800 after lending out Rs.y at 10% per annum for 2 years. Find the value of x

182

polygonis4:1.Thenumberofsidesofpolygonis?a.10b.11c.12d.14289.Thedifferencebetweentheinteriorangleandexterioranglesofaregularpolygonis60ᵒ.Thenumberofsidesofpolygonis?a.4b.5c.6d.8290.Theratioofaninteriorangletotheexteriorangletotheexteriorangleofaregularpolygonis5:1.Thenumberofsidesofpolygonis?a.10b.11c.12d.14291.Theexteriorangleofaregularpolygonisonethirdofitsinteriorangle,thenumberofsidesofpolygonis?a.2b.4c.6d.8292.Theareaofthelargesttrianglethatcanbeinscribedinasemicircleofradiusxinsquareunitis?a.4x2b.x2c.2x2d.3x2293.Theexteriorangleofaregularpolygonisonefourthofitsinteriorangle,thenumberofsidesofpolygonis?a.7b.5c.10d.8294.Findouttheratiooftheareaoftheinscribedandcircumscribedcircleofthesquare?a. 2:1b.1: 2c. 2:1d.1:2295.Areaofthetrapeziumformedbyx-axis;y-axisandthelines3x+4y=12and6x+8y=60is:a.37.5sq.unitb.31.5sq.unitc.48sq.unitd.36.5sq.unit296.Thelengthofthesideofasquareis14cm.Findouttheratiooftheradiioftheinscribedandcircumscribedcircleofthesquare?a. 2:1b.1: 2c. 2:1d.2:1297.Theperimeterofarhombusis146cmandoneofitsdiagonalsis55cm.theotherdiagonalis?a.92cmb.73cmc.48cmd.72cm298.Ifacirclewithradiusof10cmhastwoparallelchords16cmand12cmandtheyareonthesamesideofthecentreofthecircle,thenthedistancebetweenthetwoparallelchordsis?a.2cmb.3cmc.5cmd.8cm299.Ifthelengthofachordofacircleatadistanceof12cmfromthecentreis10cm,thenthediameterofthecircleis?a.13cmb.15cmc.26cmd.30cm

Page 183: Maths book part 2 obtains Rs. 1100 after lending out Rs.x at 5% per annum for 2 years and obtains Rs. 1800 after lending out Rs.y at 10% per annum for 2 years. Find the value of x

183

300.Areaofthein-circleofanequilateraltrianglewithside6cmis?a.π/2cm2b. 3πcm2c.6πcm2d.3πcm2301.Ifthecircumradiusofanequilateraltrianglebe10cm,thenthemeasureofitsin-radiusis?a.5cmb.10cmc.20cmd.15cm302.Atthecentresoftwocircles,twoarcsofequallengthsubtendanglesof60ᵒand75ᵒrespectively.Theratiooftheradiiofthetwocirclesis?a.5:2b.5:4c.3:2d.2:1303.Ametalwirewhenbentintheformofasquareenclosesanarea484cm2,ifthesamewireisbentintheformofacircle,thenitsareais?a.308cm2b.506cm2c.600cm2d.616cm2304.Sidesofaparallelogramareintheratio5:4.itsareais1000sq.units.Altitudeonthegreatersideis20units.Altitudeonthesmallersideis?a.30unitsb.25unitsc.10unitsd.15units305.Theperimeterofarhombusis40cmandthemeasureofanangleis60ᵒ,thentheareaofitis?a.100 3cm2b.50 3cm2c.160 3cm2d.100cm2306.Theadjacentsideofaparallelogramare36cmand27cminlength.Ifthedistancebetweentheshortersidesis12cm.thenthedistancebetweenthelongersidesis?a.10cmb.12cmc.16cmd.9cm307.Thelengthofaroomfloorexceedsitsbreadthby20m.Theareaofthefloorremainsunalteredwhenthelengthisdecreasedby10mbutthebreadthisincreasedby5m.theareaofthefloor(inm2)is?a.280b.325c.300d.420308.Arightangledisoscelestriangleisinscribedinasemi-circleofradius7cm.theareaenclosedbythesemi-circlebutexteriortothetriangleis?a.14cm2b.28cm2c.44cm2d.68cm2309.Theradiioftwocirclesare5cmand3cm,thedistancebetweentheircentresis24cm.Thenthelengthofthetransversecommontangentis?a.16cmb.15 2cmc.16 2cmd.15cm310.Eachoftheheightandradiusofthebaseofarightcircularconeisincreasedby100%.Thevolumeoftheconewillbeincreasedby?a.700%b.500%c.300%d.100%311.Theperimeterofarhombusis150cmandoneofitsdiagonalsis10cm.theotherdiagonal

Page 184: Maths book part 2 obtains Rs. 1100 after lending out Rs.x at 5% per annum for 2 years and obtains Rs. 1800 after lending out Rs.y at 10% per annum for 2 years. Find the value of x

184

is?a.30cmb.60cmc.75cmd.70cm312.Theratioofthevolumeofacubetothatofasphere,whichwillexactlyfitinsidethecubeis?a.2:πb.π:6c.6:πd.8:π313.Acubeofside1meterisreduced3timesintheratio1:2.theareaofonefaceofthereducedcubetothatoftheoriginalcubeisintheratio?a.1:4b.1:8c.1:16d.1:64314.Iftheradiusofasphereisincreasedby2m,itssurface-areaisincreasedby704m2.Whatwastheradiusoftheoriginalsphere?a.16mb.15mc.14md.12m315.IfasphereofradiusRisdividedintofouridenticalparts,thenthetotalsurfaceareaofthefourpartsis?a.4πR2b.2πR2c.8πR2d.3πR2316.Threesphericalballsofradius1cm,2cm,and3cmaremeltedtoformasinglesphericalball.Intheprocess,thelossofmaterialis25%.Theradiusofthenewballis?a.6cmb.5cmc.3cmd.2cm317.Thebaseofarightprismisanequilateraltriangleofarea173cm2andthevolumeoftheprismis10380cm3.Theareaofthelateralsurfaceoftheprismis(use 3=1.73)?a.1200cm2b.2400cm2c.3600cm2d.4380cm2318.Thereisapyramidonabasewhichisaregularhexagonofside2acm.ifeveryslantedgeofthispyramidisoflength5a/2cm,thenthevolumeofthispyramidis?a.3a3cm3b.3 2a3cm3c.3 3a3cm3d.6a3cm3319.Thebaseofarightprismisanequilateraltriangleofside8cmandheightoftheprismis10cm.Thenthevolumeoftheprismis?a.320 3cm3b.160 3cm3c.150 3cm3d.300 3cm3320.Theheightofaconeis30cm.Asmallconeiscutoffatthetopbyaplaneparalleltothebase.Ifitsvolumebe1/27ofthevolumeofthegivencone,atwhatheightabovethebase,thesectionhasbeenmade?a.10cmb.12cmc.16cmd.20cm

Answers

Page 185: Maths book part 2 obtains Rs. 1100 after lending out Rs.x at 5% per annum for 2 years and obtains Rs. 1800 after lending out Rs.y at 10% per annum for 2 years. Find the value of x

185

1.D 2.C 3.A 4.B 5.C 6.D 7.A 8.A 9.D 10.C11.D 12.C 13.C 14.B 15.C 16.A 17.A 18.C 19.D 20.B21.A 22.B 23.D 24.B 25.D 26.D 27.C 28.B 29.B 30.C31.B 32.D 33.C 34.A 35.B 36.D 37.C 38.B 39.A 40.A41.D 42.C 43.B 44.D 45.A 46.B 47.B 48.D 49.D 50.D51.A 52.C 53.A 54.D 55.C 56.B 57.B 58.B 59.B 60.B61.A 62.A 63.A 64.D 65.A 66.C 67.C 68.D 69.D 70.C71.B 72.C 73.A 74.B 75.C 76.C 77.D 78.D 79.C 80.D81.B 82.D 83.C 84.B 85.B 86.A 87.B 88.B 89.C 90.B91.A 92.D 93.B 94.A 95.A 96.C 97.D 98.A 99.C 100.B101.A 102.A 103.B 104.B 105.C 106.C 107.B 108.C 109.C 110.A111.C 112.A 113.C 114.A 115.B 116.B 117.C 118.A 119.B 120.B121.C 122.B 123.D 124.A 125.D 126.C 127.C 128.D 129.C 130.B131.A 132.C 133.B 134.C 135.A 136.B 137.B 138.D 139.C 140.B141.D 142.D 143.A 144.C 145.A 146.D 147.D 148.C 149.C 150.D151.D 152.D 153.B 154.D 155.B 156.C 157.C 158.D 159.B 160.C161.C 162.C 163.D 164.C 165.C 166.B 167.D 168.A 169.D 170.B171.B 172.A 173.A 174.A 175.C 176.D 177.A 178.C 179.A 180.B181.C 182.D 183.C 184.B 185.B 186.B 187.A 188.D 189.C 190.A191.A 192.D 193.D 194.D 195.A 196.B 197.A 198.D 199.D 200.C201.C 202.A 203.A 204.B 205.A 206.A 207.A 208.B 209.B 210.C211.A 212.B 213.B 214.C 215.A 216.D 217.C 218.C 219.B 220.D221.D 222.C 223.B 224.B 225.C 226.B 227.B 228.B 229.A 230.D231.A 232.A 233.A 234.A 235.C 236.A 237.D 238.D 239.A 240.D241.B 242.C 243.C 244.D 245.B 246.C 247.A 248.C 249.B 250.D251.D 252.C 253.B 254.B 255.B 256.D 257.D 258.D 259.C 260.B261.D 262.D 263.C 264.B 265.C 266.B 267.C 268.B 269.A 270.A271.B 272.D 273.B 274.C 275.B 276.C 277.D 278.D 279.D 280.D281.B 282.D 283.A 284.C 285.C 286.B 287.B 288.A 289.C 290.C291.D 292.B 293.C 294.D 295.B 296.B 297.C 298.A 299.C 300.D301.A 302.B 303.D 304.B 305.B 306.D 307.C 308.B 309.C 310.A311.A 312.B 313.C 314.D 315.A 316.C 317.C 318.C 319.B 320.D

Page 186: Maths book part 2 obtains Rs. 1100 after lending out Rs.x at 5% per annum for 2 years and obtains Rs. 1800 after lending out Rs.y at 10% per annum for 2 years. Find the value of x

186

TrigonometryIfoneangleofatriangleis90degreesandoneoftheotheranglesisknown,thethirdistherebyfixed,becausethethreeanglesofanytriangleaddupto180degrees.Thetwoacuteanglesthereforeaddupto90degrees:theyarecomplementaryangles.Theshapeofatriangleiscompletelydetermined,exceptforsimilarity,bytheangles.Oncetheanglesareknown,theratiosofthesidesaredetermined,regardlessoftheoverallsizeofthetriangle.Ifthelengthofoneofthesidesisknown,theothertwoaredetermined.TheseratiosaregivenbythefollowingfunctionsoftheknownangleA,wherea,bandcrefertothelengthsofthesidesintheaccompanyingfigure:

Sinefunction(sin),definedastheratioofthesideoppositetheangletothehypotenuse.

SinA=opposite/hypotenuse=a/cCosinefunction(cos),definedastheratiooftheadjacentlegtothehypotenuse.CosA=adjacent/hypotenuse=b/cTangentfunction(tan),definedastheratiooftheoppositelegtotheadjacentleg.

TanA=opposite/adjacent=a/b=sinA/cosA

Thehypotenuseisthesideoppositetothe90degreeangleinarighttriangle;itisthelongestsideofthetriangle,andoneofthetwosidesadjacenttoangleA.TheadjacentlegistheothersidethatisadjacenttoangleA.TheoppositesideisthesidethatisoppositetoangleA.Thetermsperpendicularandbasearesometimesusedfortheoppositeandadjacentsidesrespectively.Thereciprocalsofthesefunctionsarenamedthecosecant(cosecorcsc),secant(sec),andcotangent(cot),respectively:

Page 187: Maths book part 2 obtains Rs. 1100 after lending out Rs.x at 5% per annum for 2 years and obtains Rs. 1800 after lending out Rs.y at 10% per annum for 2 years. Find the value of x

187

CosecA=1/sinA=c/aSecA=1/cosA=c/bCotA=1/tanA=cosA/sinA=b/aComplementaryanglesinTrigonometryComplementaryanglesintrigonometry:Twoanglesaresaidtobecomplementary,iftheirsumis900.Itfollowsfromtheabovedefinitionthatθand(90-θ)arecomplementaryanglesintrigonometryforanacuteangleθInΔABC,∠B=900∴∠A+∠C=900∠C=900-∠Asin(900-A)=cosAtan(900-A)=cotAsec(900-A)=cosecAcos(900-A)=sinAcot(900-A)=tanAcosec(900-A)=secATrigonometricEquationsAnequationinvolvingtrigonometricratiosofanangleθ(say)issaidtobeatrigonometricequations,ifitissatisfiedforallvaluesofθforwhichthegiventrigonometricratiosaredefined.SomeTrigonometricequations(Identities)areasfollows:1)Sin²θ+cos2θ=12)sin2θ=1-cos2θ3)cos2θ=1-sin2θ4)1+tan2θ=sec2θ5)tan2θ=sec2θ-16)sec2θ-tan2θ=17)1+cot2θ=cosec2θ8)cot2θ=cosec2θ-19)cosec2θ-cot2θ=1TheseTrigonometricequationsaretrueforanyangleθforwhichthetrigonometricratiosaremeaningful.

Page 188: Maths book part 2 obtains Rs. 1100 after lending out Rs.x at 5% per annum for 2 years and obtains Rs. 1800 after lending out Rs.y at 10% per annum for 2 years. Find the value of x

188

QuadrantsBelowisasimplediagramtohelpyoudeterminethesign(positiveornegative)ofthetrigratiointheirrespectivequadrants.Wecallthisdiagramthe‘ASTC'diagram

Forexampleweneedtofindvalueofsine1300

θIsinthe2ndquadrantandthebasicangleofθis1300.ThebasicangleismeasuredastheacuteanglewhichOPmakeswiththex-axis.Sincesineispositiveinthe2ndquadrantasseeninthe‘ASTC'diagram,sin 130° = sin 50°.Thusthetrigratiowouldbesine130°

Page 189: Maths book part 2 obtains Rs. 1100 after lending out Rs.x at 5% per annum for 2 years and obtains Rs. 1800 after lending out Rs.y at 10% per annum for 2 years. Find the value of x

189

Trigonometricvalues

RadiansIntrigonometryweconsiderπ=180°

Theradianmeasure,θ,oftheangleAOBisdefinedby:

S=lengthofarcr=radiusofcircleToconvertbetweendegreesandradians:

Page 190: Maths book part 2 obtains Rs. 1100 after lending out Rs.x at 5% per annum for 2 years and obtains Rs. 1800 after lending out Rs.y at 10% per annum for 2 years. Find the value of x

190

1.Multiplyadegreemeasureby 0180rad π andsimplifytoconverttoradians.

2.Multiplyaradianmeasurebyrad

1800

πandsimplifytoconverttodegrees.

Chartofsomepopularradians

Page 191: Maths book part 2 obtains Rs. 1100 after lending out Rs.x at 5% per annum for 2 years and obtains Rs. 1800 after lending out Rs.y at 10% per annum for 2 years. Find the value of x

191

Someimportantpoints:-

1.Thelengthsofthesidesofa450-450-900triangleareintheratioof1:1:√2.

2.Thelengthsofthesidesofa30°-60°-90°triangleareintheratioof1:√3:2

3.𝝅radian=𝟏𝟖𝟎°

4.If𝒙 + 𝟏𝒙=2then𝒙=1&If𝒙 + 𝟏

𝒙=−2then𝒙=−1

5.Pythagorastriplet:(3,4,5),(5,12,13),(8,15,17),(7,24,25),(9,40,41),(20,21,29)

6.Twoanglesaresaidtobecomplementary,iftheirsumis900.

7.Twoanglesaresaidtobesupplementary,iftheirsumis1800.

8.IftanA.tanB=1thenA&BarecomplementaryanglesitmeansA+B=900

Page 192: Maths book part 2 obtains Rs. 1100 after lending out Rs.x at 5% per annum for 2 years and obtains Rs. 1800 after lending out Rs.y at 10% per annum for 2 years. Find the value of x

192

Example1:-cosec(90o−A)=a.cosecA b.sinA c.cosA d.tanA e.secASolution:cosec(90o−A)=1/sin(90o−A)=1/cosA=secA.Example2:-sec(90o−A)=a.secA b.sinA c.cosA d.cosecA e.tanASolution:sec(90o−A)=1/cos(90o−A)=1/sinA=cosecA.Example3:-cot(90o−A)=a.tanA b.cosA c.sinA d.secA e.cosecASolution:cot(90o−A)=1/tan(90o−A)=1/cotA=tanA.Example4:-sin(90o−A)/cos(90o−A)=a.tanA b.cot(90o−A) c.cotA d.sinA e.noneSolution:Weknowsin(90o−A)=cosA.Similarlycos(90o−A)=sinA.Substitutingthesevaluesintheexpression,wegetsin(90o−A)/cos(90o−A)=cosA/sinA=cotA.Example5:-cos(90o−A)/sin(90o−A)=a.tanA b.cotA c.tan(90o−A) d.sec(90o−A) e.noneSolution:Substitutingthevaluescos(90o−A)=sinAandsin(90o−A)=cosAintheexpression,wegetcos(90o−A)/sin(90o−A)=sinA/cosA=tanA.Example6:-[cosec2A−1]scos(90o−A)/sin(90o−A)=a.tanA b.sinA c.cotA d.cosASolution:Thevalueofcosec2A−1=cot2A.Alsothevalueofcos(90o−A)/sin(90o−A)=sinA/cosA=tanA.Substitutingthesevalues,theanswerisfoundtobecot2AtanA=cotA.Example7:-cotA[cos(90o−A)/sin(90o−A)]=a.tanA b.cot2A c.tan2A d.cotA e.1Solution:Thevalueofcos(90o−A)/sin(90o−A)=sinA/cosA=tanA.ThereforethegivenexpressionreducestocotAtanAwhichequals1.Example8:-Thevalueoftan45o−cos45osin45oisa.1/2 b.1 c.0 d.3/4 e.1/4Solution:Fromthetableofvaluesoftrigonometricfunctions,tan45o−cos45osin45o=1−(1/2)=1/2.Example9:-Thevalueofsin230o+cos230oisa.0 b.1 c.1/2d.3/2 e.1/4Solution:Fromthetableofvaluesoftrigonometricfunctions,sin230o+cos230o=¼+¾=1.Butitshouldberememberedthatsin2A+cos2A=1,forallvaluesofA.

Page 193: Maths book part 2 obtains Rs. 1100 after lending out Rs.x at 5% per annum for 2 years and obtains Rs. 1800 after lending out Rs.y at 10% per annum for 2 years. Find the value of x

193

Example10:-Thevalueoftan45o+cos0+sin90oisa.2 b.1 c.1/2d.0 e.3Solution:Fromthetableofvaluesoftrigonometricfunctions,tan45o+cos0+sin90o=1+1+1=3.Example11:-Thevalueoftan60ocos30o−sin60otan30oisa.0 b.1/2 c.3/2 d.1 e.2Solution:Fromthetableofvaluesoftrigonometricfunctions,tan60ocos30o−sin60otan30o=(3/2)−(1/2)=1.Example12:-Thevalueof[1+sin60o+sin230o+sin260o+sin445o][cos30o−sin60o]isa.1 b.13/12 c.12/13 d.1/2 e.0Solution:Thevalueofthesecondterm[cos30o−sin60o]is0.Hencethevalueoftheentireexpressionis0,irrespectiveofthevalueofthefirstterm.Example13:-(sinA+cosA)2−2sinAcosA=a.2 b.0 c.1 d.tanA e.sin2A−cos2ASolution:(sinA+cosA)2−2sinAcosA=sin2A+cos2A=1.Example14:-sin2A−sec2A+cos2A+tan2A=a.1 b.0 c.cotA d.cosecA e.cosec2ASolution:Herethetermsneedtobegroupedproperly.Thegivenexpressioncanbewrittenas(sin2A+cos2A)−(sec2A−tan2A)=1−1=0.Example15:-1/(1+cot2A)+1/(1+tan2A)=a.0 b.sin2A c.1 d.cos2A e.sin2A/cos2ASolution:Theexpression1/(1+cot2A)+1/(1+tan2A)=1/cosec2A+1/sec2A=sin2A+cos2A=1.Example16:-cotAtanA=a.sinA b.cosA c.sinAcosA d.1 e.1/(sinAcosA)Solution:cotA=1/tanA.HencecotAtanA=1.AlternativelycotA=cosA/sinAandtanA=sinA/cosA.SocotAtanA=(cosA/sinA)(sinA/cosA)=1.Example17:-

Fromthefigure,thevalueofcosecA+cotAisa.(a+b)/c b.(b+c)/a c.a/(b+c) d.b/(a+c)e.(a+c)/bSolution:-WeknowcosecA=b/aandcotA=c/a.HencecosecA+cotA=(b+c)/a.

Page 194: Maths book part 2 obtains Rs. 1100 after lending out Rs.x at 5% per annum for 2 years and obtains Rs. 1800 after lending out Rs.y at 10% per annum for 2 years. Find the value of x

194

Example18:-Whichofthefollowingrelationshipsistrue:a.sinAcotA=1b.sinA+cosecA=1c.cosAsecA=1d.secA-cosA=1e.secAcotA=1Solution:cosAsecA=1Bydefinition,secA=1/cosA.SocosAsecA=1istrue.Example19:-

Fromthefigure,thevalueofsin2A+cos2Aisa.a/b+c/b b.b/a+c/b c.1 d.(a/b+c/b)2 e.(b/a+c/b)2Solution:Thisquestionisabittricky.WeknowsinA=a/bandcosA=c/b.Sosin2A+cos2A=(a2+c2)/b2.ByPythagorasTheorem,a2+c2=b2foraright-angledtriangle.Hencesin2A+cos2A=1,whichisafamousidentity.Example20:-Fromthefigure,thevalueofcotC+cosecCis

a.(a+c)/b b.(a+b)/c c.(c+b)/a d.a/c+c/b e.c/a+b/cSolution:cotCisBase/OppositeSideandcosecCisHypotenuse/OppositeSide.Fromthesedefinitions,thevaluesofcotCandcosecCaregivenbya/candb/crespectively.Hencetheansweris(a+b)/c.Example21:-cosecA/secA=a.tanA b.sinA c.cosA d.cotA e.sinA+cosASolution:Bydefinition,cosecA=1/sinAandsecA=1/cosA.SocosecA/secA=cosA/sinA=cotA.Example22:-Forthefiguregivenontheright,thevalueofcotAis

Page 195: Maths book part 2 obtains Rs. 1100 after lending out Rs.x at 5% per annum for 2 years and obtains Rs. 1800 after lending out Rs.y at 10% per annum for 2 years. Find the value of x

195

a.sinA/cosA b.tanC c.cosC/sinCd.a/c e.c/bSolution:ThevalueofcotAisc/a.SimilarlythevalueoftanCisc/a.HencecotA=tanC.Example23:-Theangleofelevationofthetopofatower30mhigh,fromtwopointsonthelevelgroundonitsoppositesidesare45degreesand60degrees.Whatisthedistancebetweenthetwopoints?a.30 b.51.96 c.47.32 d.81.96Solution:LetOTbetetower.Therefore,Heightoftower=OT=30mLetAandBbethetwopointsonthelevelgroundontheoppositesideoftowerOT.Then,angleofelevationfromA= TAO=45oandangleofelevationfromB= TBO=60oDistancebetweenAB=AO+OB=x+y(say)Now,inrighttriangleATO,AO=OT=30&inrighttriangleBTO OB=30/√3=30√3/3=10√3=10×1.732=17.32Hence,therequireddistance=x+y=30+17.32=47.32m

Example24:-IfSin(A+B)= 3/2andSin(A-B)=1/2thenwhatarethevaluesofAandB?(giventhatbothAandBareacuteanglesandA>B)

a.30,60 b.45,45 c.45,15 d.noneofabove

Solution:

Sin(A+B)=root3/2(thisisgiveninthequestionitself).

Ifyoulookatthetable,sin60= 3/2.ThatmeansA+B=60………eq1

Similarlywe’llgetA-B=30……….eq2

Sowe’vetwoequations:

A+B=60&A-B=30

Nowaddthesetwoequationseq1+eq2

(A+B)+(A-B)=60+30

2A=90

A=45

Putthisvaluebackineq1(oreq2).AndyougetB=15

Page 196: Maths book part 2 obtains Rs. 1100 after lending out Rs.x at 5% per annum for 2 years and obtains Rs. 1800 after lending out Rs.y at 10% per annum for 2 years. Find the value of x

196

FinalanswerC:45,15

Example25:-Findthevalueofcos2×cos4×cos6×cos8×….×cos92

Solution:

Asyoucanseetheanglesareincreasingasperthemultiplicationtableof2.Sointhechain,you’llalsogetcos90(because2x45=90).Sowecanwritethechainas

cos2×cos4×cos6×cos8×….cos88×cos90×cos92

butweknowthatcos90=0,hencethewholemultiplicationwillbecomezero.

Example26:-Theangleofelevationofthetopofatowerfromapointontheground,whichis30mawayfromthefootofthetower,is30°.Findtheheightofthetower.

Solution:

tan30°=p/30orp=30/ 3=10 3=17.32

Example27:-Akiteisflyingataheightof60mabovetheground.Thestringattachedtothekiteistemporarilytiedtoapointontheground.Theinclinationofthestringwiththegroundis60°.Findthelengthofthestring,assumingthatthereisnoslackinthestring.

Solution:

Sin60°=p/h=60/ℎ

Or 3/2=60/horh=120/ 3=40 3

Example28:-A1.5mtallboyisstandingatsomedistancefroma30mtallbuilding.Theangleofelevationfromhiseyestothetopofthebuildingincreasesfrom30°to60°ashewalkstowardsthebuilding.Findthedistancehewalkedtowardsthebuilding.

Solution:AB=30m(heightofbuilding)

Page 197: Maths book part 2 obtains Rs. 1100 after lending out Rs.x at 5% per annum for 2 years and obtains Rs. 1800 after lending out Rs.y at 10% per annum for 2 years. Find the value of x

197

DC=EG=1.5m(heightofboy)AngleD=30°AngleE=60°AF=30-1.5=28.5mIn∆AFD,tan30°=AF/FD=28.5/FDor1/ 3=28.5/FDFD=28.5 3In∆AFE,tan60°=AF/FE=28.5/FEor1/ 3=28.5/FEFD=28.5/ 3Requireddistance=ED=28.5 3-28.5/ 3=19 3Example29:-Fromapointontheground,theanglesofelevationofthebottomandthetopofatransmissiontowerfixedatthetopofa20mhighbuildingare45°and60°respectively.Findtheheightofthetower.

Solution:Heightofbuilding=DB=20mAngleDCB=45°In∆DBC,tan45°=1=DB/BCorDB=BC=20mIn∆ABC,tan60°= 3=AB/BCorAB=20 3NowAD=AB–DB=20 3 − 20=14.64mExample30:-Astatue,1.6mtall,standsonthetopofapedestal.Fromapointontheground,theangleofelevationofthetopofthestatueis60°andfromthesamepointtheangleofelevationofthetopofthepedestalis45°.Findtheheightofthepedestal.

Solution:Heightofstatute=AD=1.6mAngleACB=60°AngleDCB=45°In∆DBC,tan45°=1=DB/BC

Page 198: Maths book part 2 obtains Rs. 1100 after lending out Rs.x at 5% per annum for 2 years and obtains Rs. 1800 after lending out Rs.y at 10% per annum for 2 years. Find the value of x

198

OrBC=DBIn∆ABC,tan60°= 3=AB/BCorBC=AB/ 3=(BC+16)/ 3orBC 3-BC=16orBC=16/0.732=2.18mExample31:-Theangleofelevationofthetopofabuildingfromthefootofthetoweris30°andtheangleofelevationofthetopofthetowerfromthefootofthebuildingis60°.Ifthetoweris50mhigh,findtheheightofthebuilding.

Solution:heightoftower=AB=50mAngleACB=60°AngleDBC=60°In∆ABC,tan60°= 3=50/BCorBC=50/ 3In∆DCB,tan30°=1/ 3=DC/BCorDC=50/3mExample32:-Twopolesofequalheightsarestandingoppositeeachotheroneithersideoftheroad,whichis80mwide.Fromapointbetweenthemontheroad,theanglesofelevationofthetopofthepolesare60°and30°,respectively.Findtheheightofthepolesandthedistancesofthepointfromthepoles.

Solution:BD=widthofroad=80m

Page 199: Maths book part 2 obtains Rs. 1100 after lending out Rs.x at 5% per annum for 2 years and obtains Rs. 1800 after lending out Rs.y at 10% per annum for 2 years. Find the value of x

199

AngleACB=60°AngleECD=30°AB=ED=heightofpolesIn∆ABC,tan60°= 3=AB/BCorAB=BC 3……..(i)In∆EDC,tan30°=1/ 3=ED/CD=AB/(80-BC)OrAB=(80-BC)/ 3……(ii)OrBC 3=(80-BC)/ 3Or3BC=80–BCOr4BC=80OrBC=20soAB=20 3Example33:-ATVtowerstandsverticallyonabankofacanal.Fromapointontheotherbankdirectlyoppositethetower,theangleofelevationofthetopofthetoweris60°.Fromanotherpoint20mawayfromthispointonthelinejoiningthispointtothefootofthetower,theangleofelevationofthetopofthetoweris30°.Findtheheightofthetowerandthewidthofthecanal.

Solution:widthofcanal=BCCD=20mAngleACB=60°AngleADB=30°In∆ABC,AB=BC 3In∆ABD,AB=(BC+20)/ 3or3BC=BC+20or2BC=20orBC=10(widthofcanal)Example34:-Fromthetopofa7mhighbuilding,theangleofelevationofthetopofacabletoweris60°andtheangleofdepressionofitsfootis45°.Determinetheheightofthetower.

Page 200: Maths book part 2 obtains Rs. 1100 after lending out Rs.x at 5% per annum for 2 years and obtains Rs. 1800 after lending out Rs.y at 10% per annum for 2 years. Find the value of x

200

Solution: AB=heightofbuilding=7m AngleACB=45°AngleEAD=60°In∆ABC,AB=BCorAB=BC=AD=7m,In∆EAD,ED= 3AD=7 3Soheightoftower=DC+ED=7+7 3=7(1+ 3)=19.124mExample35:-Asobservedfromthetopofa75mhighlighthousefromthesea-level,theanglesofdepressionoftwoshipsare30°and45°.Ifoneshipisexactlybehindtheotheronthesamesideofthelighthouse,findthedistancebetweenthetwoships.

Solution:AB=75mAngleEAC=AngleACB=45°AngleEAD=AngleADB=30°In∆ABC,AB=AC=75m,In∆ABD,BD=AB 3=75 3SodistancebetweenshipsC&D=CD=75 3-75=75( 3 − 1)=54.9mExample36:-A1.2mtallgirlspotsaballoonmovingwiththewindinahorizontallineataheightof88.2mfromtheground.Theangleofelevationoftheballoonfromtheeyesofthegirlatanyinstantis60°.Aftersometime,theangleofelevationreducesto30°.Findthedistancetravelledbytheballoonduringtheinterval.

Solution:

Page 201: Maths book part 2 obtains Rs. 1100 after lending out Rs.x at 5% per annum for 2 years and obtains Rs. 1800 after lending out Rs.y at 10% per annum for 2 years. Find the value of x

201

AC=GH=heightofballoon=88.2mBC=EF=heightofGirl=1.2mAngleGFB=60°AngleAFB=30°In∆ABF,BF=87 3In∆GFI,FI=87/ 3Distancetraveledbyballoon=BI=58 3Example37:-Theanglesofelevationofthetopofatowerfromtwopointsatadistanceof4mand9mfromthebaseofthetowerandinthesamestraightlinewithitarecomplementary.Provethattheheightofthetoweris6m.

Solution:BD=4mangleAngleADB=𝜃AngleACB=90–𝜃In∆ABD,AB=tan𝜃×4In∆ABC,tan(90-𝜃)=AB/BCORAB=Cot𝜃×9or4tan𝜃=9Cot𝜃ottan2𝜃=9/4ortan𝜃=3/2=AB/4orAB=6m

Example38:-Amaniswalkingalongastraightroad.HenoticesthetopofatowersubtendinganangleA=60owiththegroundatthepointwhereheisstanding.Iftheheightofthetowerish=30m,thenwhatisthedistance(inmeters)ofthemanfromthetower?Solution:

LetBCrepresentthetowerwithheighth=30m,andArepresentthepointwherethemanisstanding.AB=ddenotesthedistanceofthemanfromtower.TheanglesubtendedbythetowerisA=60o.Fromtrigonometry,

tanA=tan60o=h/d=√3Sod=30/√3sm.Hencethedistanceofthemanfromthetoweris17.32m.

Page 202: Maths book part 2 obtains Rs. 1100 after lending out Rs.x at 5% per annum for 2 years and obtains Rs. 1800 after lending out Rs.y at 10% per annum for 2 years. Find the value of x

202

Example39:-Alittleboyisflyingakite.Thestringofthekitemakesanangleof30owiththeground.Iftheheightofthekiteish=18m,findthelength(inmeters)ofthestringthattheboyhasused.Solution:

IfthekiteisatCandtheboyisatA,thenAC=lrepresentsthelengthofthestringandBC=hrepresentstheheightofthekite.Fromthefigure,sinA=sin30o=h/l=1/2.Hencethelengthofthestringusedbythelittleboyisl=2h=2(18)=36m.

Example40:-Twotowersfaceeachotherseparatedbyadistanced=45m.Asseenfromthetopofthefirsttower,theangleofdepressionofthesecondtower'sbaseis60oandthatofthetopis30o.Whatistheheight(inmeters)ofthesecondtower?Solution:

ThefirsttowerABandthesecondtowerCDaredepictedinthefigureontheleft.FirstconsiderthetriangleBAC.AngleC=60o.tanBCA=tan60o=AB/AC.ThisgivesAB=dtan60o.SimilarlyforthetriangleBED,BE=dtan30o.NowheightofthesecondtowerCD=AB−BE=d(tan60o−tan30o)=45(√3−1/√3)=45×2/√3=51.96m.

Example41:-Ashipofheighth=12missightedfromalighthouse.Fromthetopofthelighthouse,theangleofdepressiontothetopofthemastandthebaseoftheshipequal30oand45orespectively.Howfaristheshipfromthelighthouse(inmeters)?Solution:LetABrepresentthelighthouseandCDrepresenttheship.Fromthefigure,tanBCA=tan45o=AB/AC.SimilarlyforthetriangleBED,tanBDE=tan30o=BE/ED.Now,AC=ED=d.Heightoftheship=CD=AB−BE=d(tan45o−tan30o)=12m.Thusdistanceoftheshipfromthelighthoused=12/(1−1/√3)=28.39m

Page 203: Maths book part 2 obtains Rs. 1100 after lending out Rs.x at 5% per annum for 2 years and obtains Rs. 1800 after lending out Rs.y at 10% per annum for 2 years. Find the value of x

203

Example42:-TwomenonoppositesidesofaTVtowerofheight28mnoticetheangleofelevationofthetopofthistowertobe45oand60orespectively.Findthedistance(inmeters)betweenthetwomen.Solution:

ThesituationisdepictedinthefigurewithCDrepresentingthetowerandABbeingthedistancebetweenthetwomen.FortriangleACD,tanA=tan60o=CD/AD.SimilarlyfortriangleBCD,tanB=tan45o=CD/DB.ThedistancebetweenthetwomenisAB=AD+DB=(CD/tan60o)+(CD/tan45o)=(28/√3)+(28/1)=44.17m.

Example43:-Twomenonthesamesideofatallbuildingnoticetheangleofelevationtothetopofthebuildingtobe30oand60orespectively.Iftheheightofthebuildingisknowntobeh=50m,findthedistance(inmeters)betweenthetwomen.Solution:

Inthefigure,AandBrepresentthetwomenandCDthetallbuilding.tanA=tan30o=DC/AC=h/AC;andtanB=tan60o=DC/BC=h/BC.NowthedistancebetweenthemenisAB=x=AC−BC=(h/tan30o)−(h/tan60o)=(50√3)−(50/√3)=57.73m.

Example44:-Apoleofheighth=40fthasashadowoflengthl=40.00ftataparticularinstantoftime.Findtheangleofelevation(indegrees)ofthesunatthispointoftime.Solution:

Page 204: Maths book part 2 obtains Rs. 1100 after lending out Rs.x at 5% per annum for 2 years and obtains Rs. 1800 after lending out Rs.y at 10% per annum for 2 years. Find the value of x

204

Inthefigure,BCrepresentsthepoleandABitsshadow.tanA=BC/AB=h/l=40/40.00=1.000Fromtrigonometrictables,wenotethattanA=1.000forA=45o.Hencetheangleofelevationofthesunatthispointoftimeis45o.

Example45:-Youarestationedataradarbaseandyouobserveanunidentifiedplaneatanaltitudeh=6000mflyingtowardsyourradarbaseatanangleofelevation=30o.Afterexactlyoneminute,yourradarsweeprevealsthattheplaneisnowatanangleofelevation=60omaintainingthesamealtitude.Whatisthespeed(inm/s)oftheplane?Solution:

Inthefigure,theradarbaseisatpointA.TheplaneisatpointDinthefirstsweepandatpointEinthesecondsweep.ThedistanceitcoversintheoneminuteintervalisDE.Fromthefigure,tanDAC=tan30o=DC/AC=h/AC.Similarly,tanEAB=tan60o=EB/AB=h/AB.Distancecoveredbytheplaneinoneminute=DE=AC−AB=(h/tan30o)−(h/tan60o)=(6000√3)−(6000/√3)=6928.20m.ThevelocityoftheplaneisgivenbyV=distancecovered/timetaken=DE/60=115.47m/s.

Example46:-cos2Ú[+ D

<-sin2 Ú

[+ D

<isequalto

Solution:-Weknowthatcos2A–sin2B=cos(A+B)cos(A–B),therefore

cos2Ú[+ D

<–sin2

Ú[− D

<=cos

Ú[+ D

<+ Ú

[− D

<×cos

Ú[+ D

<− Ú

[+ D

<

=cosÚ>cosx= $

<cosx.

Example47:-Ifsecθ-tanθ=½,thenθliesinwhichquadrant?Solution:-secθ–tanθ=½secθ+tanθ=2( sec2θ–tan2θ=1)secθ=5/4andtanθ=¾.cosθ=4/5andsinθ=4/5 ¾=3/5

Page 205: Maths book part 2 obtains Rs. 1100 after lending out Rs.x at 5% per annum for 2 years and obtains Rs. 1800 after lending out Rs.y at 10% per annum for 2 years. Find the value of x

205

Asbothsinθandcosθare+ve theangleθliesinthe1stquadrant.Example48:-Ifsin(A+B+C)=1,tan(A–B)=1/ 3,sec(A+C)=2,thenSolution:-Assin(A+B+C)=1A+B+C=90oAs,tan(A–B)=1/ 3&sec(A+C)=2A–B=30&A+C=60oFromtheabovestatementswecanconclude,A=60o,B=30o,C=0oORAssin(A+B+C)=1

A+B+C=900{Note:wecaneasilyruleoutoptions(1)&(3)asA+B+C>900}Nowcheckingoption(2)sin900=1,tan30=1/ 3,sec60=2,thussatisfy.

Example49:-Findthevalueoftan0×tan1×tan2×tan3×..….tan89?

Solution:-

Fromthetableweknowthattan0=0.Sonomatterwhatyoumultiplywithzero,finalanswerwillalwaysbezero.

Example50:-Findvalueoftan1xtan2xtan3x….x…tan88xtan89

Solution:-

youmakepairsofcomplimentarynumbers:1+89=90,2+88=90….Thereisonlyoneangleleftwhodoesn’tgetapair(45)

Soit’lllooklikethis=(tan1xta89)(tan2xtan88)x…xtan45

Ineachofthosepairs,youconvertonetanintoitscomplimentarycot.

=(tan1xtan(90-89))x(tan2xcot(90-88))…..x1;becausetan45=1

=(tan1xcot1)x(tan2xcot2)x…..x1=1becausetanandcotareinverseofeachother.

Example51:-.Findvalueoftan48xtan23xtan42xtan67

Solution:-

Page 206: Maths book part 2 obtains Rs. 1100 after lending out Rs.x at 5% per annum for 2 years and obtains Rs. 1800 after lending out Rs.y at 10% per annum for 2 years. Find the value of x

206

Whenyougetthistypemultiplicationchainquestion,you’vetofindoutthepairofcomplimentaryangles.Here48+42=90and23+67=90soI’llclubthemtogether

Inbothparts,we’llconvertanyonetanintocot,thentanxcot=1(becausethey’reinverseofeachother).

Finalanswer=1.

Example52:-Onthelevelground,theangleofelevationofthetopofthetoweris30o.Onmoving2metersnearer,theangleofelevationbecomes60o.Whatistheheightofthetower?Solution:-

�D=tan600

�DE<

=tan300

DE<D=3 x=1 h= 3

Example53:-TheangleofelevationofthetopofthetowerobservedfromeachofthethreepointsA,BandContheground,formingatriangleisthesameangleα,IfRisthecircum-radiusofthetriangleABC,thentheheightofthetowerisSolution:-Sincethetowermakesequalanglesattheverticesofthetriangle,thereforefootofthetowerisatthecircumcentre.FromΔOAP,wehavetanα=OP/A

Page 207: Maths book part 2 obtains Rs. 1100 after lending out Rs.x at 5% per annum for 2 years and obtains Rs. 1800 after lending out Rs.y at 10% per annum for 2 years. Find the value of x

207

orOP=OAtanAorOP=Rtan𝛼OP=OAtanAorOP=RtanαExample54:-Ifthelengthofachordofacircleisequaltothatoftheradiusofthatcircle,thentheanglesubtended(inradians)atthecentreofthecirclebythechordisSolution:-Inthiscase,thechordandthetworadiijoiningthecentretotheendsofthechordmakeanequilateraltriangle;sotheanglesubtendedis60o=π/3radians.Example55:-InatriangleABC,A=45°,thenthevalueof(1+cotB)(1+cotC)isequaltoSolution:-A=45oorB+C=180o–45=135otan(B+C)=–1ortanB+tanC=–1+tanBtanC

or $�|bº

+ $�|b¼

=-1+ $�|bº�|b¼

orcotB+cotC+cotBcotC=1or1+cotB+cotC+cotBcotC=2.or(1+cotB)(1+cotC)=2.orA=450A+B+C=1800B+C=180–45o=135oLetstake,B=900&C=450or(1+cotB)(1+cotC)=(1+cot900)(1+cot450)=(1+0)(1+1)=1×2=2Example56:-Ifcos(A-B)=3/5andtanAtanB=2,thena.cosAcosB=1/5b.sinAsinB=–2/5c.cos(A+B)=1/5d.sinAsinB=4/5Solution:-cos(A–B)=3/5andtanAtanB=2orcosAcosB+sinAsinB=3/5andsinAsinB=2cosAcosBor3cosAcosB=3/5 cosAcosB=1/5.Example57:-cos1°cos2°cos3°….cos179°isequalto

Page 208: Maths book part 2 obtains Rs. 1100 after lending out Rs.x at 5% per annum for 2 years and obtains Rs. 1800 after lending out Rs.y at 10% per annum for 2 years. Find the value of x

208

Solution:-Thegivenproductcontainsthefactorcos90o=0.Thusthevalueoftheproductbecomes0.Example58:-Ifsin2θ+3cosθ–2=0,thencos3θ+sec3θisequaltoSolution:-Givensin2θ+3cosθ–2=0cos2θ–3cosθ+1=0orcos2θ+1=3cosθorcosθ+ $

�|�Û=3

Cubingbothsides,wegetcos3θ+ $

�|�³Û+3(cosθ+ $

�|�Û)=27

orcos3θ+sec3θ=27–3×3=18

Example59:-.Findthevalueofcos18°/sin72°

Solution:-

Weknowthatcosandsinarecomplimentary.cosA=sin(90-A)

SoforCos18,Youcanwritecos18=sin(90-18)=sin72.Let’suseit

Cos18/sin72=sin72/sin72(becausecos18=sin72)=1(becausenumeratoranddenominatoraresamesothey’llcanceleach other.)

Example60:-Findvalueofsec70°xsin20°–cos20°xcosec70°

Solution:-

Ifyouconvertallfour(sin,cos,cosec,andsec)intotheircomplimentary(cos,sin,secandcosec)thenyou’llrunintoinfiniteloop.

InthepartA,ifIconvertsinintoitscomplimentarycos,thensecxcos=1becausesecandcosareinverseofeachother.SamewayinpartBifIconvertcosecintoitscomplimentarysec,thattoowillleadto1.Let’ssee

=1-1=0isthefinalanswer.

So,tanandcotarecomplimentary

1. tanA=cot(90-A)butnotvalidfor90degreesbecausetan90isnotdefined.2. cotA=tan(90-A)butthisisnotvalidfor0degreebecausecot0isnotdefined.

Thetanandcot’scomplimentaryrelationshipisalsoimportformultiplicationchainquestionsbecausetanandcotarealsoinverseofeachother.ThatistanAxcotA=tanAx1/tanA=1.

Page 209: Maths book part 2 obtains Rs. 1100 after lending out Rs.x at 5% per annum for 2 years and obtains Rs. 1800 after lending out Rs.y at 10% per annum for 2 years. Find the value of x

209

Exercise

Q1.IfCosA=1−2sin230°,thenfindvalueofA?

a.30 b.45 c.60 d.90

Q2.Ifcos60°=cos2A−sin230°,thenfindvalueofA?

a.30 b.45 c.60 d.90

Q3.IfsinA=2sin30°×cos30°,thenfindvalueofA?

a.30 b.45 c.60 d.90

Q4.IfsinA=2tan30°/(1+tan230°),thenfindvalueofA?

a.30 b.45 c.60 d.90

Q5.IfcosA=(1−tan230°)/(1+tan230°),thenfindvalueofA?

a.30 b.45 c.60 d.90

Q6.IfcosA=4cos330°−3cos30°,thenfindvalueofA?

a.30 b.45 c.60 d.90

Q7.Findvalueof(5cos260°+4sec230°−tan245°)/(sin230°+cos230°)?

a.67 b.12 c.67/12 d.12/67

Q8.Findvalueof3cos230°+sec230°+2cos0°+3sin90° −tan260°?

a.67 b.12 c.67/12 d.12/67

Q9.IfsinA=cosA,whatisthevalueof2tan2A+sin2A+1

Page 210: Maths book part 2 obtains Rs. 1100 after lending out Rs.x at 5% per annum for 2 years and obtains Rs. 1800 after lending out Rs.y at 10% per annum for 2 years. Find the value of x

210

a.2 b.7 c.7/2 d.2/7

Q10.WhatisthevalueofsinAcosB+cosAsinB,ifA=30°andB=60°

a.0 b.½ c.2 d.1

Q11.WhatisthevalueofcosAcosB−sinAsinB,ifA=30°andB=60°

a.0 b.½ c.2 d.1

Q12.secA=cosec60°,whatisthevalueof2cos2A−1

a.0 b.½ c.2 d.1

Q13.(tan10°xtan15°xtan75°xtan80°)

a.0 b.1 c.3 d.2

Q14.Whatisthevalueof5cos90°−cot30°+(sin60°/cos245°)

a.1 b.3 c.0 d.5

Q15.Findthevalueof(cos30°+sin60°)/(sin30°+cos60°+1)

a.½ b. 2 c. 3/2 d.2/ 3

Q16.Findthevalueoftan260°/(sin245°+cos245°)

a.3 b.1/2 c.2/3 d.1/3

Q17.Ifcot(A+B)=1/ 3andCot(A−B)= 3.FindthevaluesofAandB

a.30,60 b.60,30 c.15,45 d.45,15

Q18.Findthevalueofcos10°×cos20°×cos30°×………×cos90°

a.1 b.½ c.0 d.Notdefined.

Q19.IfsecA−cosecA=0thenfindvalueofsecA+cosecA,giventhatAisanacuteangle.

a.2 b.2 2 c.0 d.Notdefined.

Q20.Giventhat4Aisanacuteangleandsec4A=cosec(A−20),whatisthevalueofA?

a.22 b.23 c.24 d.25

Page 211: Maths book part 2 obtains Rs. 1100 after lending out Rs.x at 5% per annum for 2 years and obtains Rs. 1800 after lending out Rs.y at 10% per annum for 2 years. Find the value of x

211

Q21.Findvalueofcosec32-sec58?

a.1 b.2 c.3 d.0

Q22.Findvalueofcot12°xcot48°xcot52°xcot60°xcot78°

a.1 b.0 c. 3 d.1/ 3

Q23.(tan10°xtan45°xtan40°xtan50°xtan80°)

a.0 b.1 c.3 d.2

Q24.Cos48°xcosec42°+sin48°xsec42°

a.0 b.1 c.2 d.3

Q25.(cos70°/sin20°)+(cos59°xcosec31°)

a.1 b.2 c.3 d.0

Q26.(sec20°/cosec70°)+[(cos55°xcosec35°)/(tan5°xtan25°xtan45°xtan65°xtan85°)]

a.1 b.2 c.3 d.0

Q27.(sin70°/cos20°)+(cosec20°/sec70°)-(2cos70°xcosec20°)

a.1 b.2 c.3 d.0

Q28.Ifsin3A=cos(A−26),thenwhatisthevalueofA?giventhat3Aisanacuteangle.

a.21 b.23 c.24 d.29

Q29.Ifsec2A=cosec(A−42),thenwhatisthevalueofA?giventhat2Aisanacuteangle.

a.22 b.33 c.44 d.55

Q30.IfsinA=cos30°,whatisthevalueof2tan2A−tan45,giventhatAisanacuteangle.a.3 b.4 c.5 d.7

Q31.Ifsin7x=cos11xthenthevalueoftan9x+cot9xisa.1b.2c.3d.4

Q32.Ifcot2A=tan(A+6),findthevalueofA.giventhat2AandA+6areacuteangles?a.18 b.28 c.30 d.noneofabove

Q33.tan10×tan20×tan880×tan890

Page 212: Maths book part 2 obtains Rs. 1100 after lending out Rs.x at 5% per annum for 2 years and obtains Rs. 1800 after lending out Rs.y at 10% per annum for 2 years. Find the value of x

212

a.0 b.1 c.3 d.1/ 3Q34.tan40×tan430×tan470×tan860a.0 b.1 c.3 d.1/ 3

Q35.Ifõö÷ θEøùõθõö÷ θWøùõθ

=2thenthevalueofsinθisa.2/ 3 b. 3/2 c.1/2 d.1

Q36.Ifcosec390=xthenthevalueof$

�|���;?$°+sin239°+tan251°−

$�z9;?$°���;C_°

is

a. 𝑥< − 1 b. 1 − 𝑥< c.𝑥< − 1 d.1 − 𝑥<Q37.IftanA=2/3findthevalueof(3sinA+4cosA)/(3sinA–4cosA)a.-3 b.-4 c.-5 d.NoneoftheseQ38.sin21+sin23+sin25+sin27+….+sin289=a.22.5 b.23 c.24 d.25Q39.sin4A−cos4A=a.1 b.0c.sin2A−cos2Ad.tan2A Q40.[(secA−tanA)(secA+tanA)]+[(cosecA−cotA)(cosecA+cotA)]a.1 b.0 c.2 d.1/2

Q41.Thevalueof$

$E�|b;¸+ $

$Eby9;¸

a.2b.1c.1/4d.1/2Q42.IfsinA=1/3thencosAcosecA+tanAsecA=

a.$` <EC[

b.$` <E?

[ c.7 d.

$` <ECX

Q43.IfsinA=3/5,thenfindthevalueof4tanA+3sinAisequaltoa.6cosA b.6secA c.6secA d.tanAQ44.IfcosA=0.96then $

�z9¸+ $

by9¸isequalto

a.0.98b.2c.4d.7Q45.Ifsec𝜃-cos𝜃=3/2(𝜃isapositiveacuteangle),thensec𝜃isequaltoa.2 b.–1/2 c.0 d.1/2 Q46.Thevalueof[1+sin60o+sin230o+sin260o+sin445o][cos30o−sin60o]isa.1 b.13/12 c.12/13 d.0

Page 213: Maths book part 2 obtains Rs. 1100 after lending out Rs.x at 5% per annum for 2 years and obtains Rs. 1800 after lending out Rs.y at 10% per annum for 2 years. Find the value of x

213

Q47.Iftan2𝜃tan4𝜃=1thenthevalueoftan3𝜃=a.1/√3 b.0 c.1 d.√3Q48.Iftan22$

<°=xthencos67$

a. $$ED;

b. D$ED;

c.𝑥 d. 1 + 𝑥< Q49.Ifcos43°=

DD;E{;

,thenthevalueoftan47°

a.D

D;E{; b. {

D;E{; c.{

D d.D

{

Q50.úûø ÛEby9Ûúûø ÛWby9Û

=<E C<W C

thenthevalueof𝜃inthecircularmeasurewillbe

a.𝜋/6 b.𝜋/4 c.𝜋/12 d.𝜋/3Q51.Iftan𝜃+cot𝜃=2where0<𝜃<900;findthevalueoftan17𝜃+cot19𝜃.a.1 b.2 c.3 d.4Q52.sin22+sin24+sin26+sin28+………+sin288=a.22.5b.23c.24d.22Q53.tanA/sinA=a.cosecA b.sinA c.secA d.1/sinAQ54.(sinA/tanA)+cosA=a.2secA b.secA c.2cosA d.2cosecAQ55.tan10×tan20×…………………….tan880×tan890=a.0b.1c.2d.3Q56.Iftan(x+y)tan(x-y)=1thenthevalueoftanx=a.1/√3b.0c.1d.√3Q57.1/(tan10×tan20×…………………….tan1880×tan1890)=a.0b.1c.2d.3Q58.2cosec2230cot2670–sin2230-sin2670–cot2670=a.0b.1c.sec2230d.tan2230Q59.Thelengthoftheshadowofaverticaltoweronlevelgroundincreasesby10mwhenthealtitudeofthesunchangesfrom450to300.Thentheheightofthetowerisa.10 3b.5 3c.10( 3 + 1)d.5( 3+1)

Page 214: Maths book part 2 obtains Rs. 1100 after lending out Rs.x at 5% per annum for 2 years and obtains Rs. 1800 after lending out Rs.y at 10% per annum for 2 years. Find the value of x

214

Q60.If5tan𝜃=4then?�z9ÛWC�|�Û?�z9ÛE<�|�Û

=a.1/3b.2/3c.1/4d.1/6Q61.If2(cos2𝜃−sin2𝜃)=1(𝜃isapositiveacuteangle),thencot𝜃isequaltoa. 3b.− 3c.1/ 3d.1Q62.IftanC=11,thenfindvalueofsin2C+cos2Cisequaltoa.2 b.0 c.1 d.1/2Q63.InarightangletriangleABC,rightangledatB,theratioofABtoACis1: 2then2tanA/(1-tan2A)isequalsto:a.2 b.1 c.3 d.undefinedQ64.IfSinA=(a2-b2)/(a2+b2);thentanA=a.a2+b2/2ab b.2ab/(a2-b2) c.a2-b2/2ab d.tanAQ65.cos(90o−A)/sin(90o−A)=a.tanA b.cotA c.tan(90o−A) d.sec(90o−A)Q66.[cosec2A−1]cos(90o−A)/sin(90o−A)=a.tanAb.sinAc.cotA d.cosAQ67.cotA[cos(90o−A)/sin(90o−A)]=a.tanA b.cot2A c.tan2Ad.1Q68.Thevalueoftan45o−cos45osin45oisa.1/2 b.1c.0 d.3/4Q69.sin2A−sec2A+cos2A+tan2A=a.1 b.0 c.cotA d.cosecAQ70.cotAtanA=a.sinA b.cosA c.sinAcosA d.1Q71.Findthevalueofsin21+sin22+sin288+sin289a.3 b.2 c.3 d.noneoftheseQ72.Atreebreaksduetostormandthebrokenpartbendssothatthetopofthetreetouchesthegroundmakinganangle30°withit.Thedistancebetweenthefootofthetreetothepointwherethetoptouchesthegroundis8m.Findtheheightofthetree.a.8 b.16 c.8 3 d.NoneoftheseQ73.Upperpartofatreebrokenoverbythewindmakesanangleof45°withtheground,andthehorizontaldistancefromthefootofthetreetothepointwherethetopofthetreetouchesthegroundis12m.Findtheheightofthetreebeforeitwasbroken.

Page 215: Maths book part 2 obtains Rs. 1100 after lending out Rs.x at 5% per annum for 2 years and obtains Rs. 1800 after lending out Rs.y at 10% per annum for 2 years. Find the value of x

215

a.12m b.12+12/√3 c.12(1+√3) d.NoneoftheseQ74.Fromthetopofa7mhighbuilding,theangleofelevationofthetopofacabletoweris60oandtheangleofdepressionofthefootofthetoweris30o.Findtheheightofthetower.a.15 b.28 c.32 d.NoneoftheseQ75.Amanstandingonthedeckofaship,whichis16mabovethewaterlevel,observetheangleofelevationofthetopofcliffas60°andtheangleofdepressionofthebaseofthecliffas30°.Calculatethedistanceoftheclifffromtheshipandtheheightofthecliff.a.16√3m,h=64mb.20√3m,h=32mc.15√3m,h=64md.noneoftheseQ76.Anaeroplanewhenflyingataheightof5000mfromthegroundpassesverticallyaboveanotheraeroplaneataninstantwhentheanglesoftheelevationofthetwoplanesfromthesamepointonthegroundare60°and45°respectively.Findtheverticaldistancebetweentheaeroplanesattheinstant.a.2116.5 b.2115 c.2113.5 d.noneoftheseQ77.Ifdistancebetweentwopillarsoflength16&9misxmeters.Iftwoangleofelevationoftheirrespectivetopfromapointongroundofotherarecomplementarytoeachother,thenvalueofxisa.7 b.16 c.12 d.9Q78.Evaluate:sin225o+sin265o.a.2 b.0 c.1 d.-1 Q79.Theangleofelevationofthetopofatower30mhigh,fromtwopointsonthelevelgroundonitsoppositesidesare45degreesand60degrees.Whatisthedistancebetweenthetwopoints?a.30 b.51.96 c.47.32 d.81.96Q80.Theangleofelevationofthetopofatowerfromapointontheground,whichis30mawayfromthefootofthetower,is30°.Findtheheightofthetower.a.10 b.10 3 c.11.32 d.41.96Q81.Fromapointontheground,theanglesofelevationofthebottomandthetopofatransmissiontowerfixedatthetopofa20mhighbuildingare45°and60°respectively.Findtheheightofthetower.a.20( 3 − 1) b.20 3 c.20/ 3 d.41.96Q82.Aladder15mlongjustreachesthetopofaverticalwall.Iftheladdermakesanangleof60°withthewall,findtheheightofthewall.a.7.5√3 b.7.5 c.5√3 d.Noneofthese

Page 216: Maths book part 2 obtains Rs. 1100 after lending out Rs.x at 5% per annum for 2 years and obtains Rs. 1800 after lending out Rs.y at 10% per annum for 2 years. Find the value of x

216

Q83.Apole12mhighcastsashadow4√3mlongontheground.Findtheangleofelevationa.30° b.60° c.90° d.NoneoftheseQ84.Theangleofelevationofthetopofatowerfromapointonthegroundis30°ifonwalking30mtowardsthetower,theangleofelevationbecomes60°.Findtheheightofthetower.a.15√3 b.7√3 c.5√3 d.NoneoftheseQ85.Theangleofelevationoftheaeroplanefromapointonthegroundis60°.After15secondsflight,theangleofelevationchangesto30°.Iftheaeroplaneisflyingataheightof1500√3m.Findthespeedoftheplanea.200m/s b.450m/s c.250m/s d.NoneoftheseQ86.Anobserver1.5mtallis20.5mawayfromatower22mhigh.Determinetheangleofelevationofthetopofthetowerfromtheeyeoftheobserver.a.75° b.60° c.45° d.NoneoftheseQ87.If3sin2α+7cos2α=4,thenthevalueoftanα-is(where0<α<90°)a. 3b. 6c. 2d. 5Q88.Iftanθ–cotθ=0,0°<θ<90°,thevalueof(sinθ–cosθ)isa.1b.2c.–2d.0Q89.If0<θ<$

<,thensinθ+cosθisalways

a.greaterthan1b.lessthan1c.equalto1d.greaterthan2Q90Ifsec2θ+tan2θ=7,thenthevalueofθ,when0°<θ<90°,isa.60°b.30°c.0° d.90°Q91.If�z9ÚE�|�Ú

�z9ÚW�|�Ú=3,thenthevlaueofsin4θ–cos4θis

a.$?b.<

?c.C

? d.>

?

Q92If2cosθ–sinθ= $

<,(0°<θ<90°)thevalueof2sinθ+cosθis

a. $<b. 2c. C

< d. <

C

Q93.If0°<θ<90°andsinθ+cosθ=$X

$Cthenthevlaueofsinθ–cosθis

a. C$Cb.$%

$Cc. ?

$C d. X

$C

Q94.If�z9ÛE�|�Û

�z9ÛW�|bÛ=7,thenthevalueoftanθisequalto:

a.<Cb.>

C c.$

C d. X

$C

Page 217: Maths book part 2 obtains Rs. 1100 after lending out Rs.x at 5% per annum for 2 years and obtains Rs. 1800 after lending out Rs.y at 10% per annum for 2 years. Find the value of x

217

Q95.Ifby9ÛE�|bÛ

�|bÛW�|bÛ=2,(0<θ<90°),thenthevalueofsinθis

a. <Cb. C

< c.$

< d.1

Q96.Ifsec(4θ–50°)=cosec(50°–θ),thenthevalueofθ,when0°<θ <90°,isa.33$

Cb.18°c.3$

Cd.30°

Q97.Ifcosθ+secθ= 3,thenthevalueofcos3θ+sec3θisa.–1b. 3 c.0 d.1 Q98.Themaximumvalueof24sinθ+7cosθisa.24b.25c.7 d.17 Q99.Iftan2θ.tan4θ=1,thenthevalueoftan3θisa.$

<b.2c.0 d.1

Q100.In"ABC,∠AisarightangleandADisperpendiculartoBC.IfAD=4cm,Bc=12cm,thenthevalueof(costBcotC)isa.4b.C

<c.6 d.3

Q101.Ifα+β=90°andα:β=2:1,thenthevalueofsinα:sinβis:a. 1:1 b. 2:1c. 3:1d.2:1 Q102.Ifsecθ+tanθ=2,thensecθisequaltoa.?

<b.?

> c. X

>d.X

<

Q103.Thevalueofθ[0°<θ<90°]forwhich �|�Û

$W�z9Û+ �|�Û

$E�z9Û=4is

a.45°b.60°c.30°d.noneofthese Q104.If2ycosθ=xsinθand2xsecθ–ycosecθ=3,thentherelationbetweenxandyisa.2x2+y2=2b.x2+4y2=4c. x2+4y2=1 d.4x2+y2=4 Q105.Inaright-angledtriangleABC,AB=2.5cm,cosB=0.5,∠ACB=90°.ThelengthofsideAC,incm,isa.?

>3 b. ?

$`3c.5 3d.?

<3

Q106.Iftanθ–cotθ=aandcosθ–sinθ=b,thenthevalueof(a2+4)(b2–1)2is:a.1b.2c.3d.4

Page 218: Maths book part 2 obtains Rs. 1100 after lending out Rs.x at 5% per annum for 2 years and obtains Rs. 1800 after lending out Rs.y at 10% per annum for 2 years. Find the value of x

218

Q107.Ifx=cosecθ=sinθandy=secθ–cosθ,thenthevalueofx2y2(x2+y2+3)is:a.0b.1c.2d.3 Q108.Inaright-angledtriangleABC,∠BistherightangledandAC=2 5cm.IfAB–BC=2cm,thenthevalueof(cos2A–cos2C)is:a.C

?b.`

?c. C

$%d.<

?

Q109.If0<θ <

Ú<,2ycosθ=xSinθand2xsecθ–ycosecθ=3,thenthevalueofx2+4y2is:

a.1b.2c.3d.4 Q110.Ifcosθ.cosec23°=1,thevalueofθisa.23°b.37°c.63°d.67° Q111.Ifsin(3x–20°)=cos(3y+20°),thenthevalueof(x+y)isa.20°b.30°c.40°d.45° Q112.Ifcosθ=>

?,thenthevalueof�|���Û

$E�|bÛis

a.X?b.<

Xc.?

Xd.>

X

Q113.Ifsin2θ=$

<,thenthevalueofcos(90–θ)is

a.1b.$<c. C

<d. $

<

Q114.If�z9Û

�|�Û+ �|�Û

�z9Û=2with0<θ<90°,thenwhatisθequalto?

a.30°b.45°c.60°d.75° Q115.If5tanθ=4,thenthevalueof?�z9ÛWC�|�Û

�z9ÛEC�|�Ûis:

a.$Xb.<

Xc.?

Xd.<

?

Q116. Ifx,yarepositiveacuteangles,x+y<90°andsin(2x–20)=cos(2y+20°),thenthe

valueofsec(x+y)is

a. 2𝑏. $<c. C

<d. $

<

Q117.If(a2+b2)sinθ+2abcos=a2+b2,thenthevalueoftanθisa.$

<(a2–b2)b. $

<y�(a2–b2)c.$

<(a2+b2)d. $

<y�(a2+b2)

Q118.IfA+B=90°,thenthevalueofsec2A+sec2B–sec2A.sec2Bis:a.0b.1c.2d.3

Page 219: Maths book part 2 obtains Rs. 1100 after lending out Rs.x at 5% per annum for 2 years and obtains Rs. 1800 after lending out Rs.y at 10% per annum for 2 years. Find the value of x

219

Q119.Ifcosθ+secθ= 3thenthevalueofcos3θ+sec3θis:a.0b.1c.–1 d. 3 Q120.Ifsin(x+y)=cos[3(x+y)]thenthevalueoftan[2(x+y)]isa. 3b.1c.0d. $

C

Q121.IfX+1/X=2thenthevalueofcos2θ+sec2θis:a.0b.1 c.2 d. 3 Q122.Theleastvalueof4cosec2a+9sin2ais:a.14b.10c.11d.12 Q123.Ifsecθ–cosecθ=0,thenthevalueof(secθ+cosecθ)is:

a. C<b. <

Cc.0d.2 2

Q124.Ifcosx=sinyandcot(x–40)=tan(50–y)thenthevalueofxandyare:a.1b.1c.1d.2 Q125.Ifxsin3θ+ycos3θ=sinθcosθ,andxSinθ–ycosθ=0thenvalueof(x2+y2)a.1b.sinθ–cosθc.sinθ+cosθd.0 Q126.Ifsinθ+cosθ= 2cos(90–θ),thencotθisa. 2+1b.0c. 2d. 2–1 Q127.Ifsecθ+cosec(90– θ)=4,(0<θ<90°)thenthevalueoftanθis:a. $

Cb.1c. 3d. $

<

Q128.If 2.cos(5x+5°)=cot45°,thenthevalueofxindegreeis:a.10b.8c.11d.0 Q129.Ifsinθ–cosθ=0,findthevlaueofsin Ú

<− 𝜃 +cos Ú

<+ 𝜃

a.0b.1c. 2d.2 2

Q130.Ifsinθ=y;W$y;E$

,thenthevalueofsecθ+cosθwillbe:

a. 2ab.ac. y<d. y

y;E$

Q131.Ifsin(x–2y)=cos(4y–x),thenthevalueofcot2yis:a.0b.1c. $

Cd.undefined

Page 220: Maths book part 2 obtains Rs. 1100 after lending out Rs.x at 5% per annum for 2 years and obtains Rs. 1800 after lending out Rs.y at 10% per annum for 2 years. Find the value of x

220

Q132.Iftan Ú

<− Û

<= 3,valueofcosθis:

a.0b. $<c.$

<d.1

Answers1.C 2.A 3.C 4.C 5.C 6.D 7.C 8.C 9.C 10.D11.A 12.B 13.B 14.C 15.A 16.A 17.D 18.C 19.B 20.A21.D 22.D 23.B 24.C 25.A 26.B 27.D 28.D 29.C 30.C31.B 32.B 33.B 34.B 35.B 36.C 37.A 38.A 39.C 40.C41.B 42.A 43.A 44.D 45.A 46.D 47.C 48.B 49.C 50.D51.B 52.D 53.C 54.C 55.B 56.C 57.A 58.C 59.D 60.D61.C 62.C 63.D 64.B 65.A 66.C 67.D 68.A 69.B 70.D71.B 72.C 73.D 74.B 75.A 76.C 77.C 78.C 79.C 80.B81.A 82.B 83.B 84.A 85.A 86.C 87.A 88.D 89.A 90.A91.C 92.C 93.D 94.B 95.B 96.D 97.C 98.B 99.D 100.D101.C 102.B 103.B 104.B 105.A 106.D 107.B 108.A 109.D 110.D111.B 112.C 113.B 114.B 115.A 116.A 117.B 118.A 119.A 120.B121.C 122.D 123.D 124.B 125.A 126.D 127.C 128.B 129.A 130.B131.D 132.C

Page 221: Maths book part 2 obtains Rs. 1100 after lending out Rs.x at 5% per annum for 2 years and obtains Rs. 1800 after lending out Rs.y at 10% per annum for 2 years. Find the value of x

221